hyperguides 2015 .tumor

209
1. In evaluating the radiographs of a patient with Paget’s disease, which of the following would be suggestive of malignant change: (A ) Enlargement of the bone (B ) Coarsened trabeculae (C ) Marked bowing (D ) Multiple lucent areas (E ) Cortical bone destruction Explanation: Paget’s disease is a remodeling disease in which the normal cortices are resorbed and replaced with remodeled cortices. In this condition, there are coarsened trabeculae, lucent areas, and bone enlargement. Bowing may occur and varies from mild to marked. Cortical bone destruction and the presence of a soft tissue mass are common findings in patients where Paget’s disease has undergone malignant change. Both MRI and CT scans are excellent modalities to detect malignant change. As the remodeling occurs, there should not be any destruction of the cortical bone 2. Which of the following benign bone lesions is the most likely to recur following curettage: (A ) Chondroblastoma (B ) Enchondroma (C ) Non-ossifying fibroma (D ) Osteoid osteoma (E ) Giant cell tumor

Upload: ahmad-shakir

Post on 13-Apr-2016

2 views

Category:

Documents


0 download

DESCRIPTION

Hyperguides 2015 .Tumor

TRANSCRIPT

Page 1: Hyperguides 2015 .Tumor

1. In evaluating the radiographs of a patient with Paget’s disease, which of the following would be suggestive of malignant change:(A) Enlargement of the bone(B) Coarsened trabeculae(C) Marked bowing(D) Multiple lucent areas(E) Cortical bone destruction

Explanation:

Paget’s disease is a remodeling disease in which the normal cortices are resorbed and replaced with remodeled cortices. In this condition, there are coarsened trabeculae, lucent areas, and bone enlargement. Bowing may occur and varies from mild to marked.

Cortical bone destruction and the presence of a soft tissue mass are common findings in patients where Paget’s disease has undergone malignant change. Both MRI and CT scans are excellent modalities to detect malignant change. As the remodeling occurs, there should not be any destruction of the cortical bone

2. Which of the following benign bone lesions is the most likely to recur following curettage:(A) Chondroblastoma(B) Enchondroma(C) Non-ossifying fibroma(D) Osteoid osteoma(E) Giant cell tumor

Explanation:

Of all the benign tumors listed in the possible answers, giant cell tumors are the most prone to recur locally. Historical studies have shown local failure rates as high as 30% to 40% following curettage and grafting. The most modern technique employs curettage and reconstruction with methyl methacrylate. The local failure rate with this technique is 5% to 20%. Some centers also use adjuvants such as phenol, liquid nitrogen, and hydrogen peroxide.

The other lesions have low rates of recurrance:

Chondroblastoma5%-10%

Enchondroma <5%

Non-ossifying fibroma <5%

Page 2: Hyperguides 2015 .Tumor

Osteoid osteoma <5%

3. Which of the following describes the histologic features of conventional chordoma:(A) Physaliferous cells and high-grade pleomorphic spindle cells(B) Lobular growth pattern and physaliferous cells(C) Uniform population of small blue cells(D) Organoid pattern of epithelial cells in a fibrous background(E) Sheets of cells with eccentric nucleus and peripheral nuclear chromatin pattern

Explanation:

The principal histologic features of conventional chordomas are:

Lobular growth pattern Nests of solid cells with a vacuolated cytoplasm Physaliferous cells

Dedifferentiated chordomas have the typical features: lobular growth pattern, physaliferous cells, and a myxoid background. The other answers describe specific tumors:

Uniform population of small blue cells — Ewings tumor Organized pattern of epithelial cells in a fibrous background — Metatastic carcinoma Sheets of cells with eccentric nucleus, peripheral chromatin pattern — Myeloma

4. A 17-year-old man has a 4-month history of severe pain in his right distal tibia. The pain is especially prominent at night. He has difficulty sleeping and concentrating at school. The lateral radiograph of his right distal tibia is shown in Figure 1. A CT and axial T1-weighted MRI scan are shown in Figures 2 and 3. The stage of this tumor according to the system of the Musculoskeletal Tumor Society:

Figure 1 Figure 2

Page 3: Hyperguides 2015 .Tumor

Figure 3(A) Stage 1(B) Stage 2(C) Stage 3(D) Stage II(E) Stage III

Explanation:

From the information provided in the question:

The lesion is symptomatic. The CT and MRI scan are diagnostic of an osteoid osteoma.

Although osteoid osteomas may cause severe pain, they do not grow and destroy bone structure. Accordingly, this lesion is an active benign lesion - Stage 2.

The Surgical Staging System of the Musculoskeletal Tumor Society is a useful system to both predict prognosis and plan treatment. The system for benign lesions is divided into three groups: inactive (latent), active, and aggressive.

Inactive (latent) Stage 1

Refers to lesions that are not causing pain and show no evidence of active growth. Stage 1 lesions are generally treated with observation only.

Active Stage 2

Refers to lesions which are causing pain or some form of disability. If a lesion has weakened the structure of the bone such that fracture may occur, the lesion would also be considered a Stage 2 lesion.

Aggressive Stage 3

Refers to lesions which are large, have broken into the soft tissues, or have caused a pathologic fracture. These lesions are usually prone to local recurrence and have the potential of causing a major problem for the patient.

5. Allografts are commonly used to reconstruct large segment defects. The rate of deep infection following allograft reconstruction is:(A) 1% to 2%(B) 2% to 5%(C) 10% to 15%

Page 4: Hyperguides 2015 .Tumor

(D) 25% to 50%(E) 50% to 75%

Explanation:

Allografts and custom prostheses are the 2 major methods to reconstruct defects following resection of bone tumors. The major complications following allograft reconstruction include: infection, fracture, breakdown of the articular cartilage, and non-union.Infection is the most devastating complication and occurs in about 10% to 15% of patients.

6. The most common locations of unicameral bone cysts are:(A) Proximal femur and distal(B) Proximal humerus and distal femur(C) Proximal humerus and proximal femur(D) Proximal humerus and distal tibia(E) Proximal femur and distal humerus

Explanation:

Unicameral bone cysts most commonly occur in the proximal humerus. The proximal femur is the second most common site.

Unicameral bone cysts have a typical radiographic appearance:

Metaphyseal Purely lytic Expand the bone in a symmetric fashion Often border the growth plate May have trabeculations in them once they have fractured

7. Which of the following describes the most common mineralization pattern of high-grade intramedullary osteosarcomas:(A) Scattered stippling(B) Ring-like(C) Cloudy opacities that vary in size, shape, and density(D) Flocculent(E) Stippling with multiple rings and arcs

Explanation:

It is important to be able to recognize the mineralization pattern of lesions in both bone and soft tissue. Mineralization may occur in osteoid and hyaline cartilage. One can broadly say that the mineralization is bone formation, cartilage calcification, and amorphous calcification.The following table is useful to remember when analyzing radiographs:

Page 5: Hyperguides 2015 .Tumor

Mineralization pattern Lesions

Hyaline cartilage calcification

Fine stipples*Rings, arcs, commas

EnchondromaLow-grade intramedullary chondrosarcomaDedifferentiated chondrosarcomaOsteochondromaSecondary surface chondrosarcomaSynovial chondromatosis

Bone formation Cloud- or ivory-like (may be trabecular pattern in the bone)

Bone islandsHeterotopic ossificationOsteoblastomaOsteoid osteomaHigh-grade intramedullary osteosarcomaLow grade intramedullary osteosarcoma

Amorphous calcification

Nodular and flocculent†(multiple areas in most lesions)

Tumorous calcificationCalcific tendinitis

* If one were to paint fine stippling, one would use a brush with a fine tip.† If one were to paint the amorphous calcifications, one would use the main body of the brush, not a fine tip.

8. In which of the following bone tumors is curettage performed as the primary method of treatment:(A) Dedifferentiated chondrosarcoma(B) Malignant fibrous histiocytoma(C) Adamantinoma(D) Giant cell tumor(E) Well-differentiated intramedullary osteosarcoma

Explanation:

There are four different surgical margins that can be achieved in oncologic surgery:

Intralesional: The plane of surgery goes through the tumor. Marginal: The plane of surgery goes through the reactive zone of the lesion (the reactive

zone contains edema, tumor cells, fibrous tissue, and inflammatory cells). Wide: The plane of dissection goes through normal tissue. Radical: The entire anatomic compartment of the lesion is removed.

Curettage is reserved for benign tumors or metastatic disease, in which radiation will be used to control whatever tumor cells have been left benign. Giant cell tumors of bone are treated with curettage. The curettage is extensive with exteriorization (removing the cortex off the tumor), followed by curetting the tumor with hand instruments and power tools. There is often a large defect

Page 6: Hyperguides 2015 .Tumor

left following the curettage.This defect is usually reconstructed with methyl methacrylate.

The following lesions are commonly treated with curettage:Giant cell tumorChondroblastomaAneurysmal bone cystOsteoid osteoma

In contrast, malignant bone tumors are usually removed with a wide or radical surgical margin.

9. Which of the following radiographic descriptions describes the features of sarcoma occurring in Paget’s disease:(A) Irregular increased density with a cotton wool-like appearance with interspersed radiolucency(B) Bone is enlarged with prominent deformity(C) Large areas of bone sclerosis(D) Patchy sclerosis(E) Osteolytic area with cortical bone destruction

Explanation:

Paget's disease has very characteristic radiographic patterns. The disease is divided into three phases: lytic, mixed lytic-sclerotic, and sclerotic phase.

In the sclerotic and mixed phase, the radiographic findings are easy to recognize:

Sclerotic phaseIrregular increased density with cotton wool-like appearance with interspersed radiolucencyBones are usually enlargedThere may be prominent deformityIn long bones, the disease begins at one end and proceeds to the other

Intermediate phasePatchy sclerosis

Lytic phaseIn the lytic phase, there may be purely radiolucent findingsThere is often mild expansion of the boneThe cortices are thinned but intact

When a sarcoma has developed within the bone, there is usually cortical bone destruction and a major lytic focus. Periosteal reaction is usually absent and there is often a large soft tissue mass.

10. A 55-year-old man has a three month history of increasing elbow pain. His past medical history is significant for coronary artery disease. He has a 30-pack per year smoking history. Initial radiographs of his elbow were normal. The plain radiograph of his elbow is shown three months later in Slide 1 and a biopsy of the destructive lesion is shown in Slide 2.The most likely diagnosis is:

Page 7: Hyperguides 2015 .Tumor

Slide 1 Slide 2(A) Metastatic adenocarcinoma(B) Multiple myeloma(C) Lymphoma(D) Osteomyelitis(E) Paget disease

Explanation:

The plain radiograph shows a destructive lesion in the metaphysis of the distal humerus with expansion of the medial cortex. This appearance is characteristic of a bone malignancy. The differential diagnosis includes metastatic bone disease, myeloma, lymphoma, primary mesenchymal tumors (chondrosarcoma and malignant fibrous histiocytoma).

The histology section shows epithelial cells forming glands in a background of fibroblasts. This patient has metastatic carcinoma.

11. A 20-year-old man bangs his shoulder while playing ice hockey one week ago, and now he has pain and a mass over the deltoid area. A plain radiograph is shown (Slide 1). An open biopsy is performed and the low- and high-power sections are presented (Slide 2 and Slide 3).The most likely diagnosis based on the radiographic features and biopsy is:

Page 8: Hyperguides 2015 .Tumor

Slide 1 Slide 2

Slide 3

(A) Osteochondroma(B) Low-grade intramedullary chondrosarcoma(C) Heterotopic ossification(D) Parosteal osteosarcoma(E) Periosteal chondroma

Explanation:

Periosteal chondroma is an uncommon surface cartilage tumor. The lesion is usually small (3 cm to 5 cm) and rests in a saucer-shaped depression on the surface of the bone. Characteristic locations include the proximal humerus and distal femur. Histologically, the lesion is composed of lobules of cartilage. Although this tumor is benign, the lesion is usually hyper cellular and there may be prominent atypia of the cartilage cells. The treatment is excion with a marginal margin.

12.A 16-year-old girl has a painless bump behind her knee. However, she does have pain if she her leg is hit while playing soccer. On physical examination there is a firm lump behind the knee. A plain radiograph of the knee is shown (Slide 1).The most likely diagnosis based on the history, physical examination and plain radiograph is:

Page 9: Hyperguides 2015 .Tumor

Slide 1(A) High-grade intramedullary osteosarcoma(B) Parosteal osteosarcoma(C) Periosteal osteosarcoma(D) Osteochondroma(E) Periosteal chondroma

Explanation:

Osteochondromas have a characteristic radiographic appearance. The lesion is a surface tumor and the medullary cavity of the normal bone flows into the osteochondroma. Some authors call this "cortical sharing." Osteochondromas may have a well defined stalk (pedunculated osteochondroma) or they may have a broad base as in this case (sessile osteochondroma). This diagnosis can be made with the plain radiographs without histologic confirmation.

13. Paget disease is a primary disorder of:(A) Epiphyseal plate(B) Articular cartilage(C) Synovial membrane(D) Bone(E) Intravertebral disc

Explanation:

Paget disease is a primary disorder of bone. Paget disease is a remodeling disease of bone. There is abnormal increased activity of the osteoclast. The osteoclast resorbs the bone resulting in lysis and the osteoblast lays down new bone in thickened trabeculae and cortices. The treatment of Paget's disease of bone is stopping the osteoclasts. Anti-pagetic medications include the diphosphonate class of medications.

14.A 55-year-old man presented with a 12-month history of increasing low back pain. He experiences pain during the night and often has difficulty sitting. He has also noticed that in the last 9 months constipation has become an increasing problem. The plain radiographs and computerized tomography (CT) scan are shown (Slide 1 and Slide 2). Low- and high-

Page 10: Hyperguides 2015 .Tumor

power histologic sections are also presented (Slide 3 and Slide 4).The most likely diagnosis is:

Slide 1 Slide 2

Slide 3 Slide 4

(A) Metastatic adenocarcinoma(B) Multiple myeloma(C) Malignant fibrous histiocytoma(D) Chordoma(E) Sacral insufficiency fracture

Explanation:

The classic midline bone destruction with an anterior soft tissue mass is present. The histology shows a lobular pattern of growth and the characteristic physaliferous cells can be seen on high power.

15. A 66-year-old man has a 4-month history of hip discomfort. The plain radiographs are shown in Slide 1 and a biopsy is shown in Slide 2. Which of the following tests would not be recommended to assess the extent of this condition (staging studies):

Page 11: Hyperguides 2015 .Tumor

Slide 1 Slide 2

(A) Bone marrow biopsy(B) Serum calcium level(C) Serum creatinine and blood urea nitrogen(D) Technetium bone scan(E) Skeletal survey

Explanation:

A destructive lesion in the pelvis may be secondary to five common processes in patients who are 40 to 80 years of age, including metastatic bone disease, multiple myeloma, lymphoma, chondrosarcoma, and malignant fibrous histiocytoma. Often a biopsy is necessary to establish the diagnosis. The biopsy specimen shows plasma cells. The plasma cells have these characteristics:

Eccentrically placed nucleus Peripheral clumping of the nuclear chromatin A perinuclear halo

The diagnosis is multiple myeloma. Staging is important and is performed with a skeletal survey, bone marrow biopsy, and chemistry studies, including kidney function and serum calcium determination.

Technetium bone scans have a high false-negative rate and are not used in the staging process. Neither computerized tomography scans nor brain scans are routinely used.

16. A 50-year-old woman has had severe hip pain for 4 months. Her plain radiographs are shown in Slide 1. The technetium bone scan, computerized tomography scan, and coronal T1 and T2 weighted magnetic resonance imaging scans are shown in Slides 2, 3, and 4 respectively. A needle biopsy is performed and shown in Slides 5 and 6. The most likely diagnosis is:

Page 12: Hyperguides 2015 .Tumor

Slide 1 Slide 2 Slide 3

Slide 4 Slide 5

Slide 6(A) Metastatic adenocarcinoma(B) Multiple myeloma(C) Lymphoma(D) Chondrosarcoma

Page 13: Hyperguides 2015 .Tumor

(E) Malignant fibrous histiocytoma

Explanation:

The plain radiographs show a purely lytic destructive lesion, which is poorly marginated. The technetium bone scan does not show any major uptake. The computerized tomography scan shows purely lytic bone destruction with breakthrough of the cortical bone. Complete destruction of the cortical bone is very suggestive of a malignancy. The magnetic resonance imaging scan shows a lesion that is homogenously low on T1 weighted images and high on T2 weighted images. One cannot make a definitive diagnosis based upon the radiographic features. The most common malignancies in this age group are:

Metastatic bone disease Multiple myeloma Lymphoma Chondrosarcoma Malignant fibrous histiocytoma

The biopsy specimen shows round epithelial appearing cells that are grouped in clusters. The cells are arranged in a fibrous background. The diagnosis is metastatic bone disease.

17. A 50-year-old woman has had severe hip pain for 4 months. Her plain radiographs are shown in Slide 1. The technetium bone scan, computerized tomography scan, and coronal T1 and T2 weighted magnetic resonance imaging scans are shown in Slides 2, 3, and 4 respectively. A needle biopsy is performed and shown in Slides 5 and 6. Which of the following would be the most appropriate treatment regimen:

Slide 1 Slide 2 Slide 3

Page 14: Hyperguides 2015 .Tumor

Slide 4 Slide 5 Slide 6

(A) Wide resection and allograft acetabular reconstruction(B) Internal hemi-pelvectomy alone(C) Preoperative chemotherapy followed by wide resection(D) Acetabular reconstruction followed by external beam irradiation(E) Hemi-pelvectomy

Explanation:

The plain radiographs show a purely lytic destructive lesion, which is poorly marginated. The technetium bone scan does not show any major uptake. The computerized tomography scan shows purely lytic bone destruction with breakthrough of the cortical bone. Complete destruction of the cortical bone is very suggestive of a malignancy. The magnetic resonance imaging scan shows a lesion that is homogenously low on T1 weighted images and high on T2 weighted images. One cannot make a definitive diagnosis based upon the radiographic features. The most common malignancies in this age group are:

Metastatic bone disease Multiple myeloma Lymphoma Chondrosarcoma Malignant fibrous histiocytoma

The biopsy specimen shows round epithelial-appearing cells that are grouped in clusters. The cells are arranged in a fibrous background. The diagnosis is metastatic bone disease.

The treatment of metatastic bone disease is either internal fixation with postoperative external beam irradiation or external beam irradiation alone. In this case, there is very extensive destruction and fracture is imminent. The treatment of choice would be acetabular reconstruction followed by external beam irradiation.

18. A 28-year-old woman has a 4-month history of increasing shoulder pain. Her plain radiograph is shown in Slide 1. A biopsy specimen is shown in Slides 2 and 3. Which of the following is the correct risk of pulmonary metastases for this lesion:

Page 15: Hyperguides 2015 .Tumor

Slide 1 Slide 2

Slide 3(A) 0%(B) 2% to 5%(C) 10% to 30%(D) 30% to 50%(E) 75%

Explanation:

The plain radiograph shows a destructive lesion. The lesion is purely lytic, expansile, and there is a pathologic fracture. The lesion has destroyed the metaphysis and extended into the epiphysis.

From a radiographic perspective, this lesion could be:

A telangiectatic osteosarcoma An aneurysmal bone cyst A giant cell tumor

The biopsy specimen shows a uniform distribution of multinucleated giant cells in a field of

Page 16: Hyperguides 2015 .Tumor

mononuclear cells. The diagnosis is giant cell tumor of bone.

The risk of pulmonary metastases in giant cell tumor of bone is 2% to 5%. This phenomenon is called benign metastasizing giant cell tumor. The mortality rate from these metastases is 10% to 20%. Spontaneous regression of the pulmonary lesions may occur.

19. An 18-year-old woman has a hard mass over her distal femur. The mass is removed, and the biopsy specimens are shown in Slides 1 and 2. The most likely diagnosis is:

Slide 1 Slide 2

(A) Parosteal osteosarcoma(B) Periosteal osteosarcoma(C) Periosteal chondroma(D) Osteochondroma(E) Heterotopic ossification

Explanation:

The biopsy specimen shows a lesion with a thin cartilage cap. Notice the blue cartilage matrix in the cartilage cap. The high power view shows the typical features of an osteochondroma with a linear arrangement of the chondrocytes. This lesion is an osteochondroma.

20. Myxoid soft tissue chondrosarcomas have which of the following chromosomal alterations:(A) Giant marker and ring chromosomes(B) Reciprocal translocation between chromosomes 12 and 16(C) Translocation between chromosomes 11 and 22(D) Reciprocal translocation of chromosomes X and 18(E) Translocation between chromosomes 9 and 22

Explanation:

Page 17: Hyperguides 2015 .Tumor

Many benign and malignant soft tissue tumors have specific chromosomal abnormalities. A few common abnormalities include:

Well-differentiated liposarcoma Giant marker and ring chromosomes Myxoid liposarcoma Translocation between chromosomes 12 and 16 Ewings/primitive neuroectodermal tumors Translocation between chromosomes 11 and 22 Synovial sarcoma Translocation between chromosomes X and 18 Myxoid chondrosarcoma Translocation between chromosomes 9 and 22

21. Which of the following describes the histopathology of tumoral calcinosis:(A) Malignant stromal cells with direct osteoid production(B) Lymphocytes between edematous muscle fibers and angiogenic proliferative tissue(C) Foreign-body granuloma reactions forming multilocular, cystic structures(D) Biphasic with epithelial and fibrous cells(E) Zonal pattern with mature bone at periphery and immature tissue at periphery

Explanation:

Tumoral calcinosis is a heritable condition that is characterized by periarticular metastatic calcification. Most patients are black, and the inheritance is usually autosomal recessive. Metastatic calcifications occur around joints and in the skin, marrow, teeth, and blood vessels. The periarticular masses may grow quite large and are attached to the fascia, but they are extra-articular. The masses may occur at the shoulder, hip, and elbow.

The histologic features are:

Foreign-body granuloma that react with multilocular, cystic structures Cysts with tough connective tissue walls Mature lesions filled with calcareous material (viscous milky fluid)

The other answers refer to:

Malignant stromal cells with direct osteoid production: osteosarcoma Lymphocytes between edematous muscle fibers and angiogenic proliferative tissue:

fibrodysplasia (myositis) ossificans progressiva Biphasic with epithelial and fibrous cells: synovial sarcoma Zonal pattern with mature bone at periphery and immature tissue at periphery:

heterotopic ossification (myositis ossificans)

22. Which of the following describes the histologic appearance of epithelioid sarcoma:(A) Storiform pattern of pleomorphic spindle cells and histiocytes(B) Herringbone pattern of spindle cells(C) Epithelial cells and spindle cells(D) Nodules with central necrosis and ovoid or polygonal cells with eosinophilic cytoplasm

Page 18: Hyperguides 2015 .Tumor

(E) Small blue cells in an alveolar pattern

Explanation:

There are several histologic features of epithelioid sarcoma:

Nodular growth with central necrosis Ovoid or polygonal cells with deeply eosinophilic cytoplasm

Histologically, the cells may lack pleomorphism and may be confused with granulomas, granuloma annulare, squamous cell carcinoma, necrotizing infectious granuloma, necrobiosis lipoidica, and rheumatoid nodules. 23. Which of the following forms of rhabdomyosarcoma has a poor prognosis:

(A) Botryoid(B) Spindle cell(C) Embryonal(D) Alveolar(E) Rhabdoid

Explanation:

Alveolar rhabdomyosarcoma carries the worse prognosis.

Superior prognosis

Botryoid rhabdomyosarcomaSpindle cell rhabdomyosarcoma

Intermediate prognosis

Embryonal rhabdomyosarcoma

Unevaluable prognosis

Rhabdomyosarcoma with rhabdoid features

24. Which of the following tumors may have multiple small, spotty radiopacities (focal calcification):(A) Malignant fibrous histiocytoma(B) Liposarcoma(C) Fibrosarcoma(D) Synovial sarcoma(E) Epitheloid sarcoma

Explanation:

Page 19: Hyperguides 2015 .Tumor

Synovial sarcomas may have multiple small, spotty radiopacities caused by focal calcification (less often bone) in 15% to 20% of cases.

25. A 55-year-old man presents with a 4-month history of shoulder discomfort. The plain radiographs are shown in Slide 1 and a biopsy in Slide 2. Which of the following tests is recommended to assess the extent of this condition (staging studies):

Slide 1 Slide 2

(A) Computerized tomography scan and technetium bone scan(B) Technetium bone scan alone(C) Skeletal survey and bone marrow aspiration(D) Computerized tomography of the chest and abdomen and technetium bone scan(E) Positron emision tomography scan of the brain and computerized tomography of the chest

Explanation:

The plain radiograph of the shoulder shows lytic lesions in the scapular spine, distal clavicle, and the proximal third of the clavicle. There is an elliptical erosion in the proximal clavicle. These three lesions suggest a diagnosis of metastatic bone disease or multiple myeloma. The biopsy specimen shows plasma cells. The plasma cells have these characteristic features:

Eccentrically placed nucleus Peripheral clumping of the nuclear chromatin A perinuclear halo

The diagnosis is multiple myeloma. Staging is important and is performed with a skeletal survey, bone marrow biopsy, and chemistry studies including kidney function and serum calcium determination.

Technetium bone scans have a high false-negative rate and are not used in the staging process. Computerized tomography scans and brain scans are not routinely used.

26. A 45-year-old man has pain with overhead activity and discomfort when he lies on his shoulder at night. Plain radiographs of his humerus are shown in Slide 1. T1- and T2-weighted coronal images are shown in Slide 2 and Slide 3, respectively. A low- and high-power biopsy specimen is shown in Slide 4 and Slide 5, respectively. The best form of treatment would be:

Page 20: Hyperguides 2015 .Tumor

Slide 1 Slide 2 Slide 3

Slide 4 Slide 5

(A) Internal fixation and external beam irradiation(B) External beam irradiation(C) Preoperative chemotherapy and wide resection(D) Observation(E) Wide resection alone

Explanation:

The plain radiographs of the humerus show an intramedullary lesion that is mineralized. Notice that the cortices are intact. The T1-weighted coronal magnetic resonance image (MRI) shows a well-defined, low- signal lesion and the T2-weighted image shows a well-defined, high-signal lesion. Both sequences show a lobular (small circle) pattern to the lesion. The plain radiograph and MRI scan are consistent with a cartilage lesion. The lack of cortical changes indicates a diagnosis of enchondroma.

The biopsy specimen on low power shows lobules of cartilage surrounded by enchondral bone formation. The cartilage is hypocellular, and the nuclei are consistently small and round.

The diagnosis of enchondroma can be made with plain radiographs alone or plain radiographs and MRI scans. Treatment is observation with new radiographs in 3 to 6 months.

Page 21: Hyperguides 2015 .Tumor

27. A 35-year-old man has pain and limited pronation-supination of his forearm. His plain radiograph (Slide 1) and computed tomography scan (Slide 2) are shown. The most likely diagnosis is:

Slide 1 Slide 2

(A) Sclerotic synovial sarcoma(B) Parosteal osteosarcoma(C) Heterotopic ossification(D) Parosteal lipoma(E) Parosteal osteoma

Explanation:

The plain radiograph shows heavy bone formation lateral to the proximal radius. This bone formation could be caused by a parosteal osteosarcoma, heterotopic ossification, or a parosteal osteoma.

The computed tomography scan shows the mature bone formation that is more mineralized at the periphery than in the center. The bone spans the radius and the ulna. The zonal pattern of greater mineralization at the periphery compared to the center is characteristic of heterotopic ossification.

28. A 55-year-old man has a 10-cm thigh mass that is low signal on T1-weighted images and high signal on T2-weighted images. A needle biopsy is performed and shown in Slide 1 and Slide 2. The most likely diagnosis is:

Page 22: Hyperguides 2015 .Tumor

Slide 1 Slide 2

(A) Soft tissue metastasis(B) Myositis ossificans (heterotopic ossification)(C) Schwannoma(D) Malignant fibrous histiocytoma(E) Collagenous fibroma

Explanation:

A large soft tissue mass that is located below the fascia should raise the suspicion of a sarcoma. All sarcomas have a common imaging pattern - low signal on T1-weighted sequences and high signal on T2-weighted sequences.

The biopsy shows the typical storiform pattern of malignant fibrous histiocytoma. Notice the spindle cells with different sizes and shapes of the nuclei (pleomorphism). There are also cells with round and vesicular nuclei.

29. A 23-year-old man has a 3-cm firm mass on the ulnar border of his hand. The mass is low signal on T1-weighted images and high signal on T2-weighted images. A biopsy is shown in Slide 1 and Slide 2. The most likely diagnosis is:

Slide 1 Slide 2

(A) Nodular fasciitis(B) Schwannoma(C) Foreign body granuloma(D) Epithelioid sarcoma(E) Sebaceous cyst

Page 23: Hyperguides 2015 .Tumor

Explanation:

A rare malignant tumor known as epithelioid sarcoma may occur in the upper extremity in young patients.The biopsy shows a nodule on low power with an area of central necrosis. On high power, one sees rounded cells with eosinophilic cytoplasm. This is an epithelioid sarcoma.

30. A 65-year-old woman has had severe arm and shoulder pain for 4 months. Her plain radiographs are shown in Slide 1 and Slide 2. Sections from a needle biopsy are shown in Slide 3 and Slide 4. Which of the following is the optimal treatment method:

Slide 1 Slide 2

Slide 1 Slide 2

(A) Wide resection and prosthetic reconstruction followed by irradiation(B) Internal fixation, external beam irradiation, and diphosphonate therapy(C) Wide resection and prosthetic reconstruction(D) External beam irradiation and diphosphonate therapy(E) Observation

Explanation:

The plain radiographs show a destructive lesion with a permeative pattern of bone destruction (multiple small lytic areas) and areas of bone formation - a mixed pattern of bone destruction and

Page 24: Hyperguides 2015 .Tumor

bone formation. The differential diagnosis based upon the radiographs alone in this age group (ages 40 to 80 years) includes:

Metastatic bone disease Multiple myeloma Lymphoma Primary mesenchymal bone tumors:

   Chondrosarcoma   Malignant fibrous histiocytoma

The biopsy specimen shows epithelial cells clumped together in an organoid pattern. This patient has metastatic breast carcinoma.

There is between 25% and 50% cortical bone destruction. Note that the medial and anterior cortices are completely intact. This patient can be treated with external beam irradiation and diphosphonate therapy with a low risk of fracture (about 10%). Although internal fixation is not necessary, if the patient does not want to risk having a fracture, then fixation can be performed with postoperative external beam irradiation and diphosphonate therapy.

31. A 45-year-old woman has a large soft tissue mass in her deltoid muscle, which is located in the shoulder region. The T1- and T2-weighted images are shown in Slide 1 and Slide 2. The most likely diagnosis is:

Slide 1 Slide 2

(A) Intramuscular lipoma(B) Glenohumeral ganglion(C) Malignant fibrous histiocytoma(D) Hemangioma(E) Hematoma

Explanation:

The T1-weighted image shows a large high-signal mass in the deltoid muscle. The signal quality

Page 25: Hyperguides 2015 .Tumor

exactly matches the subcutaneous fat. Note on the T2-weighted image with fat suppression that the mass completely suppresses (or turns black or low signal).

This patient has a lipoma or a benign soft tissue neoplasm. The patient can be treated with either observation or removal of the lipoma. The lipoma can be removed with a marginal or intralesional margin.

The other lesions refer to common soft tissue processes:

A malignant fibrous histiocytoma is low signal on T1-weighted images and high signal on T2-weighted images.

A hematoma shows fluid signal and is heterogeneous, with a low signal on T1-weighted images and high signal on T2-weighted images.

A glenohumeral ganglion is uniformly dark signal on T1-weighted images and high signal on T2-weighted images.

An arteriovenous malformation shows serpiginous channels that are low on T1-weighted imaged and high signal on T2-weighted images.

32. Which of the following tumors commonly occurs in the hand:(A) Extra-abdominal desmoid tumor(B) Liposarcoma(C) Myxoid chondrosarcomas(D) Fibrosarcoma(E) Epitheloid sarcoma

Explanation:

Epitheloid sarcoma has a propensity to occur in the hand and upper extremity. This particular tumor is also prone to lymph node and skin/subcutaneous metastases.

Common locations include:

A. Location: Upper and lower extremities - 60%1. Thigh is most common location2. Specific tumors and site predilection

a. Epitheloid sarcoma - most common soft tissue sarcoma of the upper extremity

b. Synovial sarcoma -most common soft tissue sarcoma of the foot and ankle

33. Patients with high-grade intramedullary osteosarcoma are best treated with which of the following therapies:(A) Wide resection and chemotherapy(B) Wide resection, chemotherapy, and diphosphonates(C) Wide resection, chemotherapy, and external beam irradiation

Page 26: Hyperguides 2015 .Tumor

(D) Wide resection and external beam irradiation(E) Wide resection alone

Explanation:

Patients with high-grade intramedullary osteosarcoma are best treated with preoperative chemotherapy, wide surgical resection, and a variable duration of postoperative chemotherapy. External beam irradiation is not useful for treating patients with intramedullary osteosarcoma.

The most important prognostic factor is response to preoperative chemotherapy. When necrosis rates >95% are achieved, a more favorable prognosis can be expected.

34. A 15-year-old boy has a Ewing's sarcoma of the proximal fibula. His staging studies are negative. A moderate-sized soft tissue mass is located circumferentially around the fibula. The soft tissue mass does not involve the neurovascular bundles. Which of the following treatment alternatives would be most appropriate:(A) Wide resection alone(B) Above knee amputation alone(C) Chemotherapy and wide resection(D) Wide resection followed by external beam irradiation(E) External beam irradiation alone

Explanation:

Ewing's sarcoma has a high propensity to metastasize to the lungs and bone. Without intensive chemotherapy, the 5-year survival is <25%. Patients are treated with systemic chemotherapy to control micro metastases and either external beam irradiation to the local site or wide surgical resection.

Wide surgical resection is often used for the sites that will not result in major morbidity. If the surgical margins are positive, then patients undergo external beam irradiation.

35. Which of the following is the most appropriate treatment for patients with periosteal osteosarcoma:(A) Wide resection alone(B) Wide resection and external beam irradiation(C) Wide resection and chemotherapy(D) Wide resection, chemotherapy, and external beam irradiation(E) Curettage and grafting

Explanation:

Periosteal osteosarcomas are intermediate-grade malignancies. The risk of pulmonary metastases is approximately 15% to 30%. Patients are treated with chemotherapy and wide surgical resection. External beam irradiation is not useful for patients with periosteal

Page 27: Hyperguides 2015 .Tumor

osteosarcoma.

36. Which of the following factors causes osteoclast activation and bone destruction in multiple myeloma:(A) Osteoprotegerin (OPG)(B) Transforming growth factor-beta (TGF-B)(C) Interferon(D) Receptor activator of nuclear factor-kB ligand (RANKL)(E) Interleukin-4 (IL-4)

Explanation:

The bone destruction in multiple myeloma is caused by the release of RANKL from the osteoblast surface. The receptor activator of nuclear factor-kB ligand binds to a receptor activator of nuclear factor-kB (RANK) on the osteoclast progenitor cell surface, beginning the process of osteoclast activation and bone destruction. Osteoprotegerin is a decoy inhibitor of the system and halts osteoclast activation.

Factors that up regulate RANKL and down regulate OPG

Parathyroid hormone, parathyroid hormone-related protein 1,25 dihydroxy vitamin D Interleukin-1beta Tumor necrosis factor alpha Prostaglandin E2

Factors that down regulate RANKL and up regulate OPG

IL-4 Interferon N gamma TGF-B

Factors that increase OPG expression

Estrogen TGF-B

37. Which of the following describes the correct treatment of a sacral chordoma:(A) Preoperative chemotherapy and wide resection(B) Wide resection and postoperative chemotherapy(C) Wide resection alone(D) Curettage and bone grafting

Page 28: Hyperguides 2015 .Tumor

(E) External beam irradiation

Explanation:

A chordoma is a low-grade malignancy that has a high rate of local recurrence, but a low risk of metastasis. Although the 10-year survival rate is only approximately 25%, the majority of patients die from local spread rather than from metastases.

Chordomas are treated by wide resection alone. There is no role for chemotherapy. External beam irradiation can be used for locally recurrent cases, but it seldom cures this condition.

38. Symptoms of pigmented villonodular synovitis (PVNS):(A) Never appear on the joints of the upper limb(B) Cause no changes on the bone(C) Do not result in locking of the joint(D) Usually involves the knee(E) Always cause painless swelling

Explanation:

The diffuse form of PVNS usually involves the knee as a monoarthropathy. Apart from the knee, this disorder has been described in the ankle, hip, wrist, spine, shoulder, and temporomandibular joint.

PVNS lesions presented in giant cell tumor of the tendon sheath are the most common soft tissue tumor of the hand. The presenting symptom in this lesion is a painless mass on a finger.

Ten percent to 25% of the lesions cause erosion and atrophic pressure on the underlying cortical bone.

Intra-articular PVNS, a rare manifestation of the disorder, can present with pain and locking of the joint.

The pedunculated nodules can undergo torsion and infarction, leading to acute pain.

39. Synovial hemangioma:(A) Is an atypical non-infectious inflammation(B) Mainly appears in young boys(C) May be a part of a generalized hemangiomatosis syndrome(D) Is relatively common(E) Is always localized

Explanation:

Page 29: Hyperguides 2015 .Tumor

Synovial hemangioma may be a part of a heterogenous syndrome, involving hemangiomas of the skin, bone, and muscles (Klippel-Trenaunay-Weber disease).

Synovial hemangioma is a proliferation of blood vessels arising in a synovium-lined surface.

Synovial hemangioma appears at any age and involves both men and women. Synovial hemangioma is rare. Synovial hemangioma appears in either a localized or diffuse form.

40. Which of the following statements is true regarding synovial osteochondromatosis:(A) Primary synovial chondromatosis is a rare tumor-like lesion.(B) Synovial osteochondromatosis is based exclusively on cartilaginous metaplasia.(C) Synovial chondromatosis is a poorly delineated histological entity.(D) Subsynovial metaplastic chondroid islands can be misleading.(E) Etiology is clarified.

Explanation:

Primary synovial chondromatosis is an uncommon tumor-like lesion.

Synovial osteochondromatosis is based on cartilaginous metaplasia, originating from the synovium.

The condition is not a simple metaplasia but has a proliferative component, as well. Synovial chondromatosis is a histologically verified entity with subsynovial metaplastic

chondroid islands. The etiology of the process is unknown.

41. Which of the following treatment options is best for synovial osteochondromatosis:(A) Nonsteroidal anti-inflammatory drugs(B) Arthroscopic surgery to remove loose chondromas and partial synovectomy to prevent locking

episodes and pain(C) Open arthrotomy and total synovectomy(D) Recurrence rate after surgery is more than 60%(E) Malignant transformation often occurs

Explanation:

Partial synovectomy and arthroscopic surgery is usually appropriate.

Nonsteroidal anti-inflammatory drugs provide only some relief of symptoms Open surgery is not necessary because the entire synovium is not involved The rate of recurrence is estimated to be 15% to 25% Synovial chondromatosis can become malignant

42. A patient presents with a hard leg mass and pain with activity. The anteroposterior and lateral

Page 30: Hyperguides 2015 .Tumor

radiographs are shown in Slide 1 and Slide 2. An axial computed tomography scan is shown in Slide 3. The risk of malignancy in this condition is approximately:

Slide 1 Slide 2

Slide 3

(A) No risk of malignancy exists.(B) 5% to 10%(C) 25%(D) 50%(E) 100%

Explanation:

This patient has multiple hereditary exostoses. Widening of the metaphysis is characteristic of multiple hereditary exostoses. Large sessile osteochondromas arise from the metaphysis and a large osteochondroma arises from the medial metaphysis with a characteristic cartilaginous cap. The computed tomography scan shows the widening and abnormal tubulation of the bone.

No evidence of malignancy exists in this large osteochondroma. The cartilage cap is regular with no areas of bone destruction. One should also look for a soft tissue mass, which often shows areas of focal calcifications. No soft tissue masses are present in this patient.

It is important to remember that this condition is autosomal dominant. The putative tumor suppressive gene mutation is EXT1, EXT2. The risk of low-grade chondrosarcoma occurring in this

Page 31: Hyperguides 2015 .Tumor

condition is approximately 10%.

When patients are symptomatic, simple excision of the osteochondroma is all that is necessary.

43. Which of the following is the major regulator of the transition from the G1 phase of the cell cycle to the S phase (synthesis and reproduction):(A) p53 protein(B) p21 protein(C) Cyclin-dependent kinase inhibitors(D) Dihydrofolate reductase (DHFR)(E) Retinoblastoma protein

Explanation:

During the reproductive cycle of cells, the most important transition is from G1 or G0 (resting states) to the S phase. In the S phase, synthesis of DNA and mitosis occurs. In humans, this transition point is regulated by the retinoblastoma protein.

The retinoblastoma protein is controlled by a group of regulatory proteins – cyclins (cyclin D and E kinases), cyclin-dependent kinases (CDK4, CDK6), and cyclin kinase inhibitors (p16, p21).

To begin cell synthesis, the retinoblastoma protein is phosphorylated by cyclin D and CDK4. This phosphorylation causes release of the E2F transcription factor. The E2F transcription factor signals increased transcription of a number of genes, including dihydrofolate reductase (DHFR), thymidylate synthetase (TS), thymidine kinase (TK), and ribonucleotide reductase (RR). These factors are necessary for DNA replication in the S phase.

The important point to remember is that the retinoblastoma gene (Rb1) and the retinoblastoma protein regulate the transition point from G1 to the S phase. Tumor cells cannot grow without going past this transition point.

44. Which of the following statements is true concerning total knee replacement in patients with pigmented villonodular synovitis (PVNS) of the knee:(A) Aseptic loosening secondary to recurrent disease is common.(B) Recurrence rates are higher than in patients treated with synovectomy alone.(C) Postoperative stiffness is common.(D) Postoperative results show failures in more than 50% of patients.(E) Recurrence rates are low.

Explanation:

Total knee replacement is successful in patients who have PVNS. The results are equal to total

Page 32: Hyperguides 2015 .Tumor

knee arthroplasty in patients in the general population and patients who have focal PVNS. Loosening and recurrence are potential complications in a patient who has active diffuse PVNS and undergoes total knee replacement.

Important points to remember:

Interval to arthroplasty, average 8 years (5 months to 23 years) Nine patients had 19 prior procedures Six patients had 14 prior recurrences Results (11 patients with active, diffuse disease, average age 59 years):

o 7/11 intact functioning knees at 11 yearso 2/7 patients required manipulationo 3/11 required revision for aseptic loosening

Loosening was not secondary to recurrent PVNSo 2/11 recurrence

One patient required above-knee amputation

45. Which of the following fusion proteins has the best prognosis in patients with Ewing’s tumor:

(A) EWS-ERG(B) EWS-FLI1(C) EWS-ETV1(D) EWS-ETV4(E) EWS-FEV

Explanation:

Ewing’s tumor is characterized by a balanced translocation – t(11;22). A chimeric protein is formed called the fusion product.

The most common fusion product is EWS-FLI1 (found in 85%). EWA-FLI1 has the best prognosis.

The other fusion products found in Ewing’s tumor include:

EWS-ERG EWS-ETV1 EWS-ETV4 EWS-FEV

46. Which of the following is released by breast cancer cells and causes direct activation of osteoclasts:

(A) Parathyroid hormone-related protein (PTHrP)

Page 33: Hyperguides 2015 .Tumor

(B) Osteoprotegerin (OPG)(C) Interleukin-8 (IL-8)(D) Endothelin-1(E) MIP-1α

Explanation:

Interleukin-8 (IL-8) is released by metastatic breast cancer cells and can cause direct activation of osteoclasts. This direct activation is independent of the RANKL activation of the RANK receptor.

The other responses refer to:

PTHrP – Released by metastatic breast cancer cells and causes activation of the osteoclast via signaling to the osteoblast for release of the RANKL

OPG – A competitive decoy inhibitor that binds to the RANKL and prevents attachment to the RANK

receptor on the osteoclast precursor cells; OPG prevents osteoclast activation and bone resorption

Endothelin-1 – Causes bone formation in metastatic prostate cancer to bone MIP-1α – Produced by multiple myeloma cells and causes osteoclast activation

47. Which of the following is the best approximation of survival after bone metastases in lung cancer:

(A) 6 months(B) 24 months(C) 36 months(D) 48 months(E) 60 months

Explanation:

The average survival of patients with lung cancer following bone metastases is approximately 6 months.

48. which of the following is released by breast cancer cells and causes direct activation of osteoclasts:

(A) Parathyroid hormone-related protein (PTHrP)(B) Osteoprotegerin (OPG)

Page 34: Hyperguides 2015 .Tumor

(C) Interleukin-8 (IL-8)(D) Endothelin-1(E) MIP-1α

Explanation:

Interleukin-8 (IL-8) is released by metastatic breast cancer cells and can cause direct activation of osteoclasts. This direct activation is independent of the RANKL activation of the RANK receptor.

The other responses refer to:

PTHrP – Released by metastatic breast cancer cells and causes activation of the osteoclast via signaling to the osteoblast for release of the RANKL

OPG – A competitive decoy inhibitor that binds to the RANKL and prevents attachment to the RANK

receptor on the osteoclast precursor cells; OPG prevents osteoclast activation and bone resorption

Endothelin-1 – Causes bone formation in metastatic prostate cancer to bone MIP-1α – Produced by multiple myeloma cells and causes osteoclast activation

49. Which of the following factors is important for invasion of the basement membrane in the metastatic process:

(A) Integrins(B) Metalloproteinases (MMPs)(C) Parathyroid hormone-related protein (PTHrP)(D) CXCL12(E) Receptor activator of nuclear factor kappa B ligand (RANKL)

Explanation:

Metalloproteinases (MMPs) are a group of enzymes that break down matrixes and membranes. The collagenases are in the group of MMPs. The MMPs are important for penetration of the basement membrane.

The other responses refer to:

Integrins – Adhesive proteins for cell attachment PTHrP – Causes osteoclast activation and bone resorption CXCL12 – Causes homing of tumor cells to bone RANKL – Causes activation of osteoclast precursor cells to osteoclasts

50. which of the following describes the microscopic appearance of a neurofibroma:

Page 35: Hyperguides 2015 .Tumor

(A) Distinct hyoid and cellular areas(B) Verocay bodies(C) Spindle- and stellate-shaped cells in a mucoid matrix(D) Disorganized admixture of fibrous tissue and regenerative nerve sprouts(E) Random bundles of coarse collagen and cells with wavy nuclei

Explanation:

Neurofibromas have a characteristic appearance:

Random bundles of collagen Wavy nuclei Myxoid matrix

Other lesions that must be distinguished from neurofibromas include:

Schwannoma: Myxoid and cellular areaso Antoni A: Cellular areaso Antoni B: Myxoid areaso Verocay bodies: Parallel arrangement of nuclei

Myxoma: Spindle- and stellate-shaped cells in a mucoid matrix Neuroma: Disorganized admixture of fibrous tissue and regenerative nerve sprouts

51. Which of the following describes the presentation of a patient with an osteoid osteoma:(A) Pain with weight bearing(B) Hard mass but no pain(C) Pain relieved by aspirin and nonsteroidal anti-inflammatory drugs(D) Intermittent discomfort relieved by rest(E) Morning stiffness

Explanation:

Osteoid osteomas have one of the most characteristic presentations of all bone tumors. The pain is often completely relieved with aspirin and nonsteroidal anti-inflammatory drugs. Parents of affected children have stated that their child will finish a whole bottle of aspirin in 1 week. The pain commonly occurs at night and will wake the child if he or she does not take some form of medication.

52. When osteoid osteomas occur in the spine, they are most commonly located:(A) Beneath the anterior longitudinal ligament(B) In the vertebral body(C) In the spinous process

Page 36: Hyperguides 2015 .Tumor

(D) In the lamina or base of the pedicle(E) In the transverse process

Explanation:

When osteoid osteomas occur in the spine, they are most commonly located in the lamina or the base of the pedicle. Generally, they do not occur in the vertebral bodies, spinous processes, or transverse processes.

Common location

In the lamina or base of the pedicle

Uncommon locations

Vertebral body Spinous process Transverse process

53. To which of the following organs do high-grade intramedullary osteosarcomas most commonly metastasize:(A) Brain(B) Lungs(C) Adrenals(D) Other bones(E) Kidneys

Explanation:

Osteosarcomas most commonly spread to the lungs. Plain chest radiographs may not reveal small lesions. Computerized tomography of the chest is the most sensitive method to detect nodules that are 3 mm to 5 mm in diameter. Other sites of metastases include other bones and visceral organs.

54. Which of the following best describes the current treatment of a Grade 2 chondrosarcoma of the distal femur:(A) Wide resection alone(B) Wide resection and multi-agent chemotherapy(C) Wide resection, multi-agent chemotherapy, and radiation(D) Wide resection and radiation(E) Interferon and radiation

Explanation:

Chondrosarcomas are treated by surgical resection alone. Chemotherapy and irradiation play a

Page 37: Hyperguides 2015 .Tumor

limited role in treatment. Generally, chondrosarcoma grow slowly, therefore irradiation or chemotherapy will have little effect on the tumor cells.

55. Which of the following lesions often involves multiple bones in the same extremity in young patients:

(A) Osteosarcoma(B) Giant cell tumor(C) Chondroblastoma(D) Hemangioendothelioma(E) Chondromyxoid fibroma

Explanation:

Most bone tumors are solitary lesions with a low incidence of multicentricity. Vascular tumors are an exception. These tumors may involve multiple bones. Patients with hemangioendothelioma present with multiple lytic lesions. These lytic lesions often occur in the same extremity in young patients.

56. All of the following statements are true concerning osteoblastomas except:(A) They may grow to large size(B) There is often a lack of nocturnal pain that is responsive to aspirin(C) Surrounding sclerosis may be minimal or absent(D) The lesion is usually 5 mm to 10 mm in diameter(E) Histologically, osteoblastoma is similar to osteoid osteoma

Explanation:

In contrast to osteoid osteomas, which have a limited growth potential (5 mm to 10 mm, occasionally 10 mm to 15 mm), osteoblastomas may grow to large size. When they occur in the spine, osteoblastomas may press on the spinal cord and nerve roots causing neurologic symptoms and even deficits.

Histologically, osteoblastomas are similar to osteoid osteomas.

Osteoblastomas differ from osteoid osteomas in that:

There is often a lack of nocturnal pain that is responsive to aspirin Surrounding sclerosis may be minimal or absent

57. A 17-year-old man has a 4-month history of severe pain in his right distal tibia. The pain is especially prominent at night. He has difficulty sleeping and concentrating at school. The lateral radiograph of his right distal tibia is shown in Figure 1. A CT and axial T1-weighted MRI scan

Page 38: Hyperguides 2015 .Tumor

are shown in Figures 2 and 3. Which of the following treatment options should be recommended to the patient and family:

Slide 1 Slide 2 Slide 3

(A) Open or needle biopsy, preoperative chemotherapy, and wide resection(B) Open or needle biopsy, preoperative chemotherapy, and external beam irradiation(C) Below knee amputation(D) Long leg cast for 4 weeks, followed by a patellar tendon bearing cast(E) Nonsteroidal anti-inflammatory medications

Explanation:

In this case, the diagnosis is osteoid osteoma. Osteoid osteomas are common tumors in young individuals. Patients usually present with intense pain, and they can often localize the exact point of discomfort.

CT findings are often characteristic:

A small nidus, 5 mm to 10 mm, in the cortex or in a sub-periosteal location Central ossification is often present in the nidus Sclerosis is found surrounding the nidus

The pain of osteoid osteomas is often dramatically relieved by aspirin or nonsteroidal anti-inflammatory drugs (NSAIDs). Kniesel and Simon reported on a series of patients treated with NSAIDs in the Journal of Bone and Joint Surgery. Approximately 50% of the patients had long-term relief of their pain.

58. A 35-year-old man has a 9-month history of a soft tissue mass in his forearm. The mass has been steadily enlarging over time. The T1- and T2-weighted axial MRI scans are shown in Figures 1 and 2, respectively. A biopsy was performed and is shown in Figure 3. The most appropriate treatment of this mass would be:

Page 39: Hyperguides 2015 .Tumor

Figure 1 Figure 2

Figure 3(A) Observation(B) Simple excision(C) Preoperative radiation therapy and wide resection(D) External beam irradiation alone(E) External beam irradiation and chemotherapy

Explanation:

The radiographic appearance of the lesion, low signal on T1-weighted images and high signal on T2-weighted images, is nonspecific. Virtually all sarcomas are low signal on T1-weighted images and high signal on T2-weighted images. However, one must remember that not all lesions that are low on T1-weighted images and high on T2-weighted images are malignant.

The histologic section shows a bimorphic tumor with both spindle cells and epithelial cells. This is the classic appearance of synovial sarcoma. This lesion is a high-grade malignancy and would be treated with preoperative irradiation and wide resection. Other acceptable answers are:

Page 40: Hyperguides 2015 .Tumor

Wide resection and postoperative radiation Preoperative chemotherapy, radiation therapy, and wide resection

High-grade sarcomas cannot be cured without surgery, hence the possible answers listed that do not include surgery are wrong. Simple excision would result in prompt local recurrence as the margins would be positive. Observation is a poor choice, as the tumor would continue to grow and probably metastasize.

59. A 12-year-old boy has a destructive lesion in the proximal humerus. The lesion is in the proximal dia-metaphyseal region with destruction of about one-third of the cortical bone. The lesion is biopsied and is an eosinophilic granuloma (Langerhans’ cell granulomatosis). The best treatment method would be:(A) Wide resection and allograft reconstruction(B) Preoperative chemotherapy followed by wide resection (allograft or prosthesis)(C) High-dose external beam irradiation and chemotherapy(D) Multi-agent chemotherapy alone(E) Intralesional corticosteroid injection

Explanation:

Eosinophilic granuloma, or Langerhans cell granulomatosis, is a reactive condition that can often cause marked bone destruction. This condition usually occurs in children and young to middle aged adults.

The characteristic radiographic finding is a well-demarcated punched out lesion. The femur, humerus, and flat bones are common sites. Occasionally, patients may present with a very aggressive lesion with a permeative pattern of bone destruction, cortical bone destruction, and a soft tissue mass.

Eosinophilic granuloma is a self-limited disorder. There have been many different treatments proposed such as biopsy only, curettage and bone grafting, and low-dose irradiation. Most recently, intralesional cortisone injection has become the standard treatment for most lesions.

60. Which of the following is the most prominent feature of the presentation of children with osteosarcoma:(A) Painless mass(B) Recurrent atraumatic effusions(C) Pain with activity, relieved by rest(D) Pain that may begin as intermittent and progresses to constant(E) Pain following sports, especially running and jumping

Explanation:

Pain is the most prominent feature of high-grade malignant tumors in children and adults. The pain is a deep-seated pain and is usually well localized when the lesion is in the extremity. The pain

Page 41: Hyperguides 2015 .Tumor

begins as intermittent pain and then progresses to constant pain. The pain often occurs at night and at rest. As the tumor grows, the pain becomes constant and relentless.

Pain from malignant bone tumors:

Is dull or sharp Is deep-seated Occurs at rest and at night Begins as intermittent discomfort and progresses to constant

61. Which of the following radiographic features helps in differentiating a chondrosarcoma from an enchondroma:(A) Stippled and ring-like calcification(B) Small endosteal erosions (< 50% cortical width)(C) Expansion of the cortex in a small bone(D) Cortical perforation and cortical thickening(E) Absence of matrix calcification

Explanation:

Enchondromas and chondrosarcomas are both intramedullary cartilage lesions. In the adult patient, enchondromas do not grow; they are inactive lesions. When an intramedullary cartilage lesion grows and destroys the bone, the lesion is a chondrosarcoma.

The hallmark feature of almost all chondrosarcomas is involvement of the cortical bone. The cortical bone involvement can manifest in several different ways:

Erosion of the cortex:

Endosteal erosion occurs slowly as the tumor grows; erosions that are greater than 50% of the thickness of the cortex are suggestive of a chondrosarcoma

Thickening of the cortex:

Most chondrosarcomas are low-grade lesions. The tumor grows slow and may gradually distort the cortex. With low-grade lesions, the endosteal surface may be resorbed with concomitant periosteal new bone formation of the periosteal surface. The cortex is then thickened by the endosteal erosion of the chondrosarcoma.

Destruction of the cortex:

Some chondrosarcomas simply destroy the cortex as they grow and the lesion then extends into the soft tissues. When there is complete bone destruction, these chondrosarcomas are usually of a higher grade.

62. Enchondromas may commonly occur in all of the following sites except:(A) Proximal femur(B) Distal femur

Page 42: Hyperguides 2015 .Tumor

(C) Proximal humerus(D) Pelvis(E) Phalanges of the hand

Explanation:

Enchondromas are hyaline cartilage tumors that occur in the medullary cavity. They are usually asymptomatic. They do not expand long bones or cause major erosions of the cortical bone. They may, however, cause cortical expansion in small bones.

In order of occurrence, the most frequent sites of enchondromas include:

Small bones of the hands and feet (hand to feet ratio 7:1) Femur Proximal humerus Less common: forearm, tibia, and fibula

It is rare for enchondromas to occur in the following sites:

Pelvis Ribs Scapula Vertebra

63. The most common site of periosteal chondromas is:(A) Proximal femur(B) Distal femur(C) Proximal humerus(D) Hand metacarpals(E) Proximal tibia

Explanation:

Periosteal chondromas are rare lesions. Over 50% of the time, they are located in the proximal humerus. Other locations include the distal femur, proximal tibia, and the hand.

Periosteal chondromas have typical radiographic appearances:

They are small, usually 3 cm to 5 cm. They sit on the top of the bone in an excavation in the cortex. There is a buttress of periosteal bone on either side of the lesion. About one-third of radiographs show mineralization from within.

Page 43: Hyperguides 2015 .Tumor

64. A 48-year-old woman has an enchondroma in the proximal humerus. She has pain with overhead activity and her shoulder hurts at night when she lies on it. The radiographs show no cortical erosions, cortical thickening, or other worrisome changes. Management at this point should be:(A) Open incisional biopsy(B) Needle biopsy through the deltoid muscle(C) Curettage and bone grafting(D) Wide resection and allograft prosthetic reconstruction(E) Observation with new radiographs in 3 to 6 months

Explanation:

Enchondromas are generally treated with observation. They are common and many patients will have symptoms secondary to other entities, such as rotator cuff tendinitis, biceps tendinitis, greater trochanteric bursitis, and other overuse and degenerative conditions. When an intramedullary hyaline cartilage lesion looks innocent on plain radiographs — no major endosteal erosions, cortical breakthrough, or thickening — the lesion is best regarded as an enchondroma. In the asymptomatic patient or the patient who has pain secondary to tendinitis or other condition, observation should be chosen rather than surgical treatment. New radiographs are taken usually at 3 to 6 months and again at 6 months. If the lesion has not changed on 3 series of radiographs over 12 to 24 months, the lesion can be safely regarded as an enchondroma.

65. A 15-year-old boy presents with a 9-month history of severe thigh pain. The pain is most severe at night and aspirin completely relieves the pain. The plain radiograph is shown in Figure 1 and the CT scan in Figure 2. The most likely diagnosis is:

Figure 1 Figure 2

Page 44: Hyperguides 2015 .Tumor

Figure 3 Figure 4 Figure 5

(A) Stress fracture(B) Osteomyelitis(C) Parosteal osteosarcoma(D) Osteoid osteoma(E) Osteoblastoma

Explanation:

The plain radiograph shows a heavily mineralized lesion on the medial aspect of the femoral shaft (Figure 3). The periosteal reaction is mature without evidence of a lytic focus. The heavy mineralization is suggestive of an osteoid osteoma, but the diagnosis cannot be made without identifying the nidus. The CT scan shows a small, 8-mm lytic focus in the medial cortex. Heavy periosteal new bone surrounds the lesion (Figure 4). When comparing the CT scans of the left and right femurs, one can see the abnormal shape of the medial aspect of the femur with a lytic focus located in the medial femoral cortex (Figure 5).

The essential features of osteoid osteoma are:

Small lytic nidus is between 5 mm and 10 mm. Nidus is surrounded by heavy periosteal new bone formation. Often, CT scans are the only way to demonstrate the nidus.

66. A 15-year-old boy presents with a 9-month history of severe thigh pain. The pain is most severe at night. He is unable to concentrate in school due to the pain. The plain radiograph is shown in Figure 1 and the CT scan in Figure 2. The most appropriate initial treatment method would be:

Page 45: Hyperguides 2015 .Tumor

Figure 1 Figure 2

Figure 3 Figure 4 Figure 5

(A) Physical therapy(B) Needle or open biopsy(C) Anti-inflammatory medication(D) Observation(E) En bloc resection and reconstruction

Explanation:

The radiograph and CT scan show the classic features of an osteoid osteoma. As many as 50% of patients can be successfully treated with the initial treatment of nonsteroidal anti-inflammatory drugs (Kniesel and Simon).

If nonoperative treatment fails, surgical therapy is aimed at removing the nidus completely. This can be done percutaneously with CT guided radiofrequency ablation or surgical removal with burring of the bed of the osteoid osteoma following removal.

To answer this question correctly, one must be able to recognize the classic features of osteoid osteoma:

Small lytic nidus is between 5 mm and 10 mm.

Page 46: Hyperguides 2015 .Tumor

Nidus is surrounded by heavy periosteal new bone formation. Often, CT scans are the only way to demonstrate the nidus.

The plain radiograph shows a heavily mineralized lesion on the medial aspect of the femoral shaft (Figure 3). The periosteal reaction is mature without evidence of a lytic focus. The heavy mineralization is suggestive of an osteoid osteoma, but the diagnosis cannot be made without identifying the nidus. The CT scan shows a small, 8-mm lytic focus in the medial cortex. Heavy periosteal new bone surrounds the lesion (Figure 4). When comparing the CT scans of the left and right femurs, one can see the abnormal shape of the medial aspect of the femur with a lytic focus located in the medial femoral cortex (Figure 5).

67. A 10-year-old boy injured his leg playing baseball. He now has pain whenever he bears weight on his extremity. He has no rest pain or other medical problems. The plain radiographs are shown in Figures 1 and 2, and a biopsy specimen is shown in Figure 3. What would be the appropriate stage according to the system of the Musculoskeletal Tumor Society:

Figure 1 Figure 2 Figure 3

(A) Stage 1(B) Stage 2(C) Stage 3(D) Stage II(E) Stage III

Explanation:

The radiographic and histologic features are consistent with non-ossifying fibroma. Because the lesion is symptomatic, the lesion is active. Non-ossifying fibromas have a low local recurrence rate and they never threaten the involved joint. Hence, this would be a Stage 2 lesion.

The Surgical Staging System of the Musculoskeletal Tumor Society

Page 47: Hyperguides 2015 .Tumor

is a useful system to both predict prognosis and plan treatment. The system for benign lesions is divided into three groups: inactive (latent), active, and aggressive:

Refers to lesions that are not causing pain and show no evidence of active growth. Stage 1 lesions are generally treated with observation only.

Refers to lesions that are causing pain or some form of disability. If a lesion has weakened the structure of the bone such that fracture may occur, the lesion would be considered a Stage 2 lesion.Refers to lesions that are large, have broken into the soft tissues, or have caused a pathologic fracture. These lesions are usually prone to local recurrence and have the potential of causing a major problem for the patient.

68. A 25-year-old man has a 4-month history of severe thigh and knee pain. He has pain both at rest and at night. He has tenderness on palpation of the thigh. He has been febrile and has an elevated white blood cell count. The anteroposterior and lateral radiographs of the femur are shown (Slide1). The high and lower power histologic sections are also shown (Slide 2) and (Slide 3). The most likely diagnosis is:

Slide 1 Slide 1

Page 48: Hyperguides 2015 .Tumor

Slide 3

(A) Osteogenic sarcoma(B) Osteomyelitis(C) Malignant fibrous histiocytoma(D) Fibrosarcoma(E) Ewing sarcoma

Explanation:

The histologic features of Ewing tumor include a monotonous blue cell tumor, indistinct cell borders, and no matrix production by the tumor cells.

69. A 50-year-old man has a lesion in the left humerus that is discovered during a routine annual physical examination. The plain radiographs (Slide 1), the coronal T1- and T2-weighted images (Slides 2 and 3), and a biopsy (Slide 4) are presented. The most appropriate treatment method is:

Page 49: Hyperguides 2015 .Tumor

Slide 1 Slide 2

Slide 3 Slide 4

(A) Observation with repeat radiograph in 3 to 6 months(B) Wide surgical resection and custom prosthetic reconstruction(C) Systemic chemotherapy and external beam radiation to the humerus(D) External beam irradiation alone to the humerus(E) Curettage, rod fixation, and external beam irradiation

Explanation:

The plain radiographs show a mineralized lesion in the medullary cavity. The mineralization

Page 50: Hyperguides 2015 .Tumor

shows stipples and small rings. This pattern is consistent with a cartilage lesion. There are no cortical changes indicating that the lesion is inactive. The magnetic resonance imaging scan shows a lesion with a low signal on the T1-weighted image and high signal on the T2-weighted image. There is no marrow edema bordering the lesion on the T2-weighted image. All of these features are compatible with the diagnosis of an enchondroma. Chondrosarcomas in long bones have easily recognized features: large cortical erosions, cortical destruction, cortical thickening, and a soft tissue mass.

The biopsy shows a cartilage lesion. There is no permeation of the trabecular bone. Both the plain radiographs and biopsy specimen are consistent with the diagnosis of an enchondroma.

Enchondromas are treated with observation and repeat radiographs in 3 to 6 months.

70. A 50-year-old man has a lesion in the left humerus that was discovered during a routine annual physical examination. The plain radiographs (Slide 1) and the coronal T1- and T2-weighted images (Slides 2 and 3) are presented. The area of mineralization in the medullary cavity marked by the arrow in Figure 1 is most likely:

Slide 1 Slide 2

Page 51: Hyperguides 2015 .Tumor

(A) Normal marrow which has calcified(B) Hyaline cartilage which has calcified(C) Areas of metastatic carcinoma(D) Reactive bone formation(E) Fibrous replacement of the marrow

Explanation:

The plain radiographs show a mineralized lesion in the medullary cavity. The mineralization shows stipples and small rings. This pattern is consistent with a cartilage lesion. There are no cortical changes indicating that this lesion is active. The magnetic resonance imaging scan shows a lesion with a low signal on the T1-weighted image and a high signal on the T2-weighted image. The high signal on the T2-weighted image is reflective of the high water content of cartilage lesions. There is no marrow edema bordering the lesion on the T2-weighted image. All of these features are consistent with the diagnosis of an enchondroma.

The other possible answers are wrong for a variety of reasons:

Normal marrow does not calcify. Metastatic carcinoma will show lytic features. Reactive bone formation shows cloud-like mineralization, and the bone is low signal on

T1- and T2-weighted images. When the marrow is replaced by fibrous tissue, there will not be any mineralization.

71. A 65-year-old woman has a 5-month history of increasing hip and thigh pain. She has severe pain with weight bearing on the right leg. Her plain radiographs (Slide 1) and T1- and T2-weighted coronal images (Slides 2 and 3) of her hip are presented. A biopsy is also performed (Slide 4). The cells directly responsible for the bone destruction are:

Page 52: Hyperguides 2015 .Tumor

Slide 1 Slide 2

Slide 3 Slide 4

(A) A. Tumor cells(B) B. Fibroblasts(C) C. Osteoclasts(D) D. Histiocytes(E) E. Plasma cells

Explanation:

The plain radiographs show an ill-defined permeative lesion in the proximal femur. The lateral cortex of the greater trochanter is thinned. The T1-weighted coronal images show a low-signal

Page 53: Hyperguides 2015 .Tumor

lesion in the entire greater trochanter extending to the base of the femoral neck. In the opposite proximal femur and acetabulum, there are multiple small low-signal lesions consistent with metastatic bone disease. The T2-weighted images show a very high-signal area in the greater trochanter. The inversion recovery image is sensitive to pathologic processes. Normal marrow will be low signal and pathologic lesions are high signal. The biopsy shows round epithelial cells forming glands. This pattern is characteristic of metastatic adenocarcinoma.

The bone destruction in metastatic bone disease is a direct result of osteoclastic resorption of the bone. The metastatic breast cells secrete a factor that activates and recruits osteoclasts that destroy the bone. Diphosphonate therapy (usually intravenous pamidronate) is given to virtually all women with metastatic breast carcinoma to stop the osteoclasts from resorbing the bone matrix.

72. A 60-year-old man has a 6-month history of increasing knee pain. He has discomfort at night and at rest. His anteroposterior and lateral radiographs are shown in Slide 1 and low and high power biopsy specimens in Slides 2 and 3. Chest radiograph and computerized tomography scans are normal. The stage of disease according to the system of the Musculoskeletal Tumor Society is:

Slide 1 Slide 2 Slide 3

(A) Stage 1(B) Stage 2(C) Stage I(D) Stage II(E) Stage III

Explanation:

The plain radiographs show two major features. The proximal portion of the lesion (top half) shows an intramedullary lesion, which is mineralized. The mineralization shows rings and stipples. There is also thickening of the cortex of the femur. This thickening is circumferential on the medial, lateral, anterior, and posterior cortices. These features strongly suggest a cartilage tumor, principally a chondrosarcoma. The distal (lower half) of the lesion has a different pattern. The intramedullary lesion is completely lytic, and the lateral cortex has been destroyed. This bimorphic pattern is seen with dedifferentiated chondrosarcoma. The bimorphic pattern is a low-grade

Page 54: Hyperguides 2015 .Tumor

chondrosarcoma and a high-grade anaplastic spindle cell sarcoma.

The histologic appearance is also bimorphic. Notice the cartilage, which is relatively hypocellular, and the intimate admixture of a high-grade spindle cell sarcoma. The high-grade spindle sarcoma is characterized by pleomorphic and pyknotic nuclei–dark staining nuclear chromatin, and different sizes and shapes of the nuclei. The bimorphic pattern is low-grade cartilage admixed with high-grade spindle cell sarcoma. This high-grade spindle cell sarcoma can be osteosarcoma, malignant fibrous histiocytoma, or fibrosarcoma.

This patient has a high-grade sarcoma and the staging studies show no evidence of pulmonary metastases. The surgical stage would be stage II. If an magnetic resonance imaging scan showed soft tissue extension, then the stage would be Stage IIB.

73. A 65-year-old woman has a 5-month history of increasing hip and thigh pain. Her plain radiographs are shown in Slide 1. She has severe pain with weight bearing on the right leg. T1 and T2 weighted coronal images of her hip are shown in Slides 2 and 3 respectively. A biopsy is performed and is shown in Slide 4. The most appropriate treatment would be:

Slide 1 Slide 2

Page 55: Hyperguides 2015 .Tumor

Slide 3 Slide 4

(A) Hip disarticulation(B) Neck exploration and removal of her parathyroid glands(C) Wide resection and custom proximal femoral replacement(D) Crutch ambulation for 6 to 8 weeks(E) Internal fixation followed by external beam irradiation

Explanation:

The plain radiographs show an ill-defined permeative lesion in the proximal femur. The lateral cortex of the greater trochanter is thinned. The T1 weighted coronal images show a low signal lesion in the entire greater trochanter extending to the base of the femoral neck. Notice in the opposite proximal femur and acetabulum, there are multiple small low signal lesions consistent with metastatic bone disease. The T2 weighted images show a very high signal area in the greater trochanter. The inversion recovery image is very sensitive to pathologic processes. Normal marrow will be very low signal while pathologic lesions are high signal. The biopsy shows round epithelial cells forming glands. This pattern is that of metastatic adenocarcinoma.

The treatment of metastatic breast carcinoma of bone is external beam irradiation to symptomatic lesions and diphosphonate therapy to retard the bone loss. Internal fixation is performed to prevent pathologic fracture. When patients have significant weight bearing pain, and a purely lytic lesion in the hip, the risk of fracture is high. The best treatment method for this patient is internal fixation followed by external beam irradiation.

74. A 60-year-old woman has severe knee pain that has been increasing for 6 months. The anteroposterior radiograph and T2 weighted coronal magnetic resonance imaging scan are shown in Slides 1 and 2. The biopsy specimen is shown in Slide 3. The most likely diagnosis is:

Page 56: Hyperguides 2015 .Tumor

Slide 1 Slide 2 Slide 3

(A) Metastatic bone disease(B) Multiple myeloma(C) Lymphoma(D) Chondrosarcoma(E) Malignant fibrous histiocytoma

Explanation:

The anteroposterior radiograph shows both arthritic changes in the medial compartment and a lytic destructive lesion in the lateral tibial plateau. Note the lucent area laterally and the area of increased density medially. The T2 weighted coronal magnetic resonance imaging scan shows complete marrow replacement of the proximal tibial plateau. The normal marrow is dark (low signal) and the tumor is high signal (white). The biopsy shows a mixture of large and small blue cells. This is the histologic appearance of lymphoma. Note that the other choices have characteristic histologic appearances.

Metastatic bone disease - epithelial cells organized in clusters in a fibrous background Multiple myeloma - sheets of plasma cells, that have an eccentric nucleus with peripheral

chromatin. There is also peri nuclear clearing or a halo where the Golgi apparatus is located

Chondrosarcoma - atypical chondrocytes in a blue chondroid matrix Malignant fibrous histiocytoma - fascicles of fibroblasts and histiocytes in a storiform

pattern

75. A 60-year-old woman has severe knee pain that has been increasing for 6 months. The anteroposterior radiograph and T2 weighted coronal magnetic resonance imaging scan are shown in Slides 1 and 2. The biopsy specimen is shown in Slide 3. The most appropriate treatment method would be:

Page 57: Hyperguides 2015 .Tumor

Slide 1 Slide 2 Slide 3

(A) Wide resection and custom arthroplasty(B) Above knee amputation(C) Curettage and cement augmentation(D) External beam irradiation and chemotherapy(E) Curettage and bone grafting

Explanation:

The anteroposterior radiograph shows both arthritic changes in the medial compartment and a lytic destructive lesion in the lateral tibial plateau. Note the lucent area laterally and the area of increased density medially. The increased bone density gives a mottled appearance to the bone. The T2 weighted coronal magnetic resonance imaging scan shows complete marrow replacement of the proximal tibial plateau. The normal marrow is dark (low signal) and the tumor is high signal (white). The biopsy shows a mixture of large and small blue cells. This is the histologic appearance of lymphoma. One should remember the characteristic histologic appearances of bone tumors in adults.

Metastatic bone disease - epithelial cells organized in clusters in a fibrous background Multiple myeloma - sheets of plasma cells, that have an eccentric nucleus with peripheral

chromatin. There is also peri nuclear clearing or a halo where the Golgi apparatus is located Chondrosarcoma - atypical chondrocytes in a blue chondroid matrix Malignant fibrous histiocytoma - fascicles of fibroblasts and histiocytes in a storiform pattern

Most lymphomas of bone are large cell lymphomas and are treated with both external beam irradiation and chemotherapy. Surgical resection is usually not performed.

76. A 28-year-old woman has a 4-month history of increasing shoulder pain. Her plain radiograph is shown in Slide 1. A biopsy specimen is shown in Slides 2 and 3. Which of the following is the correct stage according to the system of the Musculoskeletal Tumor Society:

Page 58: Hyperguides 2015 .Tumor

Slide 1 Slide 2

Slide 3(A) Stage 1(B) Stage 2(C) Stage 3(D) Stage I(E) Stage II

Explanation:

The plain radiograph shows a destructive lesion. The lesion is purely lytic, expansile, and there is a pathologic fracture. The lesion has destroyed the metaphysis and extended into the epiphysis.

From a radiographic perspective, this lesion could be:

A telangiectatic osteosarcoma An aneurysmal bone cyst A giant cell tumor

The biopsy specimen shows a uniform distribution of multinucleated giant cells in a field of mononuclear cells. The diagnosis is giant cell

Page 59: Hyperguides 2015 .Tumor

tumor of bone.

This lesion is an aggressive benign tumor – Stage 3.

Remember the Musculoskeletal Tumor Society staging system:

Stage 1 Benign inactiveStage 2 Benign activeStage 3 Benign aggressiveStage I Low grade malignantStage II High grade malignantStage III Metastases to any site

77. A 15-year-old girl has a 4-month history of increasing wrist pain. Her clinical photograph and plain radiograph are shown in Slides 1 and 2. An axial T1-weighted image and sagittal T2-weighted image are shown in Slides 3 and 4. A biopsy specimen is shown in Slide 5. Which of the following is the best treatment option:

Slide 1 Slide 2

Slide 3 Slide 4

Slide 5

Page 60: Hyperguides 2015 .Tumor

(A) Preoperative chemotherapy followed by resection/reconstruction(B) Resection/reconstruction followed by external beam irradiation(C) Resection/reconstruction followed by chemotherapy(D) Resection/reconstruction(E) Amputation

Explanation:

The plain radiograph shows a destructive lesion. The lesion is purely lytic, expansile, and there is a pathologic fracture. The lesion has destroyed the metaphysis and extended into the epiphysis.

From a radiographic perspective, this lesion could be:

A telangiectatic osteosarcoma An aneurysmal bone cyst A giant cell tumor

The biopsy specimen shows a uniform distribution of multinucleated giant cells in a field of mononuclear cells. The diagnosis is giant cell tumor of bone.

Treatment of this lesion is resection followed by reconstruction. The reconstructive options include an osteoarticular allograft, allograft fusion, or a fibular reconstruction. There is too much bone destruction to perform a curettage procedure.

78. A 15-year-old boy has a hard mass over his distal femur. He has pain when playing sporting activities. His plain radiographs are shown in Slides 1 and 2. The axial T1- and T2-weighted images are shown in Slides 3 and 4. The most appropriate treatment is:

Page 61: Hyperguides 2015 .Tumor

Slide 1 Slide 2

Slide 3 Slide 4

(A) Needle biopsy(B) Incisional biopsy(C) Removal of the mass(D) Further evaluation with a technetium bone scan(E) Further evaluation with a computerized tomography scan

Explanation:

The anteroposterior radiograph shows a lesion projecting from the medial aspect of the distal femur. The magnetic resonance imaging scan shows the cortices of the lesion are continuous with the underlying femur, and the medullary cavity of the lesion is continous with the medullary cavity of the distal femur. These findings are consistent with an osteochondroma. The bony stalk of the osteochondroma is covered by a cap of cartilage. The cartilage cap is low signal on T2-weighted images. The thickness of the cartilage cap is approximately 8 mm. There is no evidence of malignant change.

The radiographic findings of an osteochondroma are:

The underlying bone and the osteochondroma have continuous cortices. The medullary cavity of the bone and osteochondroma are continuous. The lesion may have a narrow stalk (pedunculated) or a broad base (sessile).

Secondary chondrosarcomas may develop in an osteochondroma. The radiographic features are:

Destruction of the underlying subchondral cortex of the osteochondroma Lytic destruction of the cartilage cap Formation of a soft tissue mass (often calcified)

Page 62: Hyperguides 2015 .Tumor

Magnetic resonance image shows a markedly thickened cap (larger than 1 cm in adults)

This patient has an osteochondroma. The most appropriate treatment is excision because the patient is symptomatic. Biopsy is not necessary prior to treatment and no further imaging is required.

79. Which of the following statements is false regarding liposarcomas:(A) Liposarcomas rarely develop from lipomas.(B) Liposarcomas rarely occur in children.(C) Most liposarcomas occur below the deep fascia.(D) Well-differentiated liposarcomas never have chromosomal abnormalities.(E) Reciprocal transformation (chromosome 12 and 16) occurs in myxoid liposarcoma.

Explanation:

Liposarcomas account for approximately 10% to 15% of sarcomas. Some general statements about liposarcomas are listed below:

1. Most liposarcomas occur below the fascia.2. Superficial liposarcomas have an excellent prognosis.3. Lipomas do not undergo transformation into liposarcomas.4. Liposarcomas rarely occur in children.5. Reciprocal transformation between chromosomes 12 and 16 occurs in myxoid and round

cell liposarcoma.6. Well-differentiated liposarcomas have giant marker and ring chromosome abnormalities

with amplification of MDM2, SAS, and GLI genes.7. Liposarcomas express peroxisome proliferator-activated receptor gamma (PPAR

gamma).8. Liposarcomas are not induced by irradiation.

80. Myxoid liposarcomas have which of the following chromosomal alterations:(A) Giant marker and ring chromosomes(B) Reciprocal translocation between chromosomes 12 and 16(C) Translocation between chromosomes 11 and 22(D) Reciprocal translocation of chromosomes X and 18(E) Translocation between chromosomes 9 and 22

Explanation:

Many benign and malignant soft tissue tumors have specific chromosomal abnormalities. A few common abnormalities include:

Well-differentiated liposarcoma Giant marker and ring chromosomesMyxoid liposarcoma Translocation between chromosomes 12 and 16Ewings/primitive Translocation between chromosomes 11 and 22

Page 63: Hyperguides 2015 .Tumor

neuroectodermal tumorsSynovial sarcoma Translocation between chromosomes X and 18Myxoid chondrosarcoma Translocation between chromosomes 9 and 22

81. Which of the following infections has been linked to the development of Paget’s disease:(A) Bacterial infection with spirochetes(B) Viral infection with a paramyxovirus(C) Viral infection with human immunodeficiency virus(D) Bacterial infection with streptococci(E) Bacterial infection with Yersinia

Explanation:

Infection with a paramyxovirus has been implicated in Paget’s disease. Inclusions that resemble viral nucleocapsid have been found with electron microscopy in the nuclei and cytoplasm of osteoclasts in Paget’s disease. It is still controversial whether the virus belongs to the measles, canine distemper, or respiratory syncytial viruses.

82. Which of the following is the most common form of malignant fibrous histiocytoma:(A) Storiform-pleomorphic(B) Myxoid(C) Giant cell(D) Inflammatory(E) Angiomatoid fibrous histiocytoma

Explanation:

The storiform-pleomorphic type is the most common form of malignant fibrous histiocytoma.

Myxoid: Second most common, accounts for 25% Giant cell: Contains numerous osteoclast-like giant cells Inflammatory: Most common in the retroperitoneum

83. A 25-year-old man has a 4-month history of severe thigh and knee pain. He has pain both at rest and at night. He has tenderness on palpation of the thigh. He has been febrile and has an elevated white blood cell count. The anteroposterior and lateral radiographs of the femur are shown (Slide1). The high- and lower-power histologic sections are also shown (Slide 2) and (Slide 3). The most appropriate treatment is:

Page 64: Hyperguides 2015 .Tumor

Slide 1 Slide 2 Slide 3

(A) Observation(B) Debridement and intravenous antibiotics(C) Methyl prednisolone acetate injection(D) Wide resection(E) Chemotherapy and external beam irradiation

Explanation:

The plain radiographs show permeative lytic lesion in the femur with periosteal reaction. The diagnostic considerations based upon the age of the patient and radiographic findings would be:Benign MalignantOsteomyelitis Ewing's tumorEosinophilic granuloma Osteosarcoma

Leukemia/lymphomaThe biopsy specimen shows a monotonous field of uniform blue cells. The biopsy is most consistent with Ewing's tumor.

Ewing's tumor is a high-grade malignancy with a propensity to metastasize to the lungs and bone marrow. The traditional treatment is multi-agent chemotherapy and external beam irradiation. Resection of the bone is used in selected patients in some centers.

Remember that there is a consistent chromosomal translocation, 11-22, in patients with Ewing's tumor [t (11;22)].

84. A 15-year-old boy has a soft, painful mass over the proximal portion of his elbow. The lateral plain radiograph is shown (Slide). Which of the following is the best form of treatment:

Page 65: Hyperguides 2015 .Tumor

Slide 1

(A) Angiography and sclerosing therapy(B) External beam irradiation(C) Chemotherapy(D) Surgical resection(E) Chemotherapy and surgical resection

Explanation:

The lateral radiograph shows multiple perfectly round radiodensities scattered throughout the soft tissues. These radiodensities are phleboliths, and they are virtually diagnostic of a soft tissue hemangioma.

Surgically, hemangiomas are difficult to treat because they have a high incidence of local recurrence. Symptomatic treatment with anti-inflammatory medications and compression dressing can be used. An interventional radiologist uses sclerosing therapy to treat patients who still have significant symptoms following symptomatic treatment. A sclerosing agent such as alcohol is used to close the numerous blood vessels.

85. A 10-year-old boy has shoulder pain following a fall onto his shoulder. The plain radiograph is shown (Slide). Which of the following is the most likely diagnosis:

Slide 1

(A) Osteosarcoma(B) Ewing’s tumor

Page 66: Hyperguides 2015 .Tumor

(C) Aneurysmal bone cyst(D) Unicameral bone cyst(E) Fibrous dysplasia

Explanation:

The plain radiograph shows a lytic lesion in the proximal humeral metaphysis. The medial and lateral cortices are thinned but intact (except for the minimally displaced fracture).

This radiograph is virtually diagnostic of unicameral bone cyst. The treatment is aspiration of the cyst and injection with methylprednisolone acetate.

86. Which of the following soft tissue sarcomas is most prone to metastasizing to lymph nodes:(A) Myxoid liposarcoma(B) Malignant fibrous histiocytoma(C) Clear cell sarcoma(D) Fibrosarcoma(E) Pleomorphic liposarcoma

Explanation:

Several soft tissue sarcomas are prone to spread to lymph nodes:

Synovial sarcoma Epithelioid sarcoma Rhabdomyosarcoma Clear cell sarcoma

87. Synovial sarcomas may display genetic abnormalities. Which of the following is the most common genetic alteration in synovial sarcoma:(A) Giant marker and ring chromosomes(B) Translocation between chromosomes 12 and 16(C) Translocation between chromosomes 11 and 22(D) Translocation between chromosomes X and 18(E) Translocation between chromosomes 9 and 22

Explanation:

Sarcoma►Well-differentiated liposarcoma►Myxoid liposarcoma ►Ewing's sarcoma/primitive neuroectodermal tumor►Synovial sarcoma

Genetic Alteration►Giant marker and ring chromosomes►Translocation between chromosomes 12 and 16

►Translocation between chromosomes 11 and 22►Translocation between chromosomes X and 18

Page 67: Hyperguides 2015 .Tumor

►Myxoid chondrosarcoma ►Translocation between chromosomes 9 and 22

88. A 14-year-old girl twisted her knee while playing lacrosse. Her anteroposterior (Slide 1) and lateral (Slide 2) radiographs are shown. The most likely cause of this condition is:

Slide 1 Slide 2

(A) Benign bone neoplasm(B) Malignant bone neoplasm(C) Benign bone developmental condition(D) Metabolic bone disease(E) Stress fracture

Explanation:

The plain radiographs show an oval lucent lesion in the metaphysis of the proximal tibia. Note that the lesion is based on the metaphysic. This is the classic appearance of a small nonossifying fibroma. In this case, there is no sclerotic rim.

Nonossifying fibromas of bone are best considered as variants of normal growth. Nonossifying fibromas are rare in children younger than 2 years old and commonly occur at the sites of muscle attachment.

89. A 55-year-old man has a large soft tissue mass in his arm. The T1- and T2-weighted images are shown in Slide 1 and Slide 2. The most likely diagnosis is:

Page 68: Hyperguides 2015 .Tumor

Slide 1 Slide 2

(A) Intramuscular lipoma(B) Glenohumeral ganglion(C) Malignant fibrous histiocytoma(D) Hemangioma(E) Hematoma

Explanation:

The T1-weighted image shows a large high-signal mass in the patient's arm. The signal quality exactly matches the subcutaneous fat. Note on the T2-weighted image with fat suppression that the mass completely suppresses (or turns black or low signal).

This patient has a lipoma or a benign soft tissue neoplasm. The patient can be treated with either observation or removal of the lipoma. The lipoma can be removed with a marginal or intralesional margin.

The other lesions refer to common soft tissue processes:

A malignant fibrous histiocytoma is low signal on T1-weighted images and high signal on T2-weighted images.

A hematoma shows fluid signal and is heterogeneous, with a low signal on T1-weighted images and high signal on T2-weighted images.

A glenohumeral ganglion is uniformly dark signal on T1-weighted images and high signal on T2-weighted images.

An arteriovenous malformation shows serpiginous channels that are low on T1-weighted imaged and high signal on T2-weighted images.

90. A 12-year-old girl has occasional thigh pain. Her plain radiographs are shown in Slide 1 and a biopsy specimen in Slide 2. Which of the following is the most likely genetic defect:

Page 69: Hyperguides 2015 .Tumor

Slide 1 Slide 2

(A) FGF receptor 3(B) EXT1, EXT2 genes(C) Gs alpha mutation(D) COMP, type IX collagen, (COL9A2)(E) Translocation between chromosomes X and 18

Explanation:

The plain radiographs show a lesion in the medullary cavity. Symmetric expansion of the bone is present. The cortices have been remodeled, but they are intact. The lesion has a mineralization pattern that is consistent with the ground glass appearance of fibrous dysplasia.

The biopsy specimen shows fragments of woven bone in a highly cellular fibrous stroma.

This child has fibrous dysplasia. One should remember that the genetic abnormality in fibrous dysplasia is caused by a mutation that affects the alpha subunit of the cell membrane-bound G protein (Gs alpha mutation).

The other answers refer to specific entities:

FGF receptor 3 - achondroplasia EXT1, EXT2 - multiple exostoses COMP, type IX collagen, (COL9A2) - multiple epiphyseal dysplasia Translocation between chromosomes X and 18 - synovial sarcoma

91. Malignant fibrous histiocytoma of bone:(A) Can be confused for fibrosarcoma of bone(B) Usually produces a significant osseous response(C) Are usually low grade(D) Are most commonly seen in the first 2 decades of life similar to Ewing’s sarcoma(E) Rarely metastasizes

Page 70: Hyperguides 2015 .Tumor

Explanation:

Malignant fibrous histiocytoma of bone is a lytic tumor of bone, which is often high grade. The peak age of patients is 50 years of age, and metastasis occurs in approximately 50% of patients.

92. Which of the following factors is most likely produced by the plasma cells in multiple myeloma to facilitate osteoclast activation:(A) Transforming growth factor beta (TGF-B)(B) Interleukin 4 (IL-4)(C) Interleukin 6 (IL-6)(D) Osteoprotegerin (OPG)(E) Parathyroid hormone-related protein (PTHrP)

Explanation:

The bone destruction in multiple myeloma is caused by release of the receptor activator of nuclear-kB ligand (RANKL) from the osteoblast surface. The receptor activator of nuclear-kB ligand binds to a receptor activator of nuclear-kB (RANK) on the osteoclast progenitor cell surface, and the process of osteoclast activation and bone destruction begins. Osteoprotegerin is a decoy inhibitor of the system and halts osteoclast activation.

Factors in multiple myeloma that cause osteoclast activation

IL-6 Macrophage inflammatory factor-alpha (acts directly on osteoclast precursors)

Factors that up regulate RANKL and down regulate OPG

Parathyroid hormone, PTHrP 1,25 dihydroxy vitamin D IL-1 beta Tumor necrosis factor-alpha Prostaglandin E2

Factors that down regulate RANKL and up regulate OPG

IL-4 Interferon N gamma TGF-B

93. Which of the following statements is true regarding synovial osteochondromatosis:(A) Synovial osteochondromatosis occurs mainly in women.(B) Synovial osteochondromatosis occurs predominantly in children.(C) Synovial osteochondromatosis usually develops polyarticularly.

Page 71: Hyperguides 2015 .Tumor

(D) Synovial osteochondromatosis is characterized by swelling.(E) Range of motion is always limited in patients with synovial osteochondromatosis.

Explanation:

The leading symptom in synovial osteochondromatosis is swelling of the affected joint.

Found equally in men and women Occurs between 30 and 50 years of age Condition is rarely polyarticular May develop without limitation of movement

94. Which of the following blocks the restriction checkpoint in the cell cycle (prohibits cells from passing from G1 to the S [synthesis] phase):(A) E2F(B) RB gene(C) Cyclin-dependent kinases(D) INK (inhibitor of CDK4)(E) p53

Explanation:

The retinoblastoma (RB) gene controls the checkpoint in regard to cells passing from the G1 (or G0 resting phase) into S, or synthesis, phase.

One should remember that the RB gene is phosphorylated by cyclin-dependent kinases (CDKs). When the RB protein is phosphorylated, it dissociates from E2f, which is a transcription factor for a number of enzymes for DNA synthesis.

The CDK inhibitors block the cell from progressing through the synthesis phase. These inhibitors include:

p21WAF/Cip1, p27Kip1, p57Kip2 INK

The important point is to remember that the retinoblastoma gene is a tumor suppressor gene and it controls the restriction point of the cell cycle.

95. Which of the following pathways must be inactivated for tumor cell growth:(A) Receptor activator of nuclear factor-kB ligand/receptor activator nuclear factor-kB(B) Receptor activator of nuclear factor-kB ligand/osteoprotegerin(C) Transforming growth factor beta/insulin-like growth factor 1(D) p53(E) Tumor necrosis factor/interleukin 6

Page 72: Hyperguides 2015 .Tumor

Explanation:

The retinoblastoma and p53 pathways are the two major regulators of the cell cycle progression from the resting phase to the S or synthesis phase. One should remember that phosphorylation of the retinoblastoma protein results in release of transcription factors, whereas the p53 pathway detects genetic damage and halts phosphorylation of the retinoblastoma protein.

Both of these pathways must be damaged to have progression of tumor cells through cell growth.

96. In which of the following cancers has chemotherapy shown an improvement in survival:(A) Dedifferentiated chondrosarcoma(B) Sarcoma complicating Paget’s disease(C) Ewing’s tumor(D) Parosteal osteosarcoma(E) High-grade intramedullary chondrosarcoma

Explanation:

Chemotherapy has dramatically improved the survival rate of patients with high-grade intramedullary osteosarcoma and Ewing’s tumor. The disease-free survival rate without chemotherapy is less than 20% for patients, whereas with modern multi-agent regimens, the disease-free survival rate approaches 70%. Periosteal osteosarcoma is an intermediate-grade osteosarcoma (risk of pulmonary metastases is 15% to 25%), which is generally treated with the same chemotherapy regimen as high-grade intramedullary osteosarcoma.

Chemotherapy has been used for a number of high-grade sarcomas with disappointing results:

Dedifferentiated chondrosarcoma High-grade intramedullary chondrosarcoma Sarcoma following Paget’s disease

Chemotherapy has no use with low-grade sarcomas:

Well-differentiated intramedullary osteosarcoma Parosteal osteosarcoma Low-grade intramedullary chondrosarcoma Adamantinoma Chordoma

97. Which of the following is an early effect of external beam irradiation when used as adjuvant therapy in the treatment of soft tissue sarcomas:(A) Stress fractures(B) Osteonecrosis(C) Postirradiation sarcoma(D) Delayed wound healing

Page 73: Hyperguides 2015 .Tumor

(E) Fibrosis

Explanation:

Preoperative radiation is commonly used in the treatment of soft tissue sarcomas. A frequently used regimen is 5040 cGy in 25 fractions (Monday to Friday, 180 cGy fractions daily for 5 weeks). The early effects such as radiation dermatitis are allowed to subside and, typically, at 2 to 4 weeks, the tumor is resected. Delayed wound healing is a significant problem; it occurs in up to 30% to 50% of patients. Many centers combine surgical resection with flap coverage to minimize the risk of wound dehiscence and infection.

Early side effects of treatment:

Radiation dermatitis Fatigue Delayed wound healing

Late side effects of treatment:

Stress fractures Postirradiation sarcomas Osteonecrosis Fibrosis/edema Neuropathy

98. A 55-year-old woman has a 15-year history of chronic knee pain. Her plain radiographs are shown in Slide 1. Axial and sagittal T1-weighted magnetic resonance scans are shown in Slide 2. A coronal and sagittal computed tomography reconstruction is shown in Slide 3 and a biopsy in Slide 4. Which of the following is the most likely recommended treatment:

slide 1 slide 2

Page 74: Hyperguides 2015 .Tumor

slide 3 slide 4

(A) Arthroscopic debridement and short-term antibiotics(B) Preoperative radiation and wide resection(C) Synovectomy and total knee replacement(D) Arthroscopic debridement and long-term antibiotics(E) Open debridement and long-term antibiotics

Explanation:

This patient has pigmented villonodular synovitis (PVNS). Patients often present with a long history of knee pain and recurrent atraumatic effusions.

The plain radiographs show large lucent areas on both sides of the knee joint. The lateral radiograph shows a large posterior erosion. The computed tomography reconstructions show large areas on bone lysis and erosions. Notice the prominent erosions on both sides of the joint. The magnetic resonance image shows large low signal masses and major areas of erosion.

The biopsy slide shows the typical histologic features of PVNS. Note that the macrophages have ingested hemosiderin pigment giving them a characteristic granular brown appearance.

Treatment for this patient is synovectomy and total knee replacement. The amount of bone and cartilage destruction is prominent; therefore, synovectomy alone is not sufficient.

99. A patient presents with increasing knee pain over a 2-year period. The anteroposterior and lateral radiographs are shown in Slide 1 and Slide 2. A computed tomography scan is shown in Slide 3 and axial T2-weighted magnetic resonance images in Slide 4 and Slide 5. A biopsy is shown in Slide 6. The most likely diagnosis is:

Page 75: Hyperguides 2015 .Tumor

slide 1 slide 2 slide 3

slide 4 slide 5 slide 6

(A) Synovial chondromatosis(B) Pigmented villonodular synovitis(C) Rheumatoid arthritis(D) Gout(E) Tuberculosis

Explanation:

This patient has pigmented villonodular synovitis. Patients often present with a long history of knee pain and recurrent atraumatic effusions.

The plain radiographs are often fairly normal in cases involving the knee. The lateral radiograph may show subtle erosion. The computed tomography scan shows erosions on both sides of the proximal tibiofibular joint. The axial magnetic resonance images show the characteristic large low-signal masses.

Slide 6 shows the typical histologic features of pigmented villonodular synovitis. Note that the macrophages have ingested hemosiderin pigment, giving them a characteristic granular appearance.

Page 76: Hyperguides 2015 .Tumor

100. Which of the following directly promotes tumor cell metastasis:

(A) NM-23(B) MKK4(C) Receptor activator of nuclear factor kappa B ligand (RANKL)(D) Activated RAS(E) RKIP

Explanation:

Activated RAS begins the Raf-MEK-ERK-MAPK pathway, which causes tumor cell growth and metastasis.

Tumor suppressor genes can inhibit the metastatic process: NM-23 – phosphorylates the Ksr protein RKIP – competitive inhibitor of MEK MKK4 – causes stress-induced apoptosis

RANKL binds to the RANK receptor and activates the osteoclast precursor cells.

101. Which of the following tumors has the best prognosis (least risk of pulmonary metastases):(A) Rhabdomyosarcoma(B) Primitive neuroectodermal tumor(C) Malignant fibrous histiocytoma(D) Myxoid liposarcoma(E) Pleomorphic liposarcoma

Explanation:

All of the lesions listed in the possible answers are high-grade soft tissue sarcomas, except for myxoid liposarcoma, which is an intermediate-grade sarcoma. Generally, low-, intermediate-, and high-grade sarcomas have the following approximate risk of the patient developing pulmonary metastases:

Grade Approx. Risk Tumor

Low (Grade 1) 5%-10% Grade 1 liposarcoma, Grade malignant fibrous histiocytoma

Intermediate (Grade 2) 20%-30% Myxoid liposarcoma

High (Grade 3,4) 50%+

Rhabdomyosarcoma, primitive neuroectodermal tumor, malignant fibrous histiocytoma, pleomorphic liposarcoma

Page 77: Hyperguides 2015 .Tumor

102. What is the most common site of bone metastases:(A) Proximal femur(B) Proximal humerus(C) Pelvis(D) Spine(E) Hands/feet

Explanation:

Bone metastases most commonly occur in the axial skeleton, with the vertebral bodies as the most common site. The common distribution is to:

Spine Ribs Pelvis Skull Proximal long bones

Lesions in the hands and feet are uncommon. When these distal metastases do occur, approximately one-half will be secondary to lung cancers.

103. Hypercalcemia commonly occurs in which of the following bone lesions:(A) Osteosarcoma(B) Multiple myeloma(C) Malignant fibrous histiocytoma(D) Chondrosarcoma(E) Hemangioendothelioma

Explanation:

Serum hypercalcemia is a common problem in cancer patients. The most common cancers in which this complication may occur include:

Multiple myeloma Lymphoma Metastatic lung cancer Metastatic breast cancer

The symptoms of hypercalcemia include:

Acute Chronic

Page 78: Hyperguides 2015 .Tumor

Anorexia Visual changes

Constipation Coma

Polyuria/polydipsia

Nausea/vomiting

Lethargy

Inability to concentrate

Need for more sleepThe treatment of hypercalcemia is threefold:

Hydration Saline diuresis Diphosphonates: Intravenous pamidronate can correct the hypercalcemia within 48 hours

104. Which of the following descriptions would apply to the radiographic findings in a patient with adamantinoma:

(A) Multiple poorly demarcated lytic lesions (>5)(B) Eccentric, lytic, expansile lesion in the proximal tibial metaphysis(C) Eccentric, cortically based lytic lesion with a well-formed sclerotic rim(D) Multiple lucencies in the tibial diaphysis with sclerosis of the intervening bone(E) Central medullary mineralized lesion with rings and stipples; no cortical abnormalities

Explanation:

Adamantinoma is a peculiar and rare bone tumor that almost exclusively involves the tibia. In some cases, when adamantinoma occurs in the tibia and fibula, there are multicentric lesions. If one finds multiple bone involvement (sites other than the tibia and fibula), the diagnosis of adamantinoma is unlikely.

The radiographic appearance is quite characteristic. The most common appearance is mixed sclerotic and lytic lesions. Often, one will find multiple lucencies with intervening sclerosis of the bone. Seventy percent of the lesions involve the mid-shaft of the tibia, with the remaining lesions involving the proximal or distal tibia.

The other possible answers describe other bone lesions:

Aneurysmal bone cyst: An eccentric, lytic, expansile lesion in the proximal tibial metaphysis.

Non-ossifying fibroma: An eccentric, cortically based lytic lesion with a well-formed sclerotic rim.

Page 79: Hyperguides 2015 .Tumor

Enchondroma: A central medullary mineralized lesion with rings and stipples that has no cortical abnormalities.

105. The most common radiographic appearance of a chordoma is:(A) Marked sclerosis(B) Mixed lytic-sclerosis(C) Purely lytic with scattered calcific densities(D) Marked cortical thickening with coarsened trabeculae(E) Vertebral collapse with flattening and widening

Explanation:

Chordomas most commonly have a purely lytic appearance. Occasionally, they may be purely sclerotic or have a mixed appearance of lytic-sclerosis.

In the sacrococcygeal area, chordomas are typically purely lytic. The tumor grows anteriorly and expands centripetally with a periosteal layer. The soft tissue mass often has calcific densities scattered throughout. These calcifications are usually mild to moderate.

106. Which of the following soft tissue sarcomas most commonly occurs in children younger than 10 years old:

(A) Malignant fibrous histiocytoma(B) Fibrosarcoma(C) Synovial sarcoma(D) Rhabdomyosarcoma(E) Epithelioid sarcoma

Explanation:

Rhabdomyosarcoma is the most common soft tissue sarcoma in children younger than 10 years old. The most malignant subtype of rhabdomyosarcoma, alveolar rhabdomyosarcoma, occurs in the trunk and extremities.

107. Which of the following soft tissue sarcomas is the most responsive to chemotherapy:(A) Malignant fibrous histiocytoma(B) Fibrosarcoma(C) Synovial sarcoma(D) Rhabdomyosarcoma(E) Epithelioid sarcoma

Explanation:

Rhabdomyosarcoma is the most common soft tissue sarcoma in children. Rhabdomyosarcoma is very sensitive to both chemotherapy and radiation. Long-term survival in children who present

Page 80: Hyperguides 2015 .Tumor

with localized disease is 60% to 70%.

108. Which of the following describes the typical radiographic appearance of eosinophilic granuloma in the spine:(A) Expansive, destructive lesion in the posterior elements(B) A 10-mm lytic area with surrounding sclerosis in the posterior elements(C) Diffuse osteopenia in the vertebral body(D) Completely collapsed vertebral body with increased width(E) Destruction of a pedicle on one side

Explanation:

Eosinophilic granuloma has a characteristic appearance in the spine. There is complete collapse of the vertebra and the vertebral body is often widened. This almost pathognomonic appearance is called 'vertebra plana.' Occasionally, Ewing's tumor or leukemia can have a similar appearance.

The other possible answers describe different entities:

Osteoblastoma or aneurysmal bone cyst:

Expansile, destructive lesion in the posterior elements

Osteoid osteoma:

A 10-mm lytic area with surrounding sclerosis in the posterior elements

Multiple myeloma:

Diffuse osteopenia in the vertebral body

Metastatic bone disease:

Destruction of a pedicle on one side

109. A 26-year-old man had Ewing’s tumor of the proximal humerus 14 years ago. He was treated with multi-agent chemotherapy and external beam irradiation. He now has pain in the proximal humerus and his radiographs show a lytic lesion with a small area of cortical bone destruction. The most likely diagnosis:

(A) Stress fracture(B) Radiation osteitis(C) Osteomyelitis(D) Recurrent Ewing’s tumors(E) Post-irradiation sarcoma

Explanation:

Patients who have been treated with irradiation may develop two principal problems in their

Page 81: Hyperguides 2015 .Tumor

skeleton — insufficiency, or stress fractures, and post-irradiation sarcomas.

Fortunately, post-irradiation sarcomas are rare, but they may occur in a patient who has been cured of their cancer. Patients may have received either radiation alone or a combination of chemotherapy and radiation. The latency period (interval between the completion of the radiation therapy and onset of the sarcoma) varies between 5 to 50+ years. There have been some reports with post-irradiation sarcomas occurring within 2 to 5 years.

The symptoms of a post-irradiation sarcoma are pain and swelling. Occasionally, patients present with a mass only.

The key radiographic feature of a post-irradiation sarcoma is cortical bone destruction and the presence of a soft tissue mass. The cortical bone destruction may be appreciated on plain radiographs alone or with CT or MRI scans.

NOTE: When patients with a history of cancer and irradiation present with skeletal pain and normal radiographs, the clinician should search further with a cross-sectional imaging modality, such as CT or MRI scans.

110. Which of the following osteosarcomas has minimal osteoid production when examined histologically:(A) Parosteal osteosarcoma(B) Periosteal osteosarcoma(C) Well-differentiated intramedullary osteosarcoma(D) High-grade intramedullary osteosarcoma(E) Telangiectatic osteosarcoma

Explanation:

Telangiectatic osteosarcomas are the one form of osteosarcoma where there may be minimal or no osteoid production by the tumor cells. The radiographs typically show a destructive, purely lytic lesion that is often expansile.

On low power viewing, there are usually large blood-filled spaces. The low-power appearance may be very similar to that seen with aneurysmal bone cyst. On high-power viewing, the cells that are in the cyst lining are malignant. The nuclei show pleomorphism and atypical features. There are often bizarre mitotic figures.

111. A 50-year-old woman presented with a 1-year history of shin and knee pain. A presumptive diagnosis of Paget disease had been made and she was treated with anti-Pagetic medications. Her pain continued and a new anteroposterior radiograph of her femur was taken (Slide 1). A coronal and axial magnetic resonance imaging scan (Slides 2 and Slide 3) and a biopsy of the soft tissue mass were also taken (Slide 4).Immunostaining was negative for keratin but postive for CD20

Page 82: Hyperguides 2015 .Tumor

The most likely diagnosis is:

Slide 1 Slide 2

Slide 3 Slide 4

(A) Metastatic bone disease(B) Myeloma of bone(C) Lymphoma(D) Osteosarcoma of bone(E) Sarcoma complicating Paget disease

Explanation:

The histology of lymphoma is a mixed blue cell infiltrate of round cells, which may be small and large

112. A 45-year-old man has a 4-month history of severe pain in his shoulder. His plain radiograph (Slide 1) and biopsy (Slide 2) are presented. The most appropriate treatment method is:

Page 83: Hyperguides 2015 .Tumor

Slide 1 Slide 2

(A) Shoulder disarticulation(B) Wide resection and allograft reconstruction(C) External beam irradiation(D) Internal fixation with external beam irradiation(E) Neck exploration and parathyroidectomy

Explanation:

The plain radiograph shows a destructive and purely lytic lesion. More than 50% of the cortical bone has been destroyed. This pattern of permeative bone destruction is consistent with a malignancy. The biopsy shows round, epithelial-like cells with clear cytoplasm. The cells are organized in clusters. This is metastatic clear cell carcinoma of the kidney.

This patient is prone to fracture. The humeral lesion meets the criteria of an impending fracture. The best treatment is internal fixation with an intramedullary rod and postoperative external beam irradiation. Plate fixation with methylmethacrylate supplementation would be another reasonable choice.

113. A 66-year-old man has a 4-month history of hip discomfort. The plain radiographs are shown in Slide 1 and a biopsy is shown in Slide 2. The most likely diagnosis is:

Page 84: Hyperguides 2015 .Tumor

Slide 1 Slide 2

(A) Metastatic bone disease(B) Lymphoma(C) Multiple myeloma(D) Chondrosarcoma(E) Malignant fibrous histiocytoma

Explanation:

The plain radiograph of the pelvis shows a lytic lesion in the supra-acetabular area. A destructive lesion in the pelvis may be secondary to five common processes in patients who are 40 to 80 years of age, including metastatic bone disease, multiple myeloma, lymphoma, chondrosarcoma, and malignant fibrous histiocytoma. Often a biopsy is necessary to establish the diagnosis. The biopsy specimen shows plasma cells. The plasma cells have these characteristics:

Eccentrically placed nucleus Peripheral clumping of the nuclear chromatin A perinuclear halo

The diagnosis is multiple myeloma. The treatment would be external beam irradiation.

114. A 60-year-old man has a 6-month history of increasing knee pain. He has discomfort at night and at rest. His anteroposterior and lateral radiographs are shown in Slide 1, and low and high power biopsy specimens are shown in Slides 2 and 3. The treatment of this lesion would be:

Page 85: Hyperguides 2015 .Tumor

Slide 1 Slide 2

Slide 3

(A) Intramedullary rod fixation and external beam irradiation(B) Curettage, cement augmentation and plate fixation(C) Wide resection and custom prosthetic replacement(D) External beam irradiation alone(E) Neck exploration and parathyroidectomy

Explanation:

The plain radiographs show two major features. The proximal portion of the lesion (top half) shows an intramedullary lesion, which is mineralized. The mineralization shows rings and stipples. There is also thickening of the cortex of the femur. This thickening is circumferential on the medial, lateral, anterior, and posterior cortices. These features strongly suggest a cartilage tumor, principally a chondrosarcoma. The distal (lower half) of the lesion has a different pattern. The intramedullary lesion is completely lytic, and the lateral cortex has been destroyed. This bimorphic pattern is seen with dedifferentiated chondrosarcoma. The bimorphic pattern is a low-grade chondrosarcoma and a high-grade anaplastic spindle cell sarcoma.

The histologic appearance is also bimorphic. Notice the cartilage, which is relatively hypocellular, and the intimate admixture of a high-grade spindle cell sarcoma. The high-

Page 86: Hyperguides 2015 .Tumor

grade spindle sarcoma is characterized by pleomorphic and pyknotic nuclei–dark staining nuclear chromatin, and different sizes and shapes of the nuclei. The bimorphic pattern is low-grade cartilage admixed with high-grade spindle cell sarcoma. This high-grade spindle cell sarcoma can be osteosarcoma, malignant fibrous histiocytoma, or fibrosarcoma.

The treatment of dedifferentiated chondrosarcoma is wide resection and custom replacement. Amputation with a wide margin would be another option.

The prognosis is poor with a long-term survival of only 20% to 40%. The most common site of metastases is to the lungs.

115.116. A 60-year-old man has a 6-month history of increasing knee pain. He has discomfort at night and at rest. His anteroposterior and lateral radiographs are shown in Slide 1, and low and high power biopsy specimens are shown in Slides 2 and 3. The prognosis for this patient would be:

Slide 1 Slide 2 Slide 3(A) Excellent (> 90% long-term survival)(B) Good (> 50% long-term survival)(C) Poor (< 25% long-term survival)(D) Dismal (no long-term survivors)(E) Dismal (no long-term survivors but prolonged disease course)

Explanation:

The plain radiographs show two major features. The proximal portion of the lesion (top half) shows an intramedullary lesion, which is mineralized. The mineralization shows rings and stipples. There is also thickening of the cortex of the femur. This thickening is circumferential on the medial, lateral, anterior, and posterior cortices. These features strongly suggest a cartilage tumor, principally a chondrosarcoma. The distal (lower half) of the lesion has a different pattern. The intramedullary lesion is completely lytic, and the lateral cortex has been destroyed. This bimorphic pattern is seen with dedifferentiated chondrosarcoma. The bimorphic pattern is a low-grade chondrosarcoma and a high-grade anaplastic spindle cell sarcoma.

The histologic appearance is also bimorphic. Notice the cartilage, which is relatively hypocellular, and the intimate admixture of a high-grade spindle cell sarcoma. The high-grade spindle sarcoma is characterized by pleomorphic and pyknotic nuclei–dark staining nuclear chromatin, and different

Page 87: Hyperguides 2015 .Tumor

sizes and shapes of the nuclei. The bimorphic pattern is low-grade cartilage admixed with high-grade spindle cell sarcoma. This high-grade spindle cell sarcoma can be osteosarcoma, malignant fibrous histiocytoma, or fibrosarcoma.

Dedifferentiated chondrosarcomas are very high-grade lesions with a poor prognosis. Long-term survival is < 25% of patients.

117. A 65-year-old woman has a 5-month history of increasing hip and thigh pain. Her plain radiographs are shown in Slide 1. She has severe pain with weight bearing on the right leg. T1 and T2 weighted coronal images of her hip are shown in Slides 2 and 3 respectively. A biopsy is performed and is shown in Slide 4. The cells directly responsible for the bone destruction would be:

Slide 1 Slide 2

Slide 3 Slide 4

Explanation:

(A) Tumor cells(B) Fibroblasts(C) Osteoclasts(D) Histiocytes(E) Plasma cells

Page 88: Hyperguides 2015 .Tumor

The plain radiographs show an ill-defined permeative lesion in the proximal femur. The lateral cortex of the greater trochanter is thinned. The T1 weighted coronal images show a low signal lesion in the entire greater trochanter extending to the base of the femoral neck. Notice in the opposite proximal femur and acetabulum, there are multiple small low signal lesions consistent with metastatic bone disease. The T2 weighted images show a very high signal area in the greater trochanter. The inversion recovery image is very sensitive to pathologic processes. Normal marrow will be very low signal while pathologic lesions are high signal. The biopsy shows round epithelial cells forming glands. This pattern is that of metastatic adenocarcinoma.

The bone destruction in metastatic bone disease is a direct result of osteoclastic resorption of the bone. The metastatic breast cells secrete a factor that activates and recruits osteoclasts, which then destroy the bone. Diphosphonate therapy (usually intravenous pamidronate) is given to virtually all women with metastatic breast carcinoma to bone to stop the osteoclasts from resorbing the bone matrix.

118. Which of the following are the most common sites of metastases in patients with malignant fibrous histiocytoma of bone:(A) Bones(B) Lymph nodes(C) Lungs(D) Lungs and lymph nodes(E) Bones and lymph nodes

Explanation:

The most common site of metastases in malignant fibrous histiocytoma is to the lungs. Metastases to lymph nodes are rare.

119. A 45-year-old woman has hip discomfort with prolonged sitting, and she feels a mass. The plain radiographs and computerized tomography scan are shown in Slides 1, 2, and 3. Her symptoms have increased, and she desires surgical treatment. The most appropriate treatment is:

Slide 1 Slide2 Slide3

Page 89: Hyperguides 2015 .Tumor

(A) Needle biopsy(B) Incisional biopsy(C) Removal of the mass (excisional biopsy)(D) Intramedullary rod fixation followed by irradiation(E) Resection/proximal femoral replacement

Explanation:

The anteroposterior radiograph shows a lesion in the greater trochanter, and the oblique radiograph shows a bony mass projecting from the proximal femur. The computerized tomography scan shows a lesion arising from the posterior cortical surface, which shares its cortices with the host bone. In addition, the medullary cavity of the proximal femur flows into the lesion. These findings are consistent with an osteochondroma. There is no evidence of malignant change.The radiographic findings of an osteochondroma are:

Underlying bone and the osteochondroma have continuous cortices. Medullary cavity of the bone and osteochondroma are continuous.

Secondary chondrosarcomas may develop in an osteochondroma. The radiographic features are:

Destruction of the underlying subchondral cortex of the osteochondroma Lytic destruction of the cartilage cap Formation of a soft tissue mass (often calcified)

The most appropriate treatment is removal of the mass through a posterior approach. Internal fixation may not be necessary.

120. Which of the following statements is false regarding liposarcomas:(A) Most liposarcomas occur below the fascia.(B) Lipomas may transform into a liposarcoma(C) Liposarcomas rarely occur in children.(D) Reciprocal transformation (chromosome 12 and 16) occurs in myxoid liposarcoma.(E) Liposarcomas are not induced by irradiation.

Explanation:

Liposarcomas account for approximately 10% to 15% of sarcomas. Some general statements about liposarcomas are listed below:

1. Most liposarcomas occur below the fascia.2. Superficial liposarcomas have an excellent prognosis.3. Lipomas do not undergo transformation into liposarcomas.4. Liposarcomas rarely occur in children.5. Reciprocal transformation between chromosomes 12 and 16 occurs in myxoid and round

cell liposarcoma.6. Well-differentiated liposarcomas have giant marker and ring chromosome abnormalities

with amplification of MDM2, SAS, and GLI genes.

Page 90: Hyperguides 2015 .Tumor

7. Liposarcomas express peroxisome proliferator-activated receptor gamma (PPAR gamma).8. Liposarcomas are not induced by irradiation.

121. Well-differentiated lipoma-like liposarcomas have which of the following characteristic chromosomal alterations:(A) Giant marker and ring chromosomes(B) Reciprocal translocation between chromosomes 12 and 16(C) Translocation between chromosomes 11 and 22(D) Reciprocal translocation of chromosomes X and 18(E) Translocation between chromosomes 9 and 22

Explanation:

Many benign and malignant soft tissue tumors have specific chromosomal abnormalities. A few common abnormalities include:

Well-differentiated liposarcoma Giant marker and ring chromosomes

Myxoid liposarcoma Translocation between chromosomes 12 and 16Ewings/primitive neuroectodermal tumors Translocation between chromosomes 11 and 22

Synovial sarcoma Translocation between chromosomes X and 18Myxoid chondrosarcoma Translocation between chromosomes 9 and 22

122. In which of the following countries is Paget’s disease most rare:(A) Europe and North America(B) Australia and New Zealand(C) France, Spain, and West Germany(D) Italy and Greece(E) China, India, and Malaysia

Explanation:

Paget’s disease is common in all countries except China, India, Malaysia, and sub-Saharan Africa.

Paget’s disease is common in people older than 55 years in the following countries:

Europe North America England (4.6%) France (2.4%) Ireland (0.7 to 1.7%) Spain (1.3%) West Germany (1.3%) Italy (0.5%)

Page 91: Hyperguides 2015 .Tumor

Greece (0.5%

123. Which of the following tests or procedures is most commonly used to establish the diagnosis of Paget’s disease:(A) Technetium bone scan(B) Serum alkaline phosphatase(C) Urinary hydroxyproline(D) Bone biopsy(E) Plain radiographs

Explanation:

Plain radiographs are commonly used to diagnose Paget’s disease. The radiographic features include:

Enlargement and expansion of the bone Coarsened trabeculae Cortical thickening Lucent and sclerotic changes

Technetium bone scans are sensitive to sites of involvement. In general, at the time of diagnosis, there will not be a change in the number of the involved bones.

Serum alkaline phosphatase level is elevated in approximately 95% of patients. Urinary markers of bone turnovers will also be elevated and include hyrodroxyproline, N-telopeptide, and deoxypyridinoline.

Bone biopsy is seldom required to diagnose Paget’s disease.

124. Which of the following statements is false concerning tumoral calcinosis:(A) Most patients are black.(B) Serum calcium levels are usually normal.(C) The periarticular masses are intracapsular and restrict motion.(D) Serum phosphate levels are high.(E) The most common inheritance is autosomal recessive.

Explanation:

Tumoral calcinosis is a heritable condition that is characterized by periarticular metastatic calcification. Most patients are black, and the inheritance is usually autosomal recessive. Metastatic calcifications occur around joints and in the skin, marrow, teeth, and blood vessels. The periarticular masses may grow quite large and are attached to the fascia, but they are extra-articular. The masses may occur at the shoulder, hip, and elbow.

Radiographically: The masses are composed of heavy, amorphous calcification in nodules. Laboratory:

Page 92: Hyperguides 2015 .Tumor

1. Serum calcium normal2. Serum alkaline phosphatase normal3. Urine calcium is low4. Serum phosphate is often high

When symptomatic, the masses are excised.

125. Which of the following is the most common soft tissue tumor of infancy and childhood:(A) Rhabdomyosarcoma(B) Hemangioma(C) Nodular fasciitis(D) Tendon sheath fibroma(E) Synovial sarcoma

Explanation:

Hemangiomas are the most common soft tissue tumors in infancy and childhood.Rhabdomyosarcoma is the most common sarcoma in children and synovial sarcoma is the most common soft tissue sarcoma of the foot and ankle.

126. A 58-year-old woman has diffuse pelvic and right hip pain. She has pain at rest and when she ambulates. The plain radiograph and computed tomography scan are shown in Slide 1 and Slide 2, respectively. A biopsy specimen is shown in Slide 3. The most likely diagnosis is:

Slide 1 Slide 2 Slide 3

(A) Lymphoma(B) Metastatic bone disease(C) Multiple myeloma(D) Chondrosarcoma(E) Malignant fibrous histiocytoma

Page 93: Hyperguides 2015 .Tumor

Explanation:

The plain radiograph shows a sclerotic lesion in the intertrochanteric region, ilium, ischium, and sacrum. The lesions are densely sclerotic. The biopsy specimen shows epithelial cells in clusters forming glands. These cells have completely replaced the bone marrow.

This patient has metastatic breast carcinoma.

127. A 55-year-old man presented with a 12-month history of increasing low back pain. He experiences pain during the night and often has difficulty sitting. He has also noticed that in the last 9 months constipation has become an increasing problem. The plain radiographs and computerized tomography (CT) scan are shown (Slide 1 and Slide 2). Low- and high-power histologic sections are also presented (Slide 3 and Slide 4). The most appropriate treatment is:

Slide 1 Slide 2

Slide 3 Slide 4

(A) Chemotherapy and radiation(B) External radiation(C) Observation and protected weight bearing(D) Chemotherapy and wide resection(E) Wide resection

Explanation:

The plain radiograph of the pelvis shows the sacrum and sacroiliac joints. The CT scan shows

Page 94: Hyperguides 2015 .Tumor

destruction of the sacrum with an anterior soft tissue mass. The low-power photomicrograph shows a lobular tumor. The high-power photomicrograph shows physaliferous cells in a myxoid background.

Both the CT scan and biopsy are consistent with a chordoma.

Chordoma is a low-grade malignant tumor that arises from notochordal cells. As a low-grade tumor, metastases occur late in the disease. Local failure is common following surgical resection. Progressive local disease accounts for the poor 10-year disease-free survival of only 25%.

128. A 30-year-old attorney has a 6-month history of increasing knee pain. His pain occurs both with activity and at rest. He is otherwise healthy. Laboratory studies show a normal complete blood count, sedimentation rate, and chemistry values (including normal calcium and phosphorus levels). Anteroposterior and lateral radiographs are shown (Slide 1 and Slide 2). The low- and high-power hematoxylin and eosin sections are shown (Slide 3 and Slide 4). The most appropriate treatment is:

Slide 1 Slide 2

Page 95: Hyperguides 2015 .Tumor

Slide 3 Slide 4

(A) Observation(B) Curettage(C) External beam irradiation(D) Chemotherapy and external beam irradiation(E) Wide resection and reconstruction

Explanation:

The radiograph shows purely lytic bone destruction that fills the lateral metaphysis and extends to the articular surface. The lateral cortex is very thin and almost completely destroyed. The histologic sections show collection of giant cells and mononuclear cells. Notice that the giant cells are uniformly scattered throughout the histologic section on the low power, in contrast to chondroblastoma where the giant cells are scattered throughout the mononuclear cells and that the nuclei of the mononuclear cells are the same as the nuclei of the giant cells.

The diagnosis is giant cell tumor of bone. The treatment of giant cell tumor of bone is complete curettage of the tumor. Wide exteriorization (extensive removal of the cortex over the lesion), curettage with hand and power instruments, and chemical cauterization is necessary to reduce the chance of local recurrence. Cement augmentation is often used to fill the large cavity following curettage.

129. Which of the following serum tests is most useful in the diagnosis of Paget’s disease:(A) Parathyroid hormone(B) Calcium level(C) Alkaline phosphatase(D) Phosphate level(E) Lactic dehydrogenase

Explanation:

Paget disease is a remodeling disease of bone. The osteoclast is the responsible cell. There is an increase in size, number, and activity. The treatment of Paget disease is aimed at the activity of the osteoclast. Diphosphonates are the mainstay of treatment.

The alkaline phosphatase level is elevated in 95% of patients. Collagen breakdown products can also be detected in the urine — hydroxy proline or N-telopeptide. Following treatment, there is a reduction in serum alkaline phosphate levels.

130. A woman with newly diagnosed breast cancer has multiple liver and pulmonary metastases and a lytic lesion in the midshaft of the left femur with 20% to 25% cortical bone destruction in a symmetric manner involving the femoral endosteal surfaces. She has no pain with weight bearing. Which of the following is the most accepted method to treat patients with femoral metastasis:(A) External beam irradiation alone

Page 96: Hyperguides 2015 .Tumor

(B) External beam irradiation and diphosphonate therapy(C) Intramedullary rod fixation and external beam irradiation(D) Intramedullary rod fixation, diphosphonate therapy, and external beam irradiation(E) Curettage and plate fixation, external beam irradiation

Explanation:

Patients with metastases to long bones may require prophylactic fixation. Plain radiographs are evaluated for the amount of cortical bone destruction and patients are asked if they have weight bearing pain. When cortical bone destruction exceeds 50% of the cortical diameter, the risk of fracture is high. When prominent weight bearing pain is present, the risk of fracture is also greater. This patient has a small amount of cortical bone destruction and no weight bearing pain. She can be managed nonoperatively.

External beam irradiation is used to halt the bone destruction by killing the tumor cells. Diphosphonate therapy is used to stop the osteoclastic bone resorption both in the femoral shaft and at other sites.

131. A 45-year-old man has a 5-cm hard mass over the extensor compartment of his forearm located below the fascia. A needle biopsy is shown (Slide). The most likely diagnosis is:

Slide 1

(A) Fibrosarcoma(B) Nodular fasciitis(C) Schwannoma(D) Extra-abdominal desmoid tumor(E) Soft tissue Ewing’s tumor

Explanation:

A soft tissue mass that is located below the fascia should raise the suspicion of a sarcoma. All sarcomas have a common imaging pattern - low signal on T1-weighted sequences and high signal on T2-weighted sequences.

The biopsy shows spindle cells (fibroblasts) arranged in fascicles. Some of the nuclei are cut longitudinally, causing a spindle-shaped nuclei, whereas other nuclei are cut in cross section and

Page 97: Hyperguides 2015 .Tumor

appear round. The interlacing of the fascicles is termed a herringbone pattern. This patient has a fibrosarcoma.

132. A 14-year-old girl twisted her knee while playing lacrosse. Her anteroposterior (Slide 1) and lateral (Slide 2) radiographs are shown. The most likely diagnosis is:

Slide 1 Slide 2

(A) Osteomyelitis(B) Stress fracture(C) Bone island(D) Nonossifying fibroma(E) Enchondroma

Explanation:

The plain radiographs show an oval lucent lesion in the metaphysis of the proximal tibia. Note that the lesion is based on the metaphysic. This is the classic appearance of a small nonossifying fibroma. In this case, there is no sclerotic rim.Nonossifying fibromas of bone are best considered as variants of normal growth. Nonossifying fibromas are rare in children younger than 2 years old and commonly occur at the sites of muscle attachment.

133. A 14-year-old girl twisted her knee while playing lacrosse. Her anteroposterior (Slide 1) and lateral (Slide 2) radiographs are shown. The most appropriate treatment is:

Page 98: Hyperguides 2015 .Tumor

Slide 1 Slide 2

(A) Curettage and bone grafting(B) Aspiration and cortisone injection(C) En bloc resection and grafting(D) Computed tomography-guided radiofrequency ablation(E) Observation

Explanation:

The plain radiographs show an oval lucent lesion in the metaphysis of the proximal tibia. Note that the lesion is based on the metaphysic. This is the classic appearance of a small nonossifying fibroma. In this case, there is no sclerotic rim.

Nonossifying fibromas of bone are best considered as variants of normal growth. Nonossifying fibromas are rare in children younger than 2 years old and commonly occur at the sites of muscle attachment.

Most nonossifying fibromas are asymptomatic and can be treated with observation. In this case, the nonossifying fibroma is so small that observation with a new radiograph in 3 months is all that is necessary. No activity restrictions are needed.

134. Which of the following are classic findings in Langerhans cell histiocytosis of the spine:(A) Vertebral collapse, preservation of the disk space, absence of a soft tissue mass(B) Vertebral collapse, disk space narrowing, absence of a soft tissue mass(C) Vertebral collapse, preservation of the disk space, anterior soft tissue mass(D) Vertebral expansion, disk space narrowing, absence of a soft tissue mass(E) Vertebral expansion, disk space narrowing, presence of a soft tissue mass

Explanation:

Patients with Langerhans cell histiocytosis may develop spinal involvement.

Page 99: Hyperguides 2015 .Tumor

One should remember these points from a recent study by Garg and colleagues.

Cervical spine involvement is common. Multilevel disease is common especially in the cervical and lumbar regions. Anterior elements are involved in 95% of patients. Vertebral body collapse is common and may be symmetric or asymmetric. The most common symptom in patients is dull low back pain. Neurological defects are uncommon. Symmetric vs. asymmetric collapse is not a prognostic factor for long-term deformity. Short-term bracing is effective. Approximately 10% of patients may need spinal deformity correction.

135. Which of the following is true concerning Langerhans cell histiocytosis (eosinophilic granuloma) of the spine:(A) Dull back pain without neurological symptoms is the most common symptom.(B) The most common site of involvement is the posterior element.(C) Narrowing of disk space is common.(D) Vertebral body reconstruction with correction of deformity is necessary with more than

50% collapse.(E) Multicentric spine involvement is rare.

Explanation:

Patients with Langerhans cell histiocytosis may develop spinal involvement.

One should remember these points from a recent study by Garg and colleagues.

Cervical spine involvement is common. Multilevel disease is common especially in the cervical and lumbar regions. Anterior elements are involved in 95% of patients. Vertebral body collapse is common and may be symmetric or asymmetric. The most common symptom in patients is dull low back pain. Neurological defects are uncommon. Symmetric vs. asymmetric collapse is not a prognostic factor for long-term deformity. Short-term bracing is effective. Approximately 10% of patients may need spinal deformity correction

136. A 22-year-old man has intermittent aching pain in his left hip. His plain radiograph is shown in Slide 1 and the T1- and T2-weighted coronal magnetic resonance image (MRI) scans are shown in Slide 2 and Slide 3. The most likely diagnosis is:

Page 100: Hyperguides 2015 .Tumor

Slide 1

Slide 2 Slide 3

(A) Pigmented villonodular synovitis(B) Rheumatoid arthritis(C) Synovial chondromatosis(D) Tumoral calcinosis(E) Heterotopic ossification

Explanation:

Synovial chondromatosis results when metaplasia of the subsynovial fibroblasts produces nodules of cartilage in the synovial membrane.

The most common location is the knee. The process may also occur in the hip, shoulder, and elbow.

Numerous small and large round, mineralized loose bodies are present in the plain radiograph. The MRI scan shows a joint effusion (high signal on T2-weighted images) and low signal masses. The T1-weighted images also show the numerous small low-signal masses.

137. A 10-year-old child has a mass over the anterior aspect of the tibia. A plain radiograph is shown in Slide 1 and a biopsy specimen in Slide 2 and Slide 3. Which of the following would be the most appropriate treatment:

Page 101: Hyperguides 2015 .Tumor

Slide 1 Slide 2 Slide 3

(A) Debridement and antibiotic therapy(B) Preoperative chemotherapy followed by resection/reconstruction(C) Chemotherapy and external beam irradiation(D) External beam irradiation alone(E) Observation

Explanation:

The lateral radiograph of the tibia shows a large anterior cortex lucency. Notice that there is expansion of the anterior cortex and a gentle bow to the tibia, which is diagnostic for osteofibrous dysplasia. The biopsy specimen shows spicules of woven bone that are lined by osteoblasts. This is also diagnostic of osteofibrous dysplasia. Patients are generally treated with observation.

138. Lymph node metastases are common in which of the following soft tissue tumors:(A) Extra-abdominal desmoid tumor(B) Nodular fasciitis(C) Malignant fibrous histiocytoma(D) Liposarcoma(E) Epitheloid sarcoma

Explanation:

Lymph node metastases may occur in the following soft tissue sarcomas:

Lymph nodes (these lesions can have regional metastases):o Synovial sarcomao Epithelioid sarcomao Rhabdomyosarcomao Clear cell Sarcoma

Epitheloid sarcoma is prone to metastasize to regional lymph nodes and to the skin and subcutaneous tissue.

Page 102: Hyperguides 2015 .Tumor

139. Soft tissue Ewing's sarcoma has which of the following genetic abnormalities:(A) Giant and marker ring chromosomes(B) Translocation between chromosomes 12 and 16(C) Translocation between 11 and 22(D) Translocation between X and 18(E) Translocation between 9 and 22

Explanation:

Several soft tissue sarcomas have well-recognized genetic abnormalities:

Well-differentiated liposarcoma giant marker and ring chromosomesMyxoid liposarcoma translocation between 12 and 16Ewing's/primitive neuroectodermal tumors translocation between 11 and 22

Synovial sarcoma translocation between X and 18Myxoid chondrosarcoma translocation between 9 and 22

140. Which of the following is the most important prognostic factor in a patient with osteosarcoma:(A) Size(B) Location(C) Presence of metastases(D) Age(E) Grade

Explanation:

The factors that affect prognosis are often reflected in staging systems:

Presence of metastases Grade of the tumor Size of the tumor

Remember that stage trumps all other factors. When stage is not an answer, the prognostic factors are:

Presence of metastases Grade of the tumor Size of the tumor

o For bone tumors in the AJCC system, > 8 cm is worse Site of the tumor

Page 103: Hyperguides 2015 .Tumor

o Spine and pelvic tumors generally have a worse prognosis

Important points to remember:

Osteosarcomao Response to chemotherapy is one of the most important prognostic factors

99% necrosis is best, 90% to 99% is less favorable, <90% is a poor prognostic sign

o Retinoblastoma gene (Rb1) is an associationo Some studies have shown a poorer prognosis with elevated serum alkaline

phosphatase or lactate dehydrogenase (AP, LDH) - this is controversial

141. A 13-year-old boy has a destructive lesion of the distal femur. Magnetic resonance images (MRIs) show a large soft tissue mass, and a needle biopsy shows a Ewing's sarcoma. Which of the following studies should be performed to complete the staging survey:(A) Technetium bone scan and computed tomography (CT) chest(B) Technetium bone scan and CT chest and abdomen(C) Bone marrow biopsy, CT chest, and technetium bone scan(D) Skeletal survey, CT chest and abdomen, and bone marrow biopsy(E) CT chest and abdomen and skeletal survey

Explanation:

Staging of most malignant bone tumors:

Plain radiographs and cross-sectional imaging (MRI and/or CT) Technetium bone scan - look for bone metastases CT chest - look for pulmonary metastases

Note: In Ewing's sarcoma, bone marrow biopsy is performed to detect bone marrow metastases.

142. A 15-year-old boy has a destructive lesion in his proximal tibia. The magnetic resonance image shows soft tissue extension and a computed tomography scan of the chest shows no evidence of pulmonary metastases. The technetium bone scan shows a solitary lesion, and a needle biopsy shows a high-grade osteosarcoma. Which of the following is the correct surgical stage:

(A) Stage IA(B) Stage IB(C) Stage IIA(D) Stage IIB(E) Stage III

Explanation:

Page 104: Hyperguides 2015 .Tumor

Enneking System

IA G1 T1 M0IB G1 T2 M0IIA G2 T1 M0IIB G2 T2 M0IIIA G1-G2 T1-T2 M1IIIB G1-G2 T1-T2 M1Abbreviations: G=grade (G1 low: parosteal osteosarcoma, adamantinomma, chordoma; G2 high: osteosarcoma, Ewing's, malignant fibrous histiocytoma, dedifferentiated chondrosarcoma), T=site (T1: intracompartmental, T2: extracompartmental), M=metastases (M0: no metastases, M1: regional or distant).

143. Which of the following agents inhibits bone destruction:(A) Receptor activator of nuclear-kB ligand (RANKL)(B) Macrophage inflammatory factor-1alpha (MIP-1A)(C) Interleukin-6 (IL-6)(D) Osteoprotegerin (OPG)(E) Parathyroid hormone-related protein (PTHrP)

Explanation:

The bone destruction in multiple myeloma is caused by the release of RANKL from the osteoblast surface. The receptor activator of nuclear-kB ligand binds to a receptor activator of nuclear-kB (RANK) on the osteoclast progenitor cell surface, beginning the process of osteoclast activation and bone destruction. Osteoprotegerin is a decoy inhibitor of the system and halts osteoclast activation.

Factors in multiple myeloma that cause osteoclast activation

IL-6 MIP-1A (acts directly on osteoclast precursors)

Factors that up regulate RANKL and down regulate OPG

PTHrP 1,25 dihydroxy vitamin D IL-1 beta Tumor necrosis factor-alpha Prostaglandin E2

Factors that down regulate RANKL and up regulate OPG

Interleukin-4 Interferon N gamma Transforming growth factor-beta

Page 105: Hyperguides 2015 .Tumor

144. The management of lipoma arborescens includes:(A) Differential diagnosis includes cruciate ligament lesions and patella chondromalatia.(B) Magnetic resonance imaging is not useful in diagnosis.(C) Joint effusion can be obtained via puncture.(D) Lipoma arborescens often occurs in various upper limb joints.(E) Synovectomy is the treatment of choice.

Explanation:

Synovectomy is an effective treatment with excellent results and a low recurrence rate.

Lipoma arborescens should be differentiated from villonodular synovitis and synovial hemangioma.

Magnetic resonance imaging enables diagnosis at an early stage. Effusion cannot be used in cases of lipoma arborescens. The primary location of lipoma arborescens is the knee joint.

145. Which of the following statements regarding pigmented villonodular synovitis (PVNS) is true:(A) Magnetic resonance imaging does not contribute to diagnosis and treatment planning.(B) Blood collection in the knee excludes PVNS.(C) Marginal surgical excision of the giant cell tumor of the tendon sheath is the treatment of

choice. In the knee joint, successful arthroscopic removal of the lesion has also been reported.

(D) The recurrence rate is high.(E) Osmic acid synovectomy is useless.

Explanation:

Localized or focal forms of PVNS can be successfully treated with arthroscopic partial synovectomy. Radiation treatment is generally reserved for high-risk patients when the extent of the lesion makes it inoperable or in patients who have recurrent PVNS after surgery.

Magnetic resonance imaging is especially useful for delineating the extent of the lesion to assist in surgical planning.

Hemorrhage of joints is common. The recurrence rate is low and is dependent on the degree of technical difficulty.

Adequately excised lesions do not recur. Osmic acid or radiation synovectomy is proposed when surgery cannot be performed.

146. Which of the following statements regarding synovial sarcomas is true:(A) Synovial sarcomas are rare malignant soft tissue tumors.(B) Convincing histological evidence shows that synovial sarcomas originate from the cells

of the synovium.(C) A synovial sarcoma is usually located in the vicinity of spine.(D) Synovial sarcomas never cause pain.(E) Synovial sarcoma can be mistaken for synovitis or a ganglion.

Page 106: Hyperguides 2015 .Tumor

Explanation:

A synovial sarcoma may persist for years as a small, superficial, indolent lump in the hands or feet. It can be mistaken for synovitis or a ganglion.

Synovial sarcoma is the third most frequent malignant soft tissue tumor; accounting for 5% to 10% of all soft tissue tumors.

This tumor was designated as a synovial sarcoma because of its anatomical location (most of them originate from the capsules of the joints, bursae, and tendon sheaths) and its histological similarity to normal synovial tissue. No convincing evidence exists that the tumor originates from the cells of the synovium.

A synovial sarcoma is usually located in the extremities (90%) and the lower limbs (60%). Deep, radiating pain is present in about 50% of patients.

147. Which of the following statements is true regarding the diagnosis of synovial sarcoma:(A) Radiograph shows no specific signs.(B) Magnetic resonance imaging exhibits lobular structure and extent.(C) Ultrasound can easily differentiate synovial sarcoma from other benign cysts.(D) Synovial sarcoma does not infiltrate the viscera and muscles.(E) Synovial sarcoma does not have cytogenetic abnormality.

Explanation:

Magnetic resonance imaging is the best tool for detection of the lesion's lobular structure and extent.

The tumor may be recognized on conventional plain radiographic films because tissue calcification occurs in approximately 20% to 40% of patients.

Ultrasound may be helpful in distinguishing the tumor, mainly from Baker cysts in the popliteal region, although in many patients, the synovial sarcoma may contain large multicompartmental cysts filled with mucoid substance similar to synovial fluid.

In the later stages, synovial sarcoma starts to spread into the viscera and muscles, infiltrating the surrounding joints and bones.

The reciprocal translocation t(X; 18) (p 11.2; q 11.2) is thought to be the primary cytogenetic abnormality and specific to synovial sarcoma. By cytogenetic examinations, tumor-specific chimeric transcripts such as SYT-SSX1, SYT-SSX2 can be revealed in the synovial sarcoma, which are also highly specific for the tumor.

148. Which of the following has been found in chondrosarcomas and may indicate the degree of malignancy:(A) RB(B) p53(C) MMPs (matrix metalloproteinases)(D) TIMPs(E) PTHrP

Page 107: Hyperguides 2015 .Tumor

Explanation:

Matrix metalloproteinases (MMPs) may play an important role in the degree of malignancy of chondrosarcomas. These enzymes have the ability to degrade the extra-cellular matrix. Matrix metalloproteinases include collagenases, gelatinases, and stromelysins.

Tissue inhibitors of metalloproteinases (TIMPs) act to hold these derivative enzymes in check to prevent unwanted destruction.

149. Which of the following is the major effect of chemotherapy in the treatment of osteosarcoma:(A) Improves local control(B) Reduces the risk for bone metastases(C) Reduces the risk for brain metastases(D) Reduces the risk for pulmonary metastases(E) Reduces the risk for skin metastases

Explanation:

The major effect of chemotherapy in treating patients with osteosarcoma is reducing the risk of pulmonary metastases. Prior to the advent of multi-agent chemotherapy, the survival of children with osteosarcoma was only 10% to 20% because of pulmonary metastases. With current preoperative and postoperative chemotherapy regimens, the survival has improved to 70%

150. A patient presents with a hard leg mass and pain with activity. The anteroposterior and lateral radiographs are shown in Slide 1 and Slide 2. An axial computed tomography scan is shown in Slide 3. Which of the following tumor suppressor genes is most likely involved:

slide 1 slide 2 slide 3

(A) Retinoblastoma (RB)(B) p53(C) P16INK4a(D) EXT1(E) NF1

Page 108: Hyperguides 2015 .Tumor

Explanation:

This patient has multiple hereditary exostoses. Widening of the metaphysis is characteristic of multiple hereditary exostoses. Large sessile osteochondromas arise from the metaphysis and a large osteochondroma arises from the medial metaphysis with a characteristic cartilaginous cap. The computed tomography scan shows the widening and abnormal tubulation of the bone.

No evidence of malignancy exists in this large osteochondroma. The cartilage cap is regular with no areas of bone destruction. One should also look for a soft tissue mass, which often shows areas of focal calcifications. No soft tissue masses are present in this patient.

It is important to remember that this condition is autosomal dominant. The putative tumor suppressive gene mutation is EXT1, EXT2. The risk of low-grade chondrosarcoma occurring in this condition is approximately 10%

151. A 55-year-old woman has a 15-year history of chronic knee pain. Her plain radiographs are shown in Slide 1. Axial and sagittal T1-weighted magnetic resonance scans are shown in Slide 2. A coronal and sagittal computed tomography reconstruction is shown in Slide 3 and a biopsy in Slide 4. The most likely etiology of this condition is:

slide 1 slide 2

slide 3 slide 4

(A) Systemic inflammatory(B) Reactive(C) Infectious

Page 109: Hyperguides 2015 .Tumor

(D) Benign neoplasm(E) Malignant neoplasm

Explanation:

This patient has pigmented villonodular synovitis (PVNS). Patients often present with a long history of knee pain and recurrent atraumatic effusions.

The plain radiographs show large lucent areas on both sides of the knee joint. The lateral radiograph shows a large posterior erosion. The computed tomography reconstructions show large areas on bone lysis and erosions. Notice the prominent erosions on both sides of the joint. The magnetic resonance image shows large low signal masses and major areas of erosion.The biopsy slide shows the typical histologic features of PVNS. Note that the macrophages have ingested hemosiderin pigment giving them a characteristic granular brown appearance.

Treatment for this patient is synovectomy and total knee replacement. The amount of bone and cartilage destruction is prominent; therefore, synovectomy alone is not sufficient.

This condition is most likely a benign reactive condition rather than a true neoplasm.

152. A 35-year-old man has a 9-month history of a soft tissue mass in his forearm. The mass has been steadily enlarging over time. The Tl and T2 weighted axial magnetic resonance imaging (MRI) scans are shown in Figures 1 and 2. A biopsy was performed and is shown in Figure 3. The most likely diagnosis is:

Slide 1 Slide 2

Page 110: Hyperguides 2015 .Tumor

Slide 3 Slide 4

(A) Malignant fibrous histiocytoma(B) Epitheloid sarcoma(C) Lipoma(D) Desmoid tumor(E) Synovial sarcoma

Explanation:

The forearm mass shown is low signal on TI weighted images and high signal on T2 weighted images. These signal characteristics are non-specific and may represent either a sarcoma or a benign lesion.

The histologic section shows a biphasic lesion with epithelial and spindle cells (Figure 4). This is a typical characteristic of synovial sarcoma.

The histologic appearance of the other lesions is : malignant fibrous histiocytoma — storiform pattern with pleomorphic spindles cells and histiocytesepitheloid sarcoma — nodular growth pattern with central necrosis and epithelial quality of the cellslipoma — mature adipose cellsdesmoid - mature spindle cells with abundant collagen and no cellular atypia

153. Which of the following carcinomas has the highest propensity to metastasize to the hand:(A) Prostate(B) Kidney(C) Lung(D) Thyroid(E) Breast

Explanation:

Most carcinomas metastasize to the spine, ribs, pelvis, skull, and proximal limb girdles. It is uncommon for metastases to occur in the hands or feet. A notable exception is lung carcinomas. Lung carcinomas metastasize to the distal skeleton (acral metastases). Over 50% of patients with

Page 111: Hyperguides 2015 .Tumor

distal (acral metastases) will have a lung carcinoma as the primary tumor.

154. Which of the following histologic descriptions is typical of adamantinoma of long bones:(A) A mixed pattern of inflammatory cells, edema, and new blood vessels(B) Langerhans' cells and eosinophils(C) Spindle cells and immature bone trabeculae(D) Epithelial cells in a fibrous background(E) Uniform population of small blue cells

Explanation:

There are several different histologic patterns for adamantinoma: basaloid, spindle, tubular, squamoid, and osteofibrous-dysplasia like. Each of these patterns shares the basic finding of epithelial cells in a fibrous background.

The other possible answers describe various bone lesions:

Osteomyelitis: A mixed pattern of inflammatory cells, edema, and new blood vessels

Eosinophilic granuloma: Langerhans' cells and eosinophils

Fibrous dysplasia:

Spindle cells and immature bone trabeculae

Ewing's tumor: Uniform population of small blue cells

155. Forty percent of osteoblastomas involve the axial skeleton. Which of the following statements is true:(A) The vertebral body is the most common location.(B) The posterior elements are the most common site.(C) The presence of aneurysmal bone cyst changes within the lesion is rare.(D) Neurologic symptoms are unusual.(E) Patients are generally over 50 years of age.

Explanation:

Osteoblastomas commonly occur in the spine. The lesions involve the posterior elements, may cause neurologic symptoms, and commonly occur in young patients.

The following statements are true concerning osteoblastomas of the spine:

Posterior elements are the most common site. Neurologic symptoms are common. Expansion of the posterior elements is common.

Page 112: Hyperguides 2015 .Tumor

Aneurysmal bone cyst-like changes are commonly seen histologically. The patients are generally young.

156. A 17-year-old man has a 4-month history of severe pain in his right distal tibia. The pain is especially prominent at night. He has difficulty sleeping and concentrating at school. The lateral radiograph of his right distal tibia and clinical photograph is shown in Figure 1. A CT and axial T1-weighted MRI scan are shown in Figures 2 and 3. Which of the following historical findings would help in establishing the diagnosis:

Slide 1 Slide 2 Slide 3

(A) Pain that occurs only with activity(B) Pain that occurs after running 2 to 3 miles(C) Morning stiffness(D) Complete resolution of the pain with aspirin or ibuprofen(E) Pain on the sole of his foot associated with his first step in the morning

Explanation:

Patients with osteoid osteomas have a characteristic pain pattern. Often, osteoid osteoma pain is completely relieved by aspirin or nonsteroidal anti-inflammatory drugs (NSAIDs). Children may finish an entire bottle of ibuprofen in just 1 week in order to avoid the pain.

The other possible answers describe common symptoms of other musculoskeletal conditions:

Stress fractures: Pain only with activity

Exertional compartment syndrome:

Morning stiffness; inflammatory arthritis

Ankylosing spondylitis or rheumatoid arthritis:

Pain that occurs after running 2 to 3 miles

Planter fascitis: Pain with the first step in the morning on the sole of the foot

Page 113: Hyperguides 2015 .Tumor

157. Which of the following tumors has been associated with a chromosomal translocation from chromosome 9 to 22:(A) Malignant fibrous histiocytoma(B) Fibrosarcoma(C) Parosteal osteosarcoma(D) Extra-skeletal myxoid chondrosarcoma(E) Periosteal osteosarcoma

Explanation:

Recently, 2 cartilage tumors have been associated with genetic abnormalities:Extra-skeletal myxoid chondrosarcoma:Translocation from chromosome 9 to 22Multiple exostoses:Exostosis genes:EXT 1 chromosome 8 (8q24)EXT 2 chromosome 11 (11p13)EXT 3 chromosome 19 (19q)

158. Which of the following cartilage tumors has the worst prognosis in regard to survival:(A) Grade II chondrosarcoma of the proximal femur(B) Grade I chondrosarcoma of the pelvis(C) Grade 2 chondrosarcoma of the distal femur(D) Dedifferentiated chondrosarcoma of the distal femur(E) Grade I chondrosarcoma arising in a pelvic osteochondroma

Explanation:

In cartilage tumors, the histologic grade of the lesion is the most important determinate of long-term disease-free survival. Grade I chondrosarcomas seldom, if ever, metastasize. In contrast, dedifferentiated chondrosarcomas metastasize in 75% to 90% of cases.

A study of 67 pelvic chondrosarcomas showed the following rate of metastases:

Approximate Rate of Metastases

Grade I 0%

Grade II 20%

Grade III 60%

Dedifferentiated 75%159. In patients with localized high-grade intramedullary osteosarcoma (Stage II), what is the

most important factor in regard to long-term disease-free survival:(A) Sex of the patient(B) Age of the patient(C) Size of the initial tumor(D) Response to preoperative chemotherapy(E) Presence or absence of soft tissue extension

Page 114: Hyperguides 2015 .Tumor

Explanation:

The most important determinant of long-term disease-free survival is response to preoperative chemotherapy. Multiple histologic sections are analyzed to determine the amount of tumor necrosis. The most favorable survival occurs in those patients with 99% or 100% tumor kill. When less than 90% tumor necrosis is achieved, the response is judged to be poor and there is a significant risk for both local recurrence and systemic failure.

160. In which of the following conditions is polyostotic involvement common:(A) Giant cell tumor(B) Chondroblastoma(C) Chondromyxoid fibroma(D) Fibrous dysplasia(E) Desmoplastic fibroma

Explanation:

Polyostotic involvement is common in a number of bone tumors. A useful differential diagnosis of multiple lesions is as follows in both young and old patients:

Benign Malignant

Younger Patients(age 10 to 40 years)

Eosinophilic granuloma (Langerhans cell granulomatosis)Fibrous dysplasiaMultiple exostosesMultiple enchondromatosis

Epithelioid hemangioendotheliomaMetastases:Neuroblastoma/Wilm's tumor (young children < 10 years old) Melanoma

Older Patients(age 40 to 80+ years)

Paget's diseaseFibrous dysplasiaBone infarctsHyperparathyroidismMastocytosis

Metastatic carcinomasMultiple myelomaLymphomaEpithelioid hemangioendothelioma

161. In which of the following metastatic carcinomas is preoperative embolization useful to decrease bleeding in large lesions with soft tissue extension:(A) Adenocarcinoma of the lung(B) Breast cancer(C) Renal cell carcinoma(D) Prostate cancer(E) Small cell cancer of the lung

Explanation:

Renal cell carcinomas can be very vascular lesions. There may be thousands of small and large blood vessels supplying the lesion. Lesion with marked expansion of the bone or large soft tissue

Page 115: Hyperguides 2015 .Tumor

masses are especially at risk for massive bleeding.

Preoperative embolization is an excellent method of reducing intraoperative bleeding. Patients are usually embolized once within 1 week of surgery. Occasionally, a very vascular lesion may require more than one separate angiographic procedures to complete the embolization.

162. A 15-year-old boy presents with a 9-month history of severe thigh pain. The pain is most severe at night and aspirin completely relieves the pain. The plain radiograph is shown in Figure 1. The imaging study of choice to establish the diagnosis for this patient would be:

Figure 1

(A) Magnetic resonance imaging (MRI) scan(B) Technetium bone scan(C) Single proton emission computerized tomography (SPECT) scan(D) Computerized tomography (CT) scan(E) High-resolution ultrasound

Explanation:

The history of severe night pain and pain responsive to nonsteroidal anti-inflammatory medications suggests that the patient has an osteoid osteoma. Osteoid osteomas are small tumors, with the nidus being the essential component of the lesion. The nidus is small (5 mm to 10 mm) and is often surrounded by heavy periosteal new bone. The center of the nidus is often mineralized. CT scans are the best method to identify the lytic focus embedded in the cortical bone.

A MRI scan is a superb method to identify bone and soft tissue neoplasms. However, CT scanning is the best method to identify osteoid osteomas. MRI is less sensitive and specific than CT because the nidus may be mineralized in the center causing it to blend with and not be easily distinguished from the periosteal bone.

Technetium bone scanning is a sensitive technique used to detect increased activity in an osteoid osteoma, but when one recognizes the increased activity, the increase is non-specific. The increased activity may be secondary to a stress fracture, osteomyelitis, benign or malignant neoplasm.

Page 116: Hyperguides 2015 .Tumor

163. An 18-year-old male presented with a 6-month history of hip pain. His pain occurred both at rest and at night. His parents noticed a significant limp, and he felt his leg was much weaker. The plain radiograph and computerized tomography (CT) scan of the hip are shown (Slide 1 and Slide 2). The low- and high-power hematoxylin and eosin histologic sections are also presented (Slide 3 and Slide 4).The most likely diagnosis is:

Slide 1 Slide 2

Slide 3 Slide 4

(A) Low-grade osteosarcoma(B) High-grade intramedullary osteosarcoma(C) Parosteal osteosarcoma(D) Osteoblastoma(E) Osteoid osteoma

Explanation:

Osteoid osteoma is a self-limited benign bone tumor with a small nidus (usually less than 1.5 cm) surrounded by reactive bone formation. Histologically, the lesion is well demarcated.The CT scan shows the lucent nidus is surrounded by scelentic bone.

164. A 30-year-old woman has a painless mass in the back of her knee. She has felt the mass for over six months. She has restricted range of motion in the knee and is now having difficulty

Page 117: Hyperguides 2015 .Tumor

when she walks. The plain radiographs are shown (Slide 1 and Slide 2). The low- and high-power histologic hematoxylin/eosin sections are also presented (Slide 3, Slide 4 and Slide 5).Based upon the history, physical examination, plain radiographs, and biopsy the most likely diagnosis is:(A) Heterotopic ossification(B) Osteochondroma(C) High-grade intramedullary osteosarcoma(D) Parosteal osteosarcoma(E) Periosteal osteosarcoma

Explanation:

The plain radiographs show a heavily mineralized lesion on the surface of the bone. The mass encircles the bone. Note the prominent bone formation. The low-power histologic section shows prominent bone formation and fibrous components. The high-power section shows a spindle cell tumor with prominent fibrous tissue production. The spindle cells are uniform in appearance . The nuclei lack pleomorphism. This is a parosteal osteosarcoma.165. A 30-year-old woman has a painless mass in the back of her knee. She has felt the mass for

over six months. She has restricted range of motion in the knee and is now having difficulty when she walks. The plain radiographs are shown (Slide 1 and Slide 2). The low- and high-power histologic hematoxylin/eosin sections are also presented (Slide 3, Slide 4 and Slide 5).Based upon the history, physical examination, plain radiographs, and biopsy the most likely diagnosis is:

Slide 1 Slide 2 Slide 3

Page 118: Hyperguides 2015 .Tumor

Slide 4 Slide 5

(A) Heterotopic ossification(B) Osteochondroma(C) High-grade intramedullary osteosarcoma(D) Parosteal osteosarcoma(E) Periosteal osteosarcoma

Explanation:

The plain radiographs show a heavily mineralized lesion on the surface of the bone. The mass encircles the bone. Note the prominent bone formation. The low-power histologic section shows prominent bone formation and fibrous components. The high-power section shows a spindle cell tumor with prominent fibrous tissue production. The spindle cells are uniform in appearance . The nuclei lack pleomorphism. This is a parosteal osteosarcoma.

166. A 29-year-old woman has a 5-month history of dull, aching pain in her shoulder after falling from her bicycle. Her radiograph (Slide 1) and biopsy specimen (Slide 2) are presented. Which of the following is the most appropriate treatment regimen:

Slide 1 Slide 2

Page 119: Hyperguides 2015 .Tumor

(A) Preoperative multiagent chemotherapy followed by wide resection(B) Internal fixation and external beam irradiation(C) Wide resection followed by external beam irradiation(D) Marginal resection followed by joint reconstruction(E) Forequarter amputation

Explanation:

The plain radiographs show a destructive lytic lesion. The differential diagnosis, based on the patient’s age and the pattern of bone destruction, is:

Giant cell tumor Aneurysml bone cyst Telangiectatic osteosarcoma Ewing tumor

The histology clearly shows a giant cell tumor with uniformly distributed giant cells in a field of mononuclear cells. The diagnosis is giant cell tumor.

The treatment of giant cell tumor is curettage with methylmethacrylate augmentation for lesions in which the joint surfaces can be saved. If the joint surfaces cannot be saved, marginal resection and joint reconstruction must then be performed. Joint reconstruction in this case may be either with an osteoarticular allograft, allograft prosthetic composite, or a metal prosthesis.

167. A 29-year-old woman has a 5-month history of dull, aching pain in her shoulder after falling from her bicycle. Her radiograph (Slide 1) and biopsy specimen (Slide 2) are presented. The stage of this lesion according to the Musculoskletal Tumor Society would be:

Slide 1 Slide 2

(A) Stage 1(B) Stage 2(C) Stage 3(D) Stage I(E) Stage II

Explanation:

Page 120: Hyperguides 2015 .Tumor

The plain radiographs show a destructive lytic lesion. The differential diagnosis, based on the patient’s age and the pattern of bone destruction, is:

Giant cell tumor Aneurysml bone cyst Telangiectatic osteosarcoma Ewing tumor

The histology clearly shows a giant cell tumor with uniformly distributed giant cells in a field of mononuclear cells. The diagnosis is giant cell tumor.

This tumor is aggressive and has destroyed the entire metaphysis, resulting in a fracture. The epiphysis is also completely destroyed. This is an aggressive lesion and would be classified as a Stage 3 aggressive lesion.

Remember that Arabic numerals are for benign lesions and Roman numerals are for malignant lesions.

Remember:

Stage 1 Inactive/latent Stage 2 Active Stage 3 Aggressive

Stage I Low grade Stage II High grade Stage III Metastatic disease

168. A 70-year-old man has severe shoulder pain. A plain radiograph (Slide 1) and a biopsy specimen (Slide 2) are presented. The most appropriate treatment for the humeral lesion is:

Slide 1 Slide 2

Page 121: Hyperguides 2015 .Tumor

(A) Wide resection and allograft prosthetic reconstruction(B) Diphosphonate therapy(C) Intramedullary rod fixation and external beam irradiation(D) Subacromial cortisone injection(E) External beam irradiation

Explanation:

The plain radiograph of the humerus shows a densely sclerotic lesion. The cortices are intact and show no thickening or remodeling features that are characteristic of Paget’s disease. The lesion has no apparent bone destruction. The biopsy specimen shows epithelial cells formed in clusters (or an organoid pattern). This is metastatic carcinoma.

The goals of treatment in metastatic disease are to relieve pain, prevent fracture, and maintain function. Purely blastic lesions have a low risk of fracture. This patient does not need prophylactic fixation. The best treatment choice is external beam irradiation alone.

169. A 60-year-old man has a 6-month history of increasing knee pain. He has discomfort at night and at rest. His anteroposterior and lateral radiographs are shown in Slide 1, and low and high power biopsy specimens are shown in Slides 2 and 3. The most likely diagnosis is:

Slide 1 Slide 2 Slide 3

(A) Metastatic bone disease(B) Multiple myeloma(C) Malignant fibrous histiocytoma(D) Lymphoma(E) Dedifferentiated chondrosarcoma

Explanation:

The plain radiographs show two major features. The proximal portion of the lesion (top half) shows an intramedullary lesion, which is mineralized. The mineralization shows rings and stipples. There is also thickening of the cortex of the femur. This thickening is circumferential on the medial, lateral, anterior, and posterior cortices. These features strongly suggest a cartilage tumor, principally a chondrosarcoma. The distal (lower half) of the lesion has a different pattern. The intramedullary lesion is completely lytic, and the lateral cortex has been destroyed. This bimorphic pattern is seen with dedifferentiated chondrosarcoma. The bimorphic pattern is a low-

Page 122: Hyperguides 2015 .Tumor

grade chondrosarcoma and a high-grade anaplastic spindle cell sarcoma.

The histologic appearance is also bimorphic. Notice the cartilage, which is relatively hypocellular, and the intimate admixture of a high-grade spindle cell sarcoma. The high-grade spindle sarcoma is characterized by pleomorphic and pyknotic nuclei–dark staining nuclear chromatin, and different sizes and shapes of the nuclei. The bimorphic pattern is low-grade cartilage admixed with high-grade spindle cell sarcoma. This high-grade spindle cell sarcoma can be osteosarcoma, malignant fibrous histiocytoma, or fibrosarcoma.

This tumor is a high-grade malignancy with a long-term survival of only 20% to 40%. The lungs are the most common site of metastases. Treatment is wide resection.

170. A 55-year-old man has a 4-month history of increasing knee pain. The plain radiographs are shown in Slide 1 and a coronal T1 weighted magnetic resonance scan is shown in Slide 2. A biopsy of the lesion is shown in Slides 3 and 4. Which of the following studies should be done to aid in staging of the patient’s disease:

Slide 1 Slide 2

Slide 3 Slide 4

(A) Skeletal survey(B) Serum protein electrophoresis

Page 123: Hyperguides 2015 .Tumor

(C) Bone marrow aspiration/biopsy(D) Computed tomography (CT) scan of the chest(E) CT scan of the abdomen

Explanation:

The plain radiograph shows a destructive lesion in the distal femoral metaphysis. Notice the lesion is purely lytic. The magnetic resonance imaging scan shows a very large lesion with destruction of the cortex and a large soft tissue mass. This radiograph appearance indicates a malignant lesion. The possibilities include a chondrosarcoma, malignant fibrous histiocytoma, metastasis, and lymphoma. The biopsy specimen shows a storiform pattern and fascicle of spindle cells. The high power specimen shows spindle cells with pleomorphism, which are different nuclear sizes and shapes, and histiocytes, which are cells with round and vesicular nuclei. The diagnosis is malignant fibrous histiocytoma. The most common site of metastases would be to the lungs. The best staging test would be a CT scan of the chest. Skeletal survey, serum protein electrophoresis, and bone marrow aspiration would be used for staging in multiple myeloma. Computed tomography of the chest and abdomen is used for staging in metastatic bone disease.

171. A 65-year-old woman has a 5-month history of increasing hip and thigh pain. Her plain radiographs are shown in Slide 1. T1 and T2 weighted coronal images of her hip are shown in Slides 2 and 3 respectively. A biopsy is performed and is shown in Slide 4. The most likely diagnosis is:

Slide 1 Slide 2

Slide 3 Slide 4

Page 124: Hyperguides 2015 .Tumor

(A) Multiple myeloma(B) Lymphoma(C) Metastatic adenocarcinoma(D) Chondrosarcoma(E) Paget disease

Explanation:

The plain radiographs show an ill-defined permeative lesion in the proximal femur. The lateral cortex of the greater trochanter is thinned. The T1 weighted coronal images show a low signal lesion in the entire greater trochanter extending to the base of the femoral neck. Notice in the opposite proximal femur and acetabulum, there are multiple small low signal lesions consistent with metastatic bone disease. The T2 weighted images show a very high signal area in the greater trochanter. The inversion recovery image is very sensitive to pathologic processes. Normal marrow will be very low signal while pathologic lesions are high signal. The biopsy shows round epithelial cells forming glands. This pattern is that of metastatic adenocarcinoma.

172. A 50-year-old woman has had severe hip pain for 4 months. Her plain radiographs are shown in Slide 1. The technetium bone scan, computerized tomography scan, and coronal T1 and T2 weighted magnetic resonance imaging scans are shown in Slides 2, 3, and 4 respectively. The most likely diagnosis based upon the radiographs would be:

Slide 1 Slide 2 Slide 3

Slide 4

(A) Septic arthritis(B) Osteomyelitis of the acetabulum(C) Stress fracture

Page 125: Hyperguides 2015 .Tumor

(D) Lytic phase of Paget disease(E) Malignant neoplasm

Explanation:

The plain radiographs show a purely lytic destructive lesion, which is poorly marginated. The technetium bone scan does not show any major uptake. The computerized tomography scan shows purely lytic bone destruction with breakthrough of the cortical bone. Complete destruction of the cortical bone is very suggestive of a malignancy. The magnetic resonance imaging scan shows a lesion that is homogenously low on T1 weighted images and high on T2 weighted images. One cannot make a definitive diagnosis based upon the radiographic features; however, it is highly likely that this lesion is malignant. The cortical bone destruction is highly suggestive of a malignancy. The most common malignancies in this age group are:

Metastatic bone disease Multiple myeloma Lymphoma Chondrosarcoma Malignant fibrous histiocytoma

173. A 15-year-old girl has a 4-month history of increasing wrist pain. Her clinical photograph and plain radiograph are shown in Slides 1 and 2. An axial T1-weighted image and sagittal T2-weighted image are shown in Slides 3 and 4. A biopsy specimen is shown in Slide 5. Which of the following is the correct risk of pulmonary metastases for this lesion:

Slide 1 Slide 2

Slide 3 Slide 4 Slide 5

(A) 0%(B) 2% to 5%(C) 10% to 30%(D) 30% to 50%(E) 75%

Page 126: Hyperguides 2015 .Tumor

Explanation:

The plain radiograph shows a destructive lesion. The lesion is purely lytic, expansile, and there is a pathologic fracture. The lesion has destroyed the metaphysis and extended into the epiphysis.

From a radiographic perspective, this lesion could be:

A telangiectatic osteosarcoma An aneurysmal bone cyst A giant cell tumor

The biopsy specimen shows a uniform distribution of multinucleated giant cells in a field of mononuclear cells. The diagnosis is giant cell tumor of bone.

The risk of pulmonary metastases in giant cell tumor of bone is 2% to 5%. This phenomenon is called benign metastazing giant cell tumor. The mortality rate from these metastases is 10% to 20%. Spontaneous regression of the pulmonary lesions may occur.

174. A 45-year-old woman has hip discomfort with prolonged sitting and feels a mass. The plain radiographs, and computerized tomography scan are shown in Slides 1, 2, and 3. Her symptoms have increased and she desires surgical treatment. She undergoes surgical removal and her biopsy specimens are shown in Slides 4 and 5. The diagnosis is:

Slide 1 Slide 2 Slide 3

Slide 4 Slide 5

Page 127: Hyperguides 2015 .Tumor

(A) Mesenchymal chondrosarcoma(B) Dedifferentiated chondrosarcoma(C) Low grade chondrosarcoma(D) Parosteal osteosarcoma(E) Osteochondroma

Explanation:

The anteroposterior radiograph shows a lesion in the greater trochanter, and the oblique radiograph shows a bony mass projecting from the proximal femur. The computerized tomography scan shows a lesion arising from the posterior cortical surface, which shares its cortices with the host bone. In addition, the medullary cavity of the proximal femur flows into the lesion. These findings are consistent with an osteochondroma. There is no evidence of malignant change.

The radiographic findings of an osteochondroma are:

Underlying bone and the osteochondroma have continuous cortices. Medullary cavity of the bone and osteochondroma are continuous.

Secondary chondrosarcomas may develop in an osteochondroma. The radiographic features are:

Destruction of the underlying subchondral cortex of the osteochondroma Lytic destruction of the cartilage cap Formation of a soft tissue mass (often calcified)

The biopsy specimen shows a lesion with a thin cartilage cap. The high power view shows the typical features of an osteochondroma with a linear arrangement of the chondrocytes.The lesion is benign. Observation is all that is required.

175. Which of the following soft tissue lesions has a characteristic reciprocal transformation between chromosomes 12 and 16:(A) Well-differentiated lipoma-like liposarcoma(B) Atypical lipoma(C) Intramuscular lipomas(D) Myxoid liposarcoma(E) Pleomorphic liposarcoma

Explanation:

Liposarcomas account for approximately 10% to 15% of sarcomas. Some general statements about liposarcomas are listed below.

Liposarcomas may have chromosomal abnormalities:

Page 128: Hyperguides 2015 .Tumor

1. Reciprocal transformation between chromosomes 12 and 16 occurs in myxoid and round cell liposarcoma.

2. Well-differentiated liposarcomas have giant marker and ring chromosome abnormalities with amplification of MDM2, SAS, and GLI genes.

3. Liposarcomas express peroxisome proliferator-activated receptor gamma (PPAR gamma).

176. Which of the following conditions has been associated with nonossifying fibromas in a small number of patients:(A) Primary hyperparathyroidism(B) Polycystic kidneys(C) Ventricular septal defect(D) Hypophosphatemic vitamin D refractory rickets and osteomalacia(E) Clubfoot and vertical talus

Explanation:

Nonossifying fibroma is a common benign bone tumor that may rarely be associated with hypophosphatemic vitamin D refractory rickets and osteomalacia. Once the nonossifying fibroma is removed, the rickets resolve.

177. Which of the following describes the probable inheritance pattern of Paget’s disease:(A) Spontaneous mutation in all patients(B) Autosomal recessive(C) Autosomal dominant(D) X-linked recessive(E) X-linked dominant

Explanation:

The inheritance pattern of Paget’s disease is most likely autosomal dominant in pattern. A first degree relative has a seven times greater chance of developing Paget’s disease than the general population. The exact inheritance is unknown although the human leucocyte antigen (HLA) locus and chromosome 18q (10,11) have been implicated.

178. A 38-year-old man has a 4-month history of increasing right hip pain. The plain radiographs are shown in Slide 1 and a biopsy specimen in Slide 2. The most likely cause of his hip pain is:

Page 129: Hyperguides 2015 .Tumor

Slide 1 Slide 2

(A) A post-traumatic condition(B) An infectious process(C) A benign neoplasm(D) A malignant neoplasm(E) A developmental condition

Explanation:

This patient has a large lesion arising from the surface of the bone. There is matrix mineralization of the lesion that shows rings and stipples. This matrix pattern is consistent with a cartilage lesion. The biopsy specimen shows the blue matrix of cartilage and abundant clumps of chondrocytes. This patient has a surface chondrosarcoma. Juxtacortical chondrosarcoma and periosteal chondrosarcoma are names that can be used interchangeably.

This patient has a malignant bone condition. This particular radiographic appearance can be misinterpreted as a benign condition:

Posttraumatic condition: Heterotopic ossification Infectious process: Osteomyelitis Benign neoplasm: Osteochondroma

179. The treatment of tumoral calcinosis is:(A) Excision with an intralesional or marginal margin(B) Wide excision with a cuff of normal muscle(C) Wide excision and external beam irradiation(D) Preoperative chemotherapy and wide excision(E) Preoperative external beam irradiation followed with wide excision

Explanation:

Page 130: Hyperguides 2015 .Tumor

Tumoral calcinosis is a heritable condition that is characterized by periarticular metastatic calcification. Most patients are black, and the inheritance is usually autosomal recessive. Metastatic calcifications occur around joints and in the skin, marrow, teeth, and blood vessels. The periarticular masses may grow quite large and are attached to the fascia, but they are extra-articular. The masses may occur at the shoulder, hip, and elbow.

Radiographically: The masses are composed of heavy, amorphous calcification in nodules. Laboratory:

1. Serum calcium normal2. Serum alkaline phosphatase normal3. Urine calcium is low4. Serum phosphate is often high

When symptomatic, the masses are excised. This lesion is benign and removal of normal tissue and radiation therapy are not necessary.

180. Which of the following cytogenetic findings is common in synovial sarcoma:(A) Balanced reciprocal translocation (t[X;18][p11.2;q11.2])(B) Ring and giant chromosomes(C) Reciprocal transformation between chromosome 12 and 16(D) Translocation between chromosomes 9 and 22(E) Translocation between 11 and 22

Explanation:

Approximately 90% of synovial sarcomas have a balanced reciprocal translocation (t[X;18][p11.2;q11.2]) that occurs between chromosomes X and 18.

Many benign and malignant soft tissue tumors have specific chromosomal abnormalities:

Well-differentiated liposarcoma Myxoid liposarcoma

Giant marker and ring chromosomesTranslocation between chromosomes 12 and 16

Ewings/primitive neuroectodermal tumors Translocation between chromosome 11 and 22 Synovial sarcoma Translocation between chromosomes X and 18 Myxoid chondrosarcoma Translocation between chromosomes 9 and 22

181. A 12-year-old boy has a 2-week history of increasing knee pain. He is active in cross-country running. Anteroposterior and oblique radiographs are shown in Slides 1 and 2. The most likely cause of the knee pain is:

Page 131: Hyperguides 2015 .Tumor

Slide 1 Slide 2

(A) Tumor destruction of the bone(B) Pressure within the medullary cavity(C) Stress fracture(D) Physeal separation(E) Growing pains

Explanation:

There is a stress fracture present on the lateral superior border of the lesion. Notice the condensation of bone in this area. This lesion is a nonossifying fibroma and has a characteristic appearance which includes the following radiographic features:

Nonossifying fibromas have the following radiographic features:

Eccentric Metaphyseal Sclerotic rim Overlying cortex is thinned Lesion is based on the cortex

182. A 60-year-old woman has had severe pain in her leg for the past 8 months. The plain radiograph is shown in Slide 1, and a biopsy of the tibia in Slide 2. The most appropriate treatment is:

Page 132: Hyperguides 2015 .Tumor

Slide 1 Slide 2

(A) Curettage and bone grafting(B) Resection and prosthetic reconstruction(C) External beam irradiation and chemotherapy(D) Diphosphonate therapy(E) Above the knee amputation

Explanation:

The plain radiograph shows a lesion with multiple areas of bone lysis and areas of sclerosis. There is also thickening of the anterior and posterior cortices. The biopsy specimen shows large and small blue cells. Notice that the cells do not match the description of metastatic bone disease, myeloma, malignant fibrous histiocytoma, or chondrosarcoma. This patient has a lymphoma.

Metastatic bone disease - epithelial cells forming glands in a fibrous background Multiple myeloma - sheets of plasma cells Malignant fibrous histiocytoma - storiform pattern of spindle cells and histiocytes Chondrosarcoma - cartilage forming tumor with atypical chondrocytes

Lymphoma

Uniform cells with large round nucleus and clumped chromatin Scant cytoplasm

183. Which of the following genetic abnormalities may occur in polyostotic fibrous dysplasia:(A) Chromosomal translocation, 11-22(B) Activating mutation, GNAS 1 gene(C) EXT1, EXT2 genes(D) t(X;18)(p11.2;q11.2)

Page 133: Hyperguides 2015 .Tumor

(E) t(12;16) translocation

Explanation:

In polyostotic fibrous dysplasia, an activating mutation of the GNAS 1 gene has been found. There may also be over expression of the C-fos proto-oncogene.The other responses refer to specific conditions:

Ewing’s tumor: chromosomal translocation, 11-22 Multiple exostoses: EXT1, EXT2 genes Synovial sarcoma: t(X;18)(p11.2;q11.2) Myxoid liposarcoma: t(12;16) translocation

184. Which of the following describes the radiographic features of nonossifying fibroma of bone:(A) Metaphyseal, cortical bone destruction, ivory-like bone formation(B) Metaphyseal-epiphyseal, lytic, poorly marginalized(C) Metaphyseal, eccentric, thinned cortex, sclerotic rim(D) Epiphyseal, geographic lytic area, sclerotic rim(E) Diaphyseal, multiple lucent defects with intervening sclerosis

Explanation:

The characteristic radiographic appearance of nonossifying fibroma is:

Metaphyseal Cortically based Lucent area Sclerotic rim Overlying cortex is thinned but intact

The other responses are for common bone lesions:

Osteosarcoma: metaphyseal, cortical bone destruction, ivory like bone formation Giant cell tumor: metaphyseal-epiphyseal, lytic, poorly marginalized Chondroblastoma: epiphyseal, geographic lytic area, sclerotic rim Adamantinoma: multiple lucent defects with intervening sclerosis

185. A 60-year-old woman has a 15-cm soft tissue mass on her posterior thigh. The mass is low signal on T1-weighted images and high signal on T2-weighted images and is located below the fascia. A needle biopsy is shown (Slide). The most likely diagnosis is:

Page 134: Hyperguides 2015 .Tumor

Slide 1

(A) Lipoma(B) Atypical lipoma(C) Liposarcoma(D) Schwannoma(E) Desmoid tumor

Explanation:

A soft tissue mass that is located below the fascia should raise the suspicion of a sarcoma. All sarcomas have a common imaging pattern - low signal on T1-weighted sequences and high signal on T2-weighted sequences.The needle biopsy shows a cellular lesion that has marked pleomorphism of the cells (different sizes and shapes of the nuclei). Some cells have intracytoplasmic fat globules; these cells are lipoblasts. The presence of the lipoblasts and the pleomorphic spindles cells is compatible with a liposarcoma. For examination purposes, one would not be able to distinguish this mass from a malignant fibrous histiocytoma.

186. A 65-year-old woman has had severe arm and shoulder pain for 4 months. Her plain radiographs are shown in Slide 1 and Slide 2. Sections from a needle biopsy are shown in Slide 3 and Slide 4. The final common denominator for the bone destruction in this patient is:

Slide 1 Slide 2 Slide 3 Slide 4

(A) Parathyroid-related hormone (PTHrP)(B) Receptor activator of nuclear activator ligand (RANKL)(C) Osteoprotegerin (OPG)(D) Transforming growth factor beta (TGFB)(E) Insulin-like growth factor 1 (IGF1)

Page 135: Hyperguides 2015 .Tumor

Explanation:

The plain radiographs show a destructive lesion with a permeative pattern of bone destruction (multiple small lytic areas) and areas of bone formation - a mixed pattern of bone destruction and bone formation. The differential diagnosis based upon the radiographs alone in this age group (ages 40 to 80 years) includes:

Metastatic bone disease Multiple myeloma Lymphoma Primary mesenchymal bone tumors:

   Chondrosarcoma   Malignant fibrous histiocytoma

The biopsy specimen shows epithelial cells clumped together in an organoid pattern. This patient has metastatic breast carcinoma.

The breast carcinoma cells release PTHrP, which stimulates the osteoblasts to release RANKL, which activates the osteoclast progenitor cells. Remember that RANKL is the final common denominator for osteoclast activation. These cells then become osteoclasts and resorb bone. With the bone resorption, there is release of TGFB and IGF1 that stimulate the tumors cells to release more PTHrP, causing more bone destruction.

Remember that RANKL binds to the RANK receptor on the osteoclast progenitor cell and that OPG is a competitive decoy inhibitor to RANKL.

187. A 55-year-old man has a large soft tissue mass in his arm. The T1- and T2-weighted images are shown in Slide 1 and Slide 2. The abnormal signal in the deltoid muscle is most likely:

Slide 1 Slide 2

(A) Simple fluid(B) Complex fluid(C) Malignant tumor cells(D) Fat(E) Fibrous tissue

Explanation:

The T1-weighted image shows a large high-signal mass in the patient's arm. The signal quality

Page 136: Hyperguides 2015 .Tumor

exactly matches the subcutaneous fat. Note on the T2-weighted image with fat suppression that the mass completely suppresses (or turns black or low signal).This process is benign fat cells.This patient has a lipoma or a benign soft tissue neoplasm. The patient can be treated with either observation or removal of the lipoma. The lipoma can be removed with a marginal or intralesional margin.

188. Chondromyxoid fibroma:(A) Frequently dedifferentiates into a malignant tumor if untreated(B) Rarely involves the physis(C) May cross the physis to involve the epiphysis(D) Has a microscopic picture in which the chondral element predominates(E) Often presents by fracture

Explanation:

Chondromyxoid fibroma has once been reported to dedifferentiate to chondrosarcoma. The tumor commonly abuts the physis, and is microscopically predominated by the myxoid element. Chondromyxoid fibroma rarely presents with a fracture.

189. A 24-year-old man has a 1-year history of pain in his tibia. Plain radiographs show a lesion in the tibia with multiple lucent regions and intervening sclerosis. A computed tomography scan of the chest is negative, and a technetium bone scan shows a solitary lesion. The magnetic resonance image does not show any soft tissue extension, and a needle biopsy is consistent with an adamantinoma. Which of the following is the correct surgical stage:(A) Stage IA(B) Stage IB(C) Stage IIA(D) Stage IIB(E) Stage IIII

Explanation:

To answer this question, one must know that all adamantinomas are low-grade lesions. The suffix A is used when the tumor is intracompartmental (no soft tissue extension)

Enneking System

IA G1 T1 M0IB G1 T2 M0IIA G2 T1 M0IIB G2 T2 M0IIIA G1-G2 T1-T2 M1IIIB G1-G2 T1-T2 M1Abbreviations: G=grade (G1 low: parosteal osteosarcoma, adamantinomma, chordoma; G2 high: osteosarcoma, Ewing's, malignant fibrous histiocytoma, dedifferentiated chondrosarcoma), T=site

Page 137: Hyperguides 2015 .Tumor

(T1: intracompartmental, T2: extracompartmental), M=metastases (M0: no metastases, M1: regional or distant).

190. A 27-year-old man has a 1-year history of knee pain. A needle biopsy of the tibia shows an adamantinoma. Which of the following treatment options is most appropriate:(A) Wide resection alone(B) Wide resection and low-dose chemotherapy(C) Wide resection and high-dose chemotherapy(D) Wide resection and external beam irradiation(E) Curettage and bone grafting

Explanation:

Adamantinomas are low-grade malignancies. Adamantinomas have the potential to metastasize to the lungs, lymph nodes, and other bones. The risk of metastasis is low.

Adamantinomas are treated with wide surgical resection alone. Chemotherapy and external beam irradiation are not useful for patients with adamantinoma.

191. Which of the following malignancies other than retinoblastoma most commonly affects patients with a mutation in the retinoblastoma (RB) gene:(A) Chondrosarcoma(B) Malignant fibrous histiocytoma(C) Neurofibrosarcoma(D) Osteosarcoma(E) Breast cancer

Explanation:

Patients who have mutation in the retinoblastoma gene are prone to develop retinoblastoma. Osteosarcoma is the second most common malignancy to develop in these patients.

One should remember the major tumor suppressor genes in musculoskeletal conditions:

Retinoblastoma (RB) gene — retinoblastoma, osteosarcoma p53 — osteosarcoma, high-grade chondrosarcoma, soft tissue sarcomas P16INK4a — familial melanoma — chondrosarcoma, osteosarcoma, melanoma APC — familial adenomatous polyposis — colon adenomas, desmoid tumors NF1 — neurofibromatosis 1 — neurofibroma, sarcoma

Page 138: Hyperguides 2015 .Tumor

EXT1, EXT2 — hereditary multiple exostosis — osteochondromas, chondrosarcoma

192. Chemotherapy has no role in the treatment of which of the following tumors:(A) High-grade surface osteosarcoma(B) High-grade intramedullary osteosarcoma(C) Adamantinoma(D) Periosteal osteosarcoma(E) Ewing’s tumor

Explanation:

Chemotherapy has dramatically improved the survival rate of patients with high-grade intramedullary osteosarcoma and Ewing’s tumor. The disease-free survival rate without chemotherapy is less than 20% for patients, whereas with modern multi-agent regimens, the disease-free survival rate approaches 70%. Periosteal osteosarcoma is an intermediate-grade osteosarcoma (risk of pulmonary metastases is 15% to 25%), which is generally treated with the same chemotherapy regimen as high-grade intramedullary osteosarcoma.

Chemotherapy has been used for a number of high-grade sarcomas with disappointing results:

Dedifferentiated chondrosarcoma High-grade intramedullary chondrosarcoma Sarcoma following Paget’s disease

Chemotherapy has no use with low-grade sarcomas:

Well-differentiated intramedullary osteosarcoma Parosteal osteosarcoma Low-grade intramedullary chondrosarcoma Adamantinoma Chordoma

193. External beam irradiation is an effective modality for all of the following malignancies except:(A) Ewing’s tumor(B) Lymphoma(C) Osteosarcoma(D) Multiple myeloma(E) Solitary myeloma of bone

Explanation:

Page 139: Hyperguides 2015 .Tumor

External beam irradiation can be effective for several malignancies such as:

Ewing’s tumor Lymphoma Myeloma Metastatic bone disease Soft tissue sarcomas

194. A 68-year-old woman presents with severe knee pain with ambulation for a 4-month period. Her history is significant for resection and postoperative external beam irradiation for a synovial sarcoma 15 years previously. The anteroposterior and lateral radiographs are shown in Slide 1. The coronal T1- and T2-weighted magnetic resonance images (MRIs) are shown in Slide 2. What is the most likely etiology of her pain:(A) Metastatic disease(B) Vascular insufficiency(C) Bone insufficiency(D) Osteoarthritis(E) Osteonecrosis

Explanation:

A number of late effects may occur following external beam irradiation for soft tissue sarcomas: postirradiation sarcomas, stress fractures, vascular insufficiency, chronic wound problems, and osteonecrosis.

This patient has a stress fracture of the proximal medial tibia. One can see the lucent area and the condensation of bone in a linear fashion.

The MRI shows very characteristic features:

T1-weighted coronal — low signal linear fracture line traversing the tibia T2-weighted coronal — high signal crossing the medial tibia with a low signal linear area

(condensation of bone seen in stress fractures)

Nonoperative treatment would be the initial management with protected weight bearing. These stress fractures may require a long time to heal and the nonunion rate is high with nonoperative and operative treatment. If the fracture does not heal in a 3- to 6-month period with nonoperative measures, then rigid internal fixation with or without bone grafting should be considered.

195. Which of the following is the most likely cytotoxic effect of external beam irradiation when treating malignancies:(A) Damage to microvasculature(B) Damage to large feeder vessels(C) Depletion of cellular nutrients(D) Creation of free radicals(E) Hyperthermic effect

Page 140: Hyperguides 2015 .Tumor

Explanation:

External beam irradiation creates free radicals. The free radicals damage the DNA. When the DNA cannot be repaired, cell death occurs.

The dose of irradiation is measured in grays or centigrays. A gray is a measure of the dose absorbed — 1 Gy equals 1 J/kg.One should remember the typical dose for delivered for certain tumors:

196. Which of the following genetic abnormalities occur in fibrodysplasia ossificans progressiva (FOP):(A) CBAF1/RUNX2(B) Activating mutation of FGFR3(C) Missense mutation ACVR1(D) Activating missense mutation GNAS (alpha subunit)(E) Collagen type X mutation

Explanation:

Fibrodysplasia ossificans progressiva is a rare heritable disorder with the following key features:

Recurrent episodes of soft tissue swelling that leads to heterotopic ossification Great toe malformations Incidence of 1 in 2,000,000 Autosomal dominant

Genetic findings:

Mutation — activin A type 1 receptor gene (ACVR1)o Missense mutation in glycine-serine (GS) activation domain

The other conditions refer to:

CBAF1/RUNX2 — cleidocranial dysplasia Activating mutation of FGFR3 — achondroplasia Activating missense mutation GNAS (alpha subunit) — fibrous dysplasia Collagen type X mutation — Schmid’s type metaphyseal chondrodysplasia

197. Which of the following gene fusion products occurs in patients with synovial sarcoma:

(A) EWS-FLI1(B) TLS (FUS) – CHOP(C) EWS-ATF1(D) SYT-SSX1(E) EWS-ERG

Explanation:

Page 141: Hyperguides 2015 .Tumor

Synovial sarcoma has a characteristic translocation – t(X;18). The gene fusion product is SYT-SSX1 and SYT-SSX2. The other responses address:

Ewings sarcoma     o   EWS-FLI1     o   EWS-ERG

Myxoid liposarcoma – TLS (FUS)-CHOP Clear cell sarcoma – EWS-ATF1

198. Which of the following gene fusion products occurs in patients with synovial sarcoma:

(A) EWS-FLI1(B) TLS (FUS) – CHOP(C) EWS-ATF1(D) SYT-SSX1(E) EWS-ERG

Explanation:

Synovial sarcoma has a characteristic translocation – t(X;18). The gene fusion product is SYT-SSX1 and SYT-SSX2.

The other responses address:

Ewings sarcoma

     o   EWS-FLI1     o   EWS-ERG

Myxoid liposarcoma – TLS (FUS)-CHOP Clear cell sarcoma – EWS-ATF1

199. Which of the following causes bone formation in metastatic prostate cancer:

(A) Parathyroid hormone-related protein (PTHrP)(B) Interleukin-8 (IL-8)(C) Endothelin-1(D) MIP-1α(E) Osteoprotegerin (OPG)

Explanation:

Page 142: Hyperguides 2015 .Tumor

Endothelin-1 is postulated to be responsible for the osteoblastic metastases in prostate cancer.

The other responses refer to:

PTHrP – Released by metastatic breast cancer cells and causes activation of the osteoclast via signaling to the osteoblast for release of the RANKL

OPG – A competitive decoy inhibitor that binds to the RANKL and prevents attachment to the RANK receptor on the osteoclast precursor cells; OPG prevents osteoclast activation and bone resorption

MIP-1α – Produced by multiple myeloma cells and causes osteoclast activation IL-8 – Causes direct activation of osteoclasts and bone resorption

200. Which of the following causes metastatic cells to localize to bone (homing in mechanism):

(A) NM-23(B) MKK4(C) RKIP(D) CXCL12(E) PTHrP

Explanation:

CXCL12 is called chemokine ligand 12 (stromal cell-derived factor 1). This factor is thought to be responsible for metastatic tumor cells to localize to bone. Chemostasis occurs as chemokine receptor 4 is activated on the tumor cells.

Tumor suppressor genes can inhibit the metastatic process:

NM-23 – phosphorylates the Ksr protein RKIP – competitive inhibitor of MEK MKK4 – causes stress-induced apoptosis

Parathyroid hormone-related protein (PTHrP) is released by breast cancer metastatic cells. The same receptors for parathyroid hormone (PTH) are activated.

PTH/PTHrP causes osteoclast activation and bone resorption.

201. Which of the following conditions has the highest risk of malignant change:

(A) Neurofibroma(B) Schwannoma(C) Plexiform neurofibroma(D) Myxoma within a nerve

Page 143: Hyperguides 2015 .Tumor

(E) Perineurioma

Explanation:

Patients with plexiform neurofibromas often have neurofibromatosis. The risk of malignant change is approximately 5% to 10%. Plexiform neurofibroma has the greatest chance of malignant degeneration.

202. Which of the following statements is true concerning osteoid osteomas:(A) Location in the vertebral body is common.(B) Location in flat bones is common.(C) The femoral neck is the single most common site.(D) In long bones, the lesions are most likely in the mid-diaphysis.(E) Craniofacial involvement is common.

Explanation:

Osteoid osteomas have some specific locations to remember:

The ends of long bones

Femoral neck (most common location)

Vertebra

Usually in lamina or base of the pedicle Virtually never in the vertebral body

Highly unlikely locations

Flat bones Craniofacial bones

203. Which of the following describes the radiographic features of an osteoid osteoma:(A) Ovoid, compact, and heavily mineralized intramedullary lesion with thorny spicules(B) Large, nodular, and heavily mineralized lesion on the surface of the bone(C) Surface lesion with spiculated bone formation(D) Well-demarcated nidus surrounded by a distinct zone of sclerosis(E) Cortically based lytic metaphyseal lesion with a sclerotic border

Explanation:

Page 144: Hyperguides 2015 .Tumor

Osteoid osteomas have a common radiographic appearance:

5 mm-10 mm lytic nidus surrounded by sclerosis

The other possible answers describe the radiographic features of other lesions:

Bone island: Ovoid, compact, and heavily mineralized intramedullary lesion with thorny spicules

Parosteal osteosarcoma: Large, nodular, heavily mineralized lesion on the surface of the bone

Periosteal osteosarcoma: Surface lesion with spiculated bone formation

Non-ossifying fibroma: Cortically based lytic metaphyseal lesion with a sclerotic border

204. A 17-year-old boy has a 4-month history of severe pain in his right distal tibia. The pain is especially prominent at night. He has difficulty sleeping and concentrating at school. The lateral radiograph of his right distal tibia and clinical photograph is shown in Figure 1. A computerized tomogram (CT) and axial T1 weighted magnetic resonance imaging (MRI) scan are shown in Figures 2 and 3. The most likely diagnosis is:

Slide 1 Slide 2 Slide 3

(A) Osteosarcoma(B) Osteomyelitis(C) Ewing's tumor(D) Osteod osteoma(E) Stress fracture

Explanation:

Osteoid osteomas are common tumors in young individuals. Patients usually present with intense pain and they can often localize the exact point of discomfort.

Page 145: Hyperguides 2015 .Tumor

Slide 4 Slide 5

CT findings are often quite characteristic (Figure 4):

small nidus 5 mm to 10 mm in the cortex or in a sub-periosteal location often there is central ossification in the nidus sclerosis surrounding the nidus

On MRI, it is often more difficult to see the nidus than on CT scanning. On T1 weighted images the nidus can often be seen as small round lesion (Figure 5). In the cortex, because the nidus often has mineralization which is low signal (as is the cortex which the nidus is sitting in), one may have trouble detecting the nidus.

The differential diagnosis of osteod osteomas include:

1. Stress fracture2. Osteomyelitis

Because of the characteristic imaging findings, one can make this diagnosis based upon the CT or MRI findings.

205. Which of the following cartilage tumors has the worst prognosis (highest risk of pulmonary metastases):(A) Chondrosarcoma arising in Ollier’s disease(B) Chondrosarcoma arising in Maffucci’s syndrome(C) Dedifferentiated chondrosarcoma(D) Grade 2 intramedullary chondrosarcoma(E) Grade 1 intramedullary chondrosarcoma

Explanation:

All of the possible answers (except one) are either low- or intermediate-grade bone sarcomas. Dedifferentiated chondrosarcoma is a high-grade spindle cell sarcoma. In dedifferentiated chondrosarcomas, there is a low-grade chondrosarcoma (grade 1 or 2) which is intimately admixed with a high-grade spindle sarcoma (grade 3 or 4 anaplastic spindle cell sarcoma -

Page 146: Hyperguides 2015 .Tumor

osteosarcoma, malignant fibrous histiocytoma, or fibrosarcoma).

The risk of pulmonary metastases in the lesions for all of the possible answers are as follows:

Tumor Approximate Risk

Chondrosarcoma arising in Ollier's disease

1% to 5%

Chondrosarcoma arising in Maffucci's syndrome

1% to 5%

Grade 2 intramedullary chondrosarcoma

20% to 30%

Grade 1 intramedullary chondrosarcoma

1% to 5%

Generally, low-, intermediate-, and high-grade sarcomas have the following risk of systemic metastases:

Grade Risk

Low (Grade 1) 5% to 10%

Intermediate (Grade 2) 20% to 30%

High (Grade 3,4) 50%+

206. Which of the following groups of soft tissue sarcomas commonly metastasize to lymph nodes:(A) Liposarcoma and malignant fibrous histiocytoma(B) Liposarcoma and clear cell sarcoma(C) Malignant fibrous histiocytoma and clear cell sarcoma(D) Synovial sarcoma and epithelioid sarcoma(E) Malignant fibrous histiocytoma and hemangioendothelioma

Explanation:

The four main soft tissue sarcomas that may metastasize to lymph nodes are epithelioid sarcoma, synovial sarcoma, rhabdomyosarcoma and clear cell sarcoma. It is uncommon for the other lesions to metastasize to lymph nodes.

207. Which of the following most commonly occurs in patients with pigmented villonodular synovitis of the knee:(A) Giving way with pivoting(B) Pain on ascending and descending steps(C) Recurrent non-traumatic effusions(D) Movie sign (pain following prolonged sitting)(E) Medial joint line pain

Page 147: Hyperguides 2015 .Tumor

Explanation:

Pigmented villonodular synovitis is a synovial proliferative disorder characterized by atraumatic recurrent effusions and cystic erosions of the periarticular bone surfaces. Patients commonly have multiple knee aspirations with blood-tinged fluid. The effusions are often large.

208. In patients with destructive lesions secondary to metastatic breast cancer, the cells responsible for the bone destruction are the:(A) Breast cancer cells(B) Fibroblasts(C) Osteoclasts(D) Histiocytes(E) Langerhans’ cells

Explanation:

Interestingly, in metastatic bone disease and multiple myeloma, it is not the tumor cells that destroy the bone, but the osteoclasts. The tumor cells secrete a factor that recruits and activates osteoclasts. These osteoclasts then resorb the bone, and the tumor cells grow and fill the holes in the bone.

209. Which of the following cell types is directly responsible for the resorption of bone in patients with metastatic breast carcinoma:(A) Breast carcinoma cells(B) Osteocytes(C) Osteoblasts(D) Osteoclasts(E) C cells of the parathyroid

glands

Explanation:

Bone resorption in metastatic breast cancer is directly caused by osteoclasts. The tumor cells secrete a factor that activates the osteoclasts, which then resorb the bone mineral.

Factors implicated in tumor mediated bone resorption include:

Parathyroid related protein (PTHrP) Interleukins 1 and 6 Tumor necrosis factor alpha

210. Which of the following agents is used to protect against hemorrhagic cystitis that occurs following chemotherapeutic treatment with ifosfamide or cyclophosphamide:(A) Leucovorin(B) Granulocyte colony-stimulating factor

Page 148: Hyperguides 2015 .Tumor

(C) 2-mercaptoethanesulfonate (MESNA)(D) Ondansetron (Zofran)(E) Amifostine (Ethoyl)

Explanation:

Hemorrhagic cystitis is a severe complication associated with ifosfamide and cyclophosphamide treatment. Both of these agents are used in regimens for osteosarcoma. These agents produce an acrolein metabolite that causes acute sterile hemorrhagic cystitis. 2-mercaptoethanesulfonate (MESNA) neutralizes this compound and prevents the cystitis.

The other possible answers are also used to ameliorate the side effects of chemotherapy:

Leucovorin (folic acid): Rescue cells following high dose methotrexate (folic acid analog that does not require reduction by the enzyme dihydrofolate reductase, bypasses effects of methotrexate)

Granulocyte-colony stimulating factor:

Reduces the duration of neutropenia following chemotherapy

Ondansetron (Zofran): Prevents nausea

Amifostine: Protects against cisplatin toxicity (mucositis, esophagitis)

211. Which of the following pairs of tumors are related as they share a chromosomal translocation between chromosome 11 and chromosome 22 (t [11;22]):(A) Chondrosarcoma and osteosarcoma(B) Malignant fibrous histiocytoma and fibrosarcoma(C) Adamantinoma and osteofibrous dysplasia(D) Ewing’s tumor and primitive neuroectodermal tumor(E) Osteoid osteoma and osteoblastoma

Explanation:

Ewing’s tumor and primitive neuroectodermal tumor are related as they share a chromosomal translocation between chromosome 11 and chromosome 22 (t [11;22]). A chimeric protein is produced that appears to act as transcriptional activator. This translocation is found in 95% of these tumors.

212. Metastases are common in patients with high-grade intramedullary osteosarcoma. The most common sites of metastases are:(A) Lungs and lymph nodes(B) Lungs and liver(C) Lungs and bone(D) Lungs and brain(E) Lungs and kidney

Page 149: Hyperguides 2015 .Tumor

Explanation:

Patients with high-grade intramedullary osteosarcoma have a high rate of metastases to the lungs (90%) and bone (20%). Approximately 10% to 20% of patients present with metastatic disease in the lungs or bone (Musculoskeletal Tumor Society Stage III).

Patients who have negative staging studies (normal technetium bone scan and CT scan of the chest) still have an incidence of 80% to 90% of occult metastases. In the pre-chemotherapy era, the survival in patients with osteosarcoma was only 10% to 20% because there was no method of killing the occult metastases. Now with multi-agent chemotherapy that is delivered pre- and postoperatively, we are able to save over 60% of these patients.

213. Which of the following features of soft tissue sarcomas portends a favorable prognosis:(A) Deep location (below the fascia)(B) Superficial location (above the fascia)(C) High-grade histology(D) Large size (>10 cm)(E) Lymph node involvement

Explanation:

The following features of soft tissue sarcomas are associated with a more favorable prognosis:

Small size (<5 cm) Low-grade Location above the fascia

Poor prognostic factors include:

Large size (>5 cm) High-grade Location below the fascia Presence of regional metastases

Patients who have large, deep high-grade sarcomas have a high risk (up to 40% to 60%) of pulmonary metastases.

214. Which of the following lesions is typically a Stage 1 lesion according to the staging system of the Musculoskeletal Tumor Society:

(A) Giant cell tumor(B) Chondroblastoma(C) Non-ossifying fibroma

Page 150: Hyperguides 2015 .Tumor

(D) Osteoblastoma(E) Osteoid osteoma

Explanation:

Stage 1 lesions are inactive, or latent, lesions. These lesions do not cause symptoms and generally do not grow. There is no risk of pathologic fracture or other problems.

Non-ossifying fibroma, or fibrous cortical defect, is a developmental process that is usually discovered as an incidental finding. The lesions are usually small and affect less than 50% of the diameter of the cortical bone. When these lesions are small (< 50% of the diameter of the bone) and asymptomatic, patients are not restricted in their activity. A new radiograph is obtained at 6 to 12 months to ensure that the lesion is not enlarging.

Some patients with non-ossifying fibromas have active lesions that grow and cause symptoms. Patients occasionally sustain fractures following minor trauma. In the scenarios that involve pain or fracture, the non-ossifying fibroma would be classified as a Stage 2 lesion.

215. Which of the following lesions is high signal on T1 weighted images and moderate signal on T2 weighted images:(A) Malignant fibrous histiocytoma(B) Extra-abdominal desmoid tumor(C) Synovial sarcoma(D) Lipoma(E) Ganglion

Explanation:

Lipomas are composed of mature adipose tissue. Mature adipose tissue is high signal on T1 weighted images and moderate on T2 weighted images.

It is important to remember the appearances of common tissues on both T1 and T2 weighted images:

T1 weighted T2 weighted

Fat High Moderate

Tendons Low Low

Ligaments Low Low

Fascial layers Low Low

Cortical bone Low Low

Muscle Moderate Moderate

Normal marrow High Moderate

Page 151: Hyperguides 2015 .Tumor

Soft tissue sarcomas Low High

Fluid (ganglions, effusions) Low High

Pigmented villonodular synovitis will often have signal drop out (dark black) on gradient echo sequences.

216. A 72-year-old woman presents with a 2-week history of increasing hip pain following a minor fall. All of her pain is activity related. Physical examination is normal with an excellent range of motion and no focal tenderness. The plain radiographs and T1- and T2-weighted coronal MRI scans are shown in Figures 1, 2 and 3 respectively. The high-signal areas correspond to:

Figure 1 Figure 2 Figure 3

(A) Normal marrow signal(B) Marrow with edema(C) Cortical bone(D) Articular cartilage(E) Neoplasm

Explanation:

The arrows point to high-signal areas that are normal marrow. T1-weighted coronal images are an excellent method to screen the bone marrow to look for lesions. Pathologic tissue on T1-weighted images usually show a low-signal. The low-signal abnormal areas are then sharply contrasted with the normal marrow that is high signal.

217. A 30-year-old attorney has a 6-month history of increasing knee pain. His pain occurs both with activity and at rest. He is otherwise healthy. Laboratory studies show a normal complete blood count, sedimentation rate, and chemistry values (including normal calcium and phosphorus levels). Anteroposterior and lateral radiographs are shown (Slide 1 and Slide 2). The low- and high-power hematoxylin and eosin sections are shown (Slide 3 and Slide 4).The diagnosis, based on the history, laboratory values, radiographs, and biopsy sections, most likely is:

Page 152: Hyperguides 2015 .Tumor

Slide 1 Slide 2 Slide 3 Slide 4

(A) Telangiectatic osteosarcoma(B) Osteomyelitis(C) Hyperparathyroidism(D) Giant cell tumor(E) Aneurysmal bone cyst

Explanation:

The radiograph shows purely lytic bone destruction that fills the lateral metaphysis and extends to the articular surface. The lateral cortex is very thin and almost completely destroyed. The histologic sections show collection of giant cells and mononuclear cells. Notice that the giant cells are uniformly scattered throughout the histologic section on the low power, in contrast to chondroblastoma where the giant cells are scattered throughout the mononuclear cells and that the nuclei of the mononuclear cells are the same as the nuclei of the giant cells.

218. A 60-year-old man presents with a 6-month history of increasing knee and distal thigh pain. He is otherwise healthy and has no history of prior malignancies. He has discomfort at night and at rest. The plain radiograph of the lesion is shown (Slide 1), and low and high power histologic

Page 153: Hyperguides 2015 .Tumor

sections from an open biopsy are shown (Slide 2 and Slide 3).The most likely diagnosis based on the history, plain radiographs, and biopsy material is:

Slide 1 Slide 2 Slide 3

(A) Bone infarct(B) Enchondroma(C) Lymphoma(D) Dedifferentiated chondrosarcoma(E) Metastatic bone disease

Explanation:

Dedifferentiated chondrosarcomas are high-grade cartilage tumors that are bimorphic (two distinct histologic components). Alongside the very low-grade cartilage tumor are areas of high-grade spindle cell sarcoma. The cartilage component is very low grade (grade ½, grade 1, or grade 2), while the high-grade component is an anaplastic spindle cell sarcoma; either osteosarcoma, fibrosarcoma, or malignant fibrous histiocytoma. The radiographs also reflect the histologic bimorphic component; area typical for low-grade chondrosarcoma with superimposed aggressive area. The aggressive component is usually represented by an area of:

Lytic bone destruction Cortical bone erosion or typically destruction Soft tissue mass

219. A 50-year-old man has a lesion in the left humerus that is discovered during a routine annual physical examination. The plain radiographs (Slide 1), the coronal T1- and T2-weighted images (Slides 2 and 3), and a biopsy (Slide 4) are presented. The correct stage of this lesion according to the Musculoskeletal Tumor Society is:

Page 154: Hyperguides 2015 .Tumor

Slide 1 Slide 2 Slide 3 Slide 4

(A) Stage I(B) Stage II(C) Stage III(D) Stage 1(E) Stage 2

Explanation:

The plain radiographs show a mineralized lesion in the medullary cavity. The mineralization shows stipples and small rings. This pattern is consistent with a cartilage lesion. There are no cortical changes indicating that the lesion is inactive. The magnetic resonance imaging scan shows a lesion with a low signal on the T1-weighted image and high signal on the T2-weighted image. There is no marrow edema bordering the lesion on the T2-weighted image. All of these features are compatible with the diagnosis of an enchondroma. Chondrosarcomas in long bones have easily recognized features: large cortical erosions, cortical destruction, cortical thickening, and a soft tissue mass.

Enchondromas are almost always inactive and asymptomatic. Therefore, enchondromas are Stage 1 inactive lesions.

Remember:

Stage 1 Latent, inactiveStage 2 ActiveStage 3 Aggressive

Stage I Low gradeStage II High gradeStage III Metastatic disease